Fundamentals of Nursing Final Exam

¡Supera tus tareas y exámenes ahora con Quizwiz!

4. Older adults frequently experience a change in sexual activity. Which best explains this change? 1. The need to touch and be touched is decreased. 2. The sexual preferences of older adults are not as diverse. 3. Medication side effects often impact sexual functioning. 4. Frequency and opportunities for sexual activity may decline

4. Answer: 4. As a result of loss of a loved one or a chronic illness in themselves or their partner, opportunities for sexual activity may decline. Aging does not change the need for touch, and older adults are diverse. Only select medications impact sexual functioning.

4. What factor results in vulnerable populations being more likely to develop health problems? 1. The ability to use available resources to find housing 2. Adequate transportation to the grocery store and community clinics 3. Availability of others to help provide care 4. Limited access to health care services

4. Answer: 4. It is the excess of risks and combination of risk factors that make this population more vulnerable, thus limiting access to health care services. These vulnerabilities can be associated with the individual's/community's social determinants of health or individual health disparities.

4. A patient is fearful of upcoming surgery and a possible cancer diagnosis. He discusses his love for the Bible with his nurse, who recommends a favorite Bible verse. Another nurse tells the patient's nurse that there is no place in nursing for spiritual caring. The patient's nurse replies: 1. "You're correct; spiritual care should be left to a pastoral care professional." 2. "You're correct; religion is a personal decision." 3. "Nurses should explain their own religious beliefs to patients." 4. "Spiritual, mind, and body connections can affect health."

4. Answer: 4. Spirituality offers a sense of connectedness: intrapersonally (connected with oneself), interpersonally (connected with others and the environment), and transpersonally (connected with the unseen, God, or a higher power).

5. Which comments to a patient by a new nurse regarding palliative care needs are correct? (Select all that apply.) 1. "Even though you're continuing treatment, palliative care is something we might want to talk about." 2. "Palliative care is appropriate for people with any diagnosis." 3. "Only people who are dying can receive palliative care." 4. "Children are able to receive palliative care." 5. Palliative care is only for people with uncontrolled pain

5. Answer: 1, 2, 4. Palliative care is available to all patients regardless of age, diagnosis, and prognosis.

3. A patient is receiving 40 mg of enoxaparin subcutaneously every 12 hours while on prolonged bed rest to prevent thrombophlebitis. Because bleeding is a potential side effect of this medication, the nurse should continually assess the patient for what signs of bleeding? (Select all that apply.) 1. Bruising 2. Pale yellow urine 3. Bleeding gums 4. Coffee ground-like vomitus 5. Light brown stool

3. Answer: 1, 3, 4. Because bleeding is a potential side effect of lowmolecular-weight heparin, continually assess the patient for signs of bleeding, such as hematuria, bruising, coffee ground-like vomitus or gastrointestinal aspirate, guaiac-positive stools, and bleeding gums.

5. Which of the following patients are at most risk for tachypnea? (Select all that apply.) 1. Patient just admitted with four rib fractures 2. Woman who is 9 months' pregnant 3. A patient admitted with hypothermia 4. Postoperative patient waking from general anesthesia 5. Three-pack-per-day smoker with pneumonia

5. Answer: 1, 2, 5. The patient with rib fractures is unlikely to breathe deeply, and a large fetus restricts diaphragmatic movement; both result in decreased ventilatory volume and increased respiratory rate. Pneumonia decreases gas exchange surface area; thus tachypnea occurs to increase minute ventilation. Hypothermia and general anesthesia depress respiratory rates

5. The effects of immobility on the cardiac system include which of the following? (Select all that apply.) 1. Thrombus formation 2. Increased cardiac workload 3. Weak peripheral pulses 4. Irregular heartbeat 5. Orthostatic hypotension

5. Answer: 1, 2, 5. The three major changes affecting the cardiac system are orthostatic hypotension, increased cardiac workload, and thrombus formation

6. A nurse is caring for a man who is recently retired and who appears withdrawn. He says he is "bored with life." The nurse helps this individual find meaning in life by: 1. Encouraging him to reflect on his relationships with others. 2. Encouraging relocation to a new city. 3. Explaining the need to simplify life. 4. Encouraging him to adopt a new pet.

6. Answer: 1. You can enhance feelings of integrity by encouraging older adults to reflect upon meaningful relationships, such as relationships with a higher power, family members, or the community

6. Match the following descriptions to the appropriate grand theorist 1. King 2. Henderson 3. Orem 4. Neuman a. Based on the theory that focuses on wellness and prevention of disease b. Based on the belief that people who participate in self-care activities are more likely to improve their health outcomes c. Based on 14 activities, the belief that the nurse should assist patients with meeting needs until they are able to do so independently d. Based on the belief that nurses should work with patients to develop goals for care

6. Answer: 1d, 2c, 3b, 4a.

6. The nurse is assessing the cranial nerves. Match the cranial nerve with its related function Cranial Nerves Cranial Nerve Function 1. XII Hypoglossal 2. V Trigeminal 3. VI Adducens 4. IV Trochlear 5. X Vagus a. Motor innervation to the muscles of the jaw b. Lateral movement of the eyeballs c. Sensation of the pharynx d. Downward, inward eye movements e. Position of the tongue

6. Answer: 1e, 2a, 3b, 4d, 5c

3. Which of the following statements correctly describes the evaluation process? (Select all that apply.) 1. Evaluation involves reflection on the approach to care. 2. Evaluation involves determination of the completion of a nursing intervention. 3. Evaluation involves making clinical decisions. 4. Evaluation requires the use of assessment skills. 5. Evaluation is performed only when a patient's condition changes.

3. Answer: 1, 3, 4. Evaluation is an ongoing process that involves reflection, clinical decision making, and the use of assessment skills to determine patient progress. It is not a determination of whether a nursing intervention was completed, but rather the effect of the intervention. An ongoing evaluation allows a nurse to determine whether a patient's condition has changed

6. What is the best response for the nurse to give if a patient asks the nurse to send a photo of an x-ray to him via a messaging tool in a social media site? 1. Yes, if you remove all patient identifiers before sending 2. No, because the patient's x-ray results should be discussed with a provider 3. Yes, because respect for autonomy means honoring this patient's request 4. No, because health information of any kind should not be shared on social media

6. Answer: 4. Patient information should not be shared over social media. While the patient does have a right to obtain health records, this is not the best mechanism by which to share the information. Even without specific identifiers, the information could be seen by others and attributed to the patient.

6. Place in the correct order the steps needed (below) to transfer a patient with sufficient lower body strength to a chair. 1. On count of three, instruct patient to stand while straightening hips and legs and keeping knees slightly flexed. 2. Assist patient to assume proper alignment in sitting position. 3. Help patient apply stable, nonskid shoes/socks. 4. Spread your feet apart. Flex hips and knees, aligning knees with patient's knees. 5. Apply gait/transfer belt. 6. Maintain patient's balance as you pivot foot farthest from chair and then help patient ease into chair. 7. Grasp transfer belt along patient's sides

6. Answer: 5, 3, 4, 7, 1, 6, 2

6. Which number marks the location where the nurse would auscultate the point of maximal impulse (PMI)? 1. 1 2. 2 3. 3 4. 4 5. 5 6. 6

6. Answer: 5. This is the spot where you would auscultate the PMI.

3. Which instruction should the nurse give the assistive personnel (AP) concerning a patient who has had an indwelling urinary catheter removed that day? 1. Limit oral fluid intake to avoid possible urinary incontinence. 2. Expect patient complaints of suprapubic fullness and discomfort. 3. Report the time and amount of first voiding. 4. Instruct patient to stay in bed and use a urinal or bedpan

3. Answer: 3. To adequately assess bladder function after a catheter is removed, voiding frequency and amount should be monitored. Unless contraindicated, fluids should be encouraged. To promote normal micturition, patients should be placed in as normal a posture for voiding as possible. Suprapubic tenderness and pain are possible indicators of urinary retention and/or a urinary tract infection.

6. The nurse is caring for a patient with an artificial airway. What are reasons to suction the patient? (Select all that apply.) 1. The patient has visible secretions in the airway. 2. There is a sawtooth pattern on the patient's EtCO2 monitor. 3. The patient has clear breath sounds. 4. It has been 3 hours since the patient was last suctioned. 5. The patient has excessive coughing.

6. Answers: 1, 2, and 5. Refer to Skill 41.1. Clear breath sounds are normal and do not indicate the need for suctioning. Suctioning should be based upon assessment findings and not performed on a time-oriented basis.

3. A young woman comes to a clinic for the first time for a gynecological examination. Which nursing behavior applies Swanson's caring process of "knowing" the patient? 1. Sharing feelings about the importance of having regular gynecological examinations 2. Explaining risk factors for cervical cancer 3. Recognizing that the patient is modest and maintaining her privacy during the examination 4. Asking the patient what it means to have a vaginal examination

3. Answer: 3. Understanding a patient's beliefs, values, and culture enables the nurse to know the context of a patient's illness, treatments, or screening. As a result, the nurse individualizes interventions to help the patient. Knowing the patient is essential when providing patient-centered care.

3. Which statement made by a patient who is recovering after recently experiencing third-degree burns shows connectedness? 1. "My pain medicine helps me feel better." 2. "I know I will get better if I just keep trying." 3. "I see God's grace and become relaxed when I watch the sun set at night." 4. "I feel so much closer to God after I read my Bible and pray."

3. Answer: 4. Connectedness is a dimension of spirituality that is related to the human need of belonging. Individuals can be connected to themselves, others, God or another Supreme Being, or nature. Individuals often stay connected to God through prayer.

3. A nurse has seen many cancer patients struggle with pain management because they are afraid of becoming addicted to the medicine. Pain control is a priority for cancer care. By helping patients focus on their values and beliefs about pain control, a nurse can best make clinical decisions. This is an example of: 1. Creativity. 2. Fairness. 3. Clinical reasoning. 4. Applying ethical criteria.

3. Answer: 4. Incorporating a patient's beliefs and values promotes patient autonomy and nurse advocacy, key ethical principles in clinical decision making.

3. A nurse ensures that each patient's room is clean; well ventilated; and free from clutter, excessive noise, and extremes in temperature. Which theorist's work is the nurse practicing in this example? 1. Henderson 2. Orem 3. King 4. Nightingale

3. Answer: 4. Nightingale's environmental theory directs the nurse to manipulate the environment to promote rest and healing

3. The nurse is caring for a client with dysphagia and is feeding her a pureed chicken diet when she begins to choke. What is the priority nursing intervention? 1. Suction her mouth and throat. 2. Turn her on her side. 3. Put on oxygen at 2 L nasal cannula. 4. Stop feeding her

3. Answer: 4. Stop feeding and then place patient on side. If choking persists, suction airway. Notify health care provider. Keep patient NPO

3. When assessing a young woman who was a victim of a home invasion 3 months earlier, the nurse learns that the woman has vivid images of the event whenever she hears loud yelling or a sudden noise. The nurse recognizes this as ____________.

3. Answer: Post-traumatic stress disorder (PTSD). PTSD originates with a person's experiencing or witnessing a traumatic event and responding with intense fear or helplessness. The home break-in is the traumatic event that is causing intense fear and/or flashbacks when the noises of the break-in are replicated.

4. Which statement made by a patient who is at average risk for colorectal cancer indicates an understanding about teaching related to early detection of colorectal cancer? 1. "I'll make sure to schedule my colonoscopy annually after the age of 60." 2. "I'll make sure to have a colonoscopy every 2 years." 3. "I'll make sure to have a flexible sigmoidoscopy every year once I turn 55." 4. "I'll make sure to have a fecal occult blood test annually once I turn 45

4. Answer: 4. American Cancer Society guidelines state that for people of average risk, beginning at the age of 45, anannual fecal occult blood test is recommended. Flexible sigmoidoscopy is recommended every 5 years in this population. A colonoscopy is used every 10 years if recommended by the health care provider.

7. The nurse applying effective communication skills throughout the nursing process should: (Place the following interventions in the correct order.) 1. Validate health care needs through verbal discussion with the patient. 2. Compare actual and expected patient care outcomes with the patient. 3. Provide support through therapeutic communication techniques. 4. Complete a nursing history using verbal communication techniques.

7. 4, 1, 3, 2. The correct order for the nurse to communicate with the patient is to first complete the history (part of assessment), then corroborate findings through a validation process. After this, the nurse would use therapeutic communication to address needs, and finally would complete an evaluation process to see whether the actual outcomes matched the expected outcome

7. Match the intervention for promoting child safety on the left with the correct developmental stage on the right. 1. Teach children proper bicycle and skate board safety. 2. Teach children how to cross streets and walk in parking lot. 3. Teach children proper techniques for specific sports. 4. Teach children not to operate electric toothbrushes while unsupervised. 5. Teach children not to talk to or go with a stranger. 6. Teach children not to eat items found in the grass. A. School-age child B. Preschooler

7. Answer A: 1, 2, 3 ; B: 4, 5, 6.

7. The nurse is teaching a patient how to perform a testicular self-examination. Which statement made by the patient indicates a need for further teaching? 1. "I'll recognize abnormal lumps because they are very painful." 2. "I'll start performing testicular self-examination monthly after I turn 15." 3. "I'll perform the self-examination in front of a mirror." 4. "I'll gently roll the testicle between my fingers."

7. Answer: 1 The examination should be performed monthly in all men 15 years of age and older. Feel for small, pea-size lumps on the front and side of the testicle. Abnormal lumps are usually painless

7. Before transferring a patient from the bed to a stretcher, which assessment data does the nurse need to gather? (Select all that apply.) 1. Patient's weight 2. Patient's activity tolerance 3. Patient's level of mobility 4. Recent laboratory values 5. Nutritional intake

7. Answer: 1, 2, 3. Before transferring a patient from bed to stretcher, assess height and weight and the patient's activity tolerance, noting for fatigue during previous sitting and standing. Assess the patient's mobility level, using the Banner Mobility Assessment Tool (BMAT) as an option. Nutritional intake and laboratory values will not influence a decision to transfer or the transfer technique.

7. When planning care for a dying patient, which interventions promote the patient's dignity? (Select all that apply.) 1. Providing respect 2. Viewing the patient as a whole 3. Providing symptom management 4. Showing interest 5. Being present 6. Inserting a straight catheter when the patient has difficulty voiding

7. Answer: 1, 2, 4, 5. A sense of dignity includes a person's positive self-regard, the ability to find meaning in life, to feel valued by others, and by how one is treated by caregivers

8. A patient with a malignant brain tumor requires oral care. The patient's level of consciousness has declined, with the patient only being able to respond to voice commands. Place the following steps in the correct order for administration of oral care. 1. If patient is uncooperative or having difficulty keeping mouth open, insert an oral airway. 2. Raise bed, lower side rail, and position patient close to side of bed with head of bed raised up to 30 degrees. 3. Using a brush moistened with chlorhexidine paste, clean chewing and inner tooth surfaces first. 4. For patients without teeth, use a toothette moistened in chlorhexidine rinse to clean oral cavity. 5. Remove partial plate or dentures if present. 6. Gently brush tongue, but avoid stimulating gag reflex

8. Answer: 2, 5, 1, 3, 6, 4

8. What are the physical circulatory changes that occur as death approaches? 1. Skin irritation 2. Mottling 3. Increased urine output 4. Weakness

8. Answer: 2. Patients experience circulatory changes resulting in mottling. Weakness, skin irritation, and incontinence are some of the physical changes that occur as death nears but are not related to circulatory changes

8. What should the nurse teach family caregivers when a patient has fecal incontinence because of cognitive impairment? 1. Cleanse the skin with antibacterial soap, and apply talcum powder to the buttocks. 2. Initiate bowel or habit training program to promote continence. 3. Help the patient to toilet once every hour. 4. Use sanitary pads in the patient's underwear.

8. Answer: 2. Patients who are cognitively impaired often forget how to respond to the urge to defecate and benefit from a structured program of bowel retraining

10. Match the cultural concepts on the left with the correct definitions on the right. 1. Etic world view __________ a. Factor that shapes how people perceive others and how they relate to reality 2. World view __________ b. Insider's perspective in an intercultural encounter 3. Cultural desire __________ c. A policy model that describes factors and power structures that shape and influence life 4. Intersectionality __________ d. An outsider's perspective in an intercultural encounter 5. Emic world view __________ e. The motivation of a health care professional to "want to" engage in cultural competency

9. Answer: 1. Teach-back is not a testing of patient knowledge or ability to use devices but is a confirmation of how well a nurse explains concepts to patients.

9. Which statement made by a new graduate nurse about the teachback technique requires intervention and further instruction by the nurse's preceptor? 1. "After teaching a patient how to use an inhaler, I need to use the teach-back technique to test my patient's technique." 2. "The teach-back technique is an ongoing process of asking patients for feedback." 3. "Using teach-back will help me identify explanations and communication strategies that my patients will most commonly understand." 4. "Using pictures, drawings, and models can enhance the effectiveness of the teach-back technique."

9. Answer: 1. Teach-back is not a testing of patient knowledge or ability to use devices but is a confirmation of how well a nurse explains concepts to patients.

9. A nurse is caring for a 40-year-old male diagnosed with Crohn's disease several years ago, resulting in numerous hospitalizations each year for the past 3 years. Which of the following behaviors interfere with the developmental tasks of middle adulthood? (Select all that apply.) 1. Sends birthday cards to friends and family 2. Refuses visitors while hospitalized 3. Self-absorbed in physical and psychological issues 4. Performs self-care activities 5. Communicates feelings of inadequacy

9. Answer: 2, 3, 5. Developmental tasks of adulthood can be impacted by chronic illness. Self-absorption and the refusal to stay connected with others are of concern to the nurse, as are verbalizations of inadequacy. Staying in touch with friends and performing self-care behaviors demonstrate developmental mastery of adulthood

9. A nurse used spiritual rituals as an intervention in a patient's care. Which of the following questions is most appropriate to evaluate its efficacy? 1. Do you feel the need to forgive your wife over your loss? 2. What can I do to help you feel more at peace? 3. Did either prayer or meditation prove helpful to you? 4. Should we plan on having your family try to visit you more often in the hospital?

9. Answer: 3. Rituals include participation in worship, prayer, sacraments (e.g., baptism, Holy Eucharist), fasting, singing, meditating, scripture reading, and making offerings or sacrifices. When you include the use of rituals in a patient's plan of care, evaluate whether the patient perceived these activities as useful. If not, other interventions will be necessary

9. A patient is admitted to the hospital with severe dyspnea and wheezing. Arterial blood gas levels on admission are pH 7.26; PaCO2, 55 mm Hg; PaO2, 68 mm Hg; and HCO3 - , 24. How does the nurse interpret these laboratory values? 1. Metabolic acidosis 2. Metabolic alkalosis 3. Respiratory acidosis 4. Respiratory alkalosis

9. Answer: 3. The pH is abnormally low, which indicates acidosis. The PaCO2 is high, which indicates respiratory acidosis. The HCO3 - is in the normal range, which indicates an acute respiratory acidosis that has not had time for renal compensation. The low PaO2 and the severe dyspnea and wheezing are consistent with this interpretation

9. Nurses in a community clinic are conducting an EBP project focused on improving the outcomes of children with asthma. The PICO question asked by the nurses is "In school-aged children, does the use of an electronic gaming education module versus educational book improve the usage of inhalers?" In the question, what is the "O"? 1. School-aged children 2. Educational book 3. Use of inhalers 4. Electronic gaming education

9. Answer: 3. The question is "In school-aged children, does the use of an electronic gaming education module versus educational book improve the usage of inhalers?" The population of interest is school-aged children. The "I" would be an electronic gaming education, the "C" would be educational book, and the "O" would be use of inhalers.

9. The nurse is reviewing health care provider orders that were handwritten on paper when all computers were down during a system upgrade. Which of the following orders contain an inappropriate abbreviation included on The Joint Commission's "Do Not Use" list and should be clarified with the health care provider? 1. Change open midline abdominal incision daily using wet-tomoist normal saline and gauze. 2. Lorazepam 0.5 mg PO every 4 hours prn anxiety 3. Morphine sulfate 1 mg IVP every 2 hours prn severe pain 4. Insulin aspart 8u SQ every morning before breakfast

9. Answer: 4. In option 4, the word "unit(s)" should be written out because the letter "u" can be mistaken for "0," the number "4," or "cc." The other orders are written appropriately

Chapter 4 1. The components of the nursing metaparadigm include: 1. Person, health, environment, and theory. 2. Health, theory, concepts, and environment. 3. Nurses, physicians, health, and patient needs. 4. Person, health, environment, and nursing.

. Answer: 4. Person, health, environment, and nursing are the four components that comprise the nursing metaparadigm

Chapter 28 1. A patient who has been placed on Contact Precautions for Clostridium difficile (C. difficile) asks you to explain what he should know about this organism. What is the most appropriate information to include in patient teaching? (Select all that apply.) 1. The organism is usually transmitted through the fecal-oral route. 2. Hands should always be cleaned with soap and water versus alcohol-based hand sanitizer. 3. Everyone coming into the room must be wearing a gown and gloves. 4. While the patient is in Contact Precautions, he cannot leave the room. 5. C. difficile dies quickly once outside the body

1. Answer: 1, 2, 3 Clostridium difficile is transmitted through the oral-fecal route and spread through contact with contaminated feces or surfaces touched by hands not appropriately cleaned after providing care to a patient infected with C. difficile. The organism develops a hard spore and can live for long periods of time on surfaces, making it very hard to eradicate. As long as patient is continent of stool and first cleans hands and changes gown, a patient with C. difficile may leave the room

Chapter 29 1. A 52-year-old woman is admitted with pneumonia, dyspnea, and discomfort in her left chest when taking deep breaths. She has smoked for 35 years and recently lost over 10 lb. She is started on intravenous antibiotics, high-protein shakes, and 2 L O2 via nasal cannula. Her most recent vital signs are HR 112, BP 138/82, RR 22, tympanic temperature 37.9°C (100.2° F), and oxygen saturation 94%. Which vital signs reflect a positive outcome of the treatment interventions? (Select all that apply.) 1. Temperature: 37° C (98.6° F) 2. Radial pulse: 98 3. Respiratory rate: 18 4. Oxygen saturation: 96% 5. Blood pressure: 134/78

1. Answer: 1, 2, 3, 4. Radial pulse has dropped as temperature has become within expected range. Respiratory rate has decreased with lower temperature, and oxygen saturation has improved with improved respiratory rate

Chapter 36 1. To best assist a patient in the grieving process, which factors are most important for the nurse to assess? (Select all that apply.) 1. Previous experiences with grief and loss 2. Religious affiliation and denomination 3. Ethnic background and cultural practices 4. Current financial status 5. Current medications

1. Answer: 1, 2, 3. Previous experiences, religious affiliation, and cultural practices help individuals develop coping and can be a source of support at the end of life

Chapter 12 1. Which of the following should be included in health teaching for a pregnant patient? (Select all that apply.) 1. Exposure of the fetus to alcohol, drugs, or tobacco can cause abnormal development. 2. Nutritional needs increase during pregnancy, and eating healthy foods is important. 3. Complementary and alternative therapies should always be avoided during pregnancy. 4. Provide education on self-care to reduce common discomforts of pregnancy, such as nausea. 5. Recommend birthing classes to prepare the mother for the birthing process

1. Answer: 1, 2, 4, 5. A woman's diet and exposure to alcohol, drugs, and tobacco can have a significant effect on fetal development. Providing anticipatory guidance and education regarding nutritional needs, self-care needs, and preparation for birth is critical for a pregnant patient. Complementary and alternative therapies should be evaluated for safety during pregnancy but do not always have to be avoided.

Chapter 16 1. A nurse completes the following steps during her shift of care. Which are the steps of nursing assessment? (Select all that apply.) 1. The review of patient data in the medical record 2. Confirming a patient's self-report of abdominal pain by inspecting the abdomen 3. Reporting results of an ongoing assessment to a nurse working the next scheduled shift 4. Analyzing a set of signs revealing lower leg weakness and unsteady gait with a pattern of mobility alteration 5. Conducting an interview of a family caregiver

1. Answer: 1, 2, 4, 5. Assessment involves the step of collection of information from a primary source (a patient) and secondary sources (e.g., family caregiver, family members or friends, health professionals, medical record). The second step of assessment involves the interpretation and validation of data to determine whether more data are needed or the database is complete. Reporting results is important for continuity of care but is not an assessment step for the nurse reporting. The nurse receiving the report will use the data in the report for assessment information.

Chapter 3 1. Using Healthy People 2020 as a guide, which of the following would improve delivery of care to a community? (Select all that apply.) 1. Community assessment 2. Implementation of public health policies 3. Home safety assessment 4. Increased access to care 5. Determining rates of specific illnesses

1. Answer: 1, 2, 4, 5. Improved delivery of health care occurs through assessment of health care needs of individuals, families, and communities; development and implementation of public health policies; and improved access to care. For example, assessment includes systematic data collection on the population, monitoring of the population's health status, gathering information on rates of particular diseases or infections, and accessing available information about the health of the community. Although option 3, home assessment, might be valuable to an individual patient, it may not benefit the community as a whole

Chapter 42 1. An intravenous (IV) fluid is infusing slower than ordered. The infusion pump is set correctly. Which factors could cause this slowing? (Select all that apply.) 1. Infiltration at vascular access device (VAD) site 2. Patient lying on tubing 3. Roller clamp wide open 4. Tubing kinked in bedrails 5. Circulatory overload

1. Answer: 1, 2, 4. Factors that could slow an IV infusion even if the infusion pump is set correctly include increased pressure at the outflow site (e.g., infiltration) and compression of the tubing lumen (e.g., patient lying on the tubing or tubing kinked in bedrails).

Chapter 24 1. When working with an older adult who is hearing-impaired, the use of which techniques would improve communication? (Select all that apply.) 1. Check for needed adaptive equipment. 2. Exaggerate lip movements to help the patient lip-read. 3. Give the patient time to respond to questions. 4. Keep communication short and to the point. 5. Communicate only through written information

1. Answer: 1, 3, 4. Communication techniques such as assessing the need for adaptive equipment, keeping communication short and direct, and giving the patient time to respond assist the nurse in providing clear, effective communication. Patients may have difficulty with rapid or lengthy explanations. Exaggerated lip movements may be difficult to interpret or demeaning to individuals with hearing deficits

Chapter 49 1. A patient has been on contact isolation for 4 days because of a hospital-acquired infection. He has had few visitors and few opportunities to leave his room. His ambulation is also still limited. Which are the correct nursing interventions to reduce sensory deprivation? (Select all that apply.) 1. Teaching how activities such as reading and using crossword puzzles provide stimulation 2. Moving him to a room away from the nurses' station 3. Turning on the lights and opening the room blinds 4. Sitting down, speaking, touching, and listening to his feelings and perceptions 5. Providing auditory stimulation for the patient by keeping the television on continuously

1. Answer: 1, 3, 4. Patients who are isolated in a health care setting are at risk for sensory deprivation because they are unable to enjoy normal interactions with others. To help them adjust to their environment, promote meaningful stimulation. You can do this best by sitting down, speaking, touching, and listening to his or her feelings and perceptions and teaching self-stimulation

5. Many older homes in a neighborhood are undergoing a lot of restoration. Lead paint was used to paint the homes when they were built. The community clinic in the neighborhood is initiating a lead screening program. This activity is based on which social determinant of health?

5. Answer: Physical environment. This social determinant of health describes where a person lives and how that environment affects the person's health.

Chapter 13 1. A nurse is completing an assessment on a 27-year-old female patient. Which questions best assess the psychosocial aspects of this young woman's health? (Select all that apply.) 1. Do you feel safe in your home and at work? 2. How many fruits and vegetables do you typically eat every day? 3. Describe your relationship with your family. 4. Have you had the vaccine to prevent HPV? 5. What are your long-term career goals?

1. Answer: 1, 3, 5. Young adults need physical and emotional resources and support systems to meet the many challenges, tasks, and responsibilities they face. Asking assessment questions such as feeling safe at home and at work, satisfaction with family relationships, and identifying long-term career goals provides information about the young adult's psychosocial health that supports successful maturation in this developmental stage. Assessment questions about the intake of fruits and vegetables and the HPV vaccine do not assess the patient's psychosocial health.

Chapter 18 1. Setting priorities for a patient's nursing diagnoses or health problems is an important step in planning patient care. Which of the following statements describe elements to consider in planning care? (Select all that apply.) 1. Priority setting establishes a preferential order for nursing interventions. 2. In most cases wellness problems take priority over problem-focused problems. 3. Recognition of symptom patterns helps in understanding when to plan interventions. 4. Longer-term chronic needs require priority over short-term problems. 5. Priority setting involves creating a list of care tasks.

1. Answer: 1, 3. Priority setting is the ordering of nursing diagnoses or patient problems to establish a preferential order for nursing interventions. Generally, actual needs and problems take priority over wellness, possible risk, and health promotion problems. Short-term acute patient care needs and problems typically take priority over longer-term chronic needs. Priority setting is not the ordering of a list of care tasks, but an organization of the desired outcomes for a patient. Symptom pattern recognition from your patient assessment and certain knowledge triggers help you understand which diagnoses require intervention and the associated time frame to intervene effectively

Chapter 50 1. The nurse prepares a patient with type 2 diabetes for a surgical procedure. The patient weighs 112.7 kg (248 lb) and is 5 feet, 2 inches in height. Which factors increase this patient's risk for surgical complications? (Select all that apply.) 1. Obesity 2. Prolonged bleeding time 3. Delayed wound healing 4. Ineffective vital capacity 5. Immobility secondary to height

1. Answer: 1, 3. Secondary to the physiological stress of surgery that increases cortisol levels in patients with type 2 diabetes, these patients are at risk for surgical complications. This patient is also obese, which increases surgical risk.

Chapter 46 1. A patient is scheduled to have an intravenous pyelogram (IVP) the next morning. Which nursing measures should be implemented before the test? (Select all that apply.) 1. Ask the patient about any allergies and reactions. 2. Instruct the patient that a full bladder is required for the test. 3. Instruct the patient to save all urine in a special container. 4. Ensure that informed consent has been obtained. 5. Instruct the patient that facial flushing can occur when the contrast media is given

1. Answer: 1, 4, 5. An IVP involves intravenous injection of an iodine-based contrast media. Patients who have had a previous hypersensitivity reaction to contrast media are at high risk for another reaction. Informed consent is required. The patient may experience facial flushing during injection of the contrast media. There is no need for a full bladder such as with a pelvic ultrasound or to save any urine for testing. There is no instrumentation of the urinary tract such as with a cystoscopy

Chapter 43 1. A nurse is developing a plan for a patient who was diagnosed with narcolepsy. Which interventions should the nurse include on the plan? (Select all that apply.) 1. Take brief, 20-minute naps no more than twice a day. 2. Drink a glass of wine with dinner. 3. Eat a large meal at lunch rather than dinner. 4. Establish a regular exercise program. 5. Teach the patient about the side effects of modafinil.

1. Answer: 1, 4, 5. Taking short naps, no longer than 20 minutes, during the day, and regular exercise are management strategies that help reduce the feeling of sleepiness. Modafinil is a stimulant used to treat narcolepsy; therefore, it is important for patients to understand its side effects.

Chapter 39 1. A patient has been on bed rest for over 5 days. Which of these findings during the nurse's assessment may indicate a complication of immobility? 1. Decreased peristalsis 2. Decreased heart rate 3. Increased blood pressure 4. Increased urinary output

1. Answer: 1. Immobility disrupts normal metabolic functioning: decreasing the metabolic rate; altering the metabolism of carbohydrates, fats, and proteins; causing fluid, electrolyte, and calcium imbalances; and causing gastrointestinal disturbances such as decreased appetite and slowing of peristalsis.

Chapter 35 1. The nurse is caring for a patient who has just had a near-death experience (NDE) following a cardiac arrest. Which intervention by the nurse best promotes the spiritual well-being of the patient after the NDE? 1. Allowing the patient to discuss the experience 2. Referring the patient to pastoral care 3. Having the patient talk to another patient who had an NDE 4. Offering to pray for the patient

1. Answer: 1. Patients who have a near death experience (NDE) are often reluctant to speak of the experience. Allowing the patient to discuss the NDE helps the patient find acceptance of and meaning from the event. It also allows the patient to explore what happened and promotes spiritual well-being.

Chapter 30 1. The nurse prepares to conduct a general survey on an adult patient. Which assessment is performed first while the nurse initiates the nurse-patient relationship? 1. Appearance and behavior 2. Measurement of vital signs 3. Observing specific body systems 4. Conducting a detailed health history

1. Answer: 1. The first part of the general survey is assessment of the appearance and behavior of the patient. As you are initiating the nurse-patient relationship, observe gender and race, age, signs of distress, body type, posture, gait, body movement, hygiene and grooming, dress, affect and mood, speech, and signs of patient abuse.

10. Which of the following scenarios demonstrate that learning has taken place? (Select all that apply.) 1. A patient listens to a nurse's review of the warning signs of a stroke. 2. A patient describes how to set up a pill organizer for newly ordered medicines. 3. A patient attends a spinal cord injury support group. 4. A patient demonstrates how to take his blood pressure at home. 5. A patient reviews written information about resources for cancer survivors.

10. Answer: 2, 4. Steps 2 and 4 are examples of patients exhibiting behaviors that demonstrate learning. The other three steps are examples of patient involvement in instruction

Chapter 22 1. The nurse is caring for a patient who needs a liver transplant to survive. This patient has been out of work for several months, does not have health insurance, and cannot afford the procedure. Which of the following statements speaks to the ethical elements of this case? 1. The health care team should select a plan that considers the principle of justice as it pertains to the distribution of health care resources. 2. The patient should enroll in a clinical trial of a new technology that can do the work of the liver, similar to the way dialysis treats kidney disease. 3. The social worker should look into enrolling the patient in Medicaid, since many states offer expanded coverage. 4. A family meeting should take place in which the details of the patient's poor prognosis are made clear to his family so that they can adopt a palliative approach.

1. Answer: 1. The principle of justice as it pertains to the distribution of health care resources is the ethical element present in option 1. Options 2, 3, and 4 are all potential strategies for assisting this patient, but they do not address the ethical elements of the case

Chapter 21 1. At 1200 the registered nurse (RN) says to the assistive personnel (AP), "You did a good job walking Mrs. Taylor by 0930. I saw that you recorded her pulse before and after the walk. I saw that Mrs. Taylor walked in the hallway barefoot. For safety, the next time you walk a patient, you need to make sure that the patient wears slippers or shoes. Please walk Mrs. Taylor again by 1500." Which characteristics of positive feedback did the RN use when talking to the AP? (Select all that apply.) 1. Feedback is given immediately. 2. Feedback focuses on one issue. 3. Feedback offers concrete details. 4. Feedback identifies ways to improve. 5. Feedback focuses on changeable things. 6. Feedback is specific about what is done incorrectly only

1. Answer: 2, 3, 4, 5. These are characteristics of good feedback. The nurse gives feedback on the process of the AP monitoring and ambulating a patient. The other options are not appropriate because the RN did not provide feedback immediately (the AP performed the task in the morning, but the feedback was not given until the afternoon), and you should give both positive feedback as well as feedback to improve the incorrectly done tasks.

Chapter 45 1. The nurse is caring for a client with pneumonia, who has severe malnutrition. The nurse should assess the patient for which of the following assessment findings? (Select all that apply.) 1. Heart disease 2. Sepsis 3. Hemorrhage 4. Skin breakdown 5. Diarrhea

1. Answer: 2, 3, 4. Patients who are malnourished on admission are at greater risk of life-threatening complications such as arrhythmia, skin breakdown, sepsis, or hemorrhage during hospitalization.

Chapter 14 1. A patient's family member is considering having her mother placed in a nursing center. The nurse has talked with the family before and knows that this is a difficult decision. Which of the following criteria does the nurse recommend in choosing a nursing center? (Select all that apply.) 1. The center needs to be clean, and rooms should look like a hospital room. 2. Adequate staffing is available on all shifts. 3. Social activities are available for all residents. 4. The center provides three meals daily with a set menu and serving schedule. 5. Staff encourage family involvement in care planning and assisting with physical care.

1. Answer: 2, 3, 5. Adequate staffing, provision of social activities, and active family involvement are essential. Meals should be high quality with options for what to eat and when it is served. A nursing center should be clean, but it should look like a person's home rather than a hospital

Chapter 20 1. A nurse admits a 32-year-old patient for treatment of acute asthma. The patient has labored breathing, a respiratory rate of 28 per minute, and lung sounds with bilateral wheezing. The nurse makes the patient comfortable and starts an ordered intravenous infusion to administer medication that will relax the patient's airways. The patient tells the nurse after the first medication infusion, "I feel as if I can breathe better." The nurse auscultates the patient's lungs and notes decreased wheezing with a respiratory rate of 22 per minute. Which of the following is an evaluative measure? (Select all that apply.) 1. Asking patient to breathe deeply during auscultation 2. Counting respirations per minute 3. Asking the patient to describe how his breathing feels 4. Starting the intravenous infusion 5. Auscultating lung sounds

1. Answer: 2, 3, 5. Measuring respirations by counting, asking the patient for self-report of breathing, and listening to lung sounds after medication is delivered are all evaluative measures. Asking a patient to breathe deeply is a technique used in an assessment skill. Starting the intravenous infusion is an intervention.

Chapter 19 1. A nurse is assigned to five patients, including one who was recently admitted and one returning from a diagnostic procedure. It is currently mealtime. The other three patients are stable, but one has just requested a pain medication. The nurse is working with an assistive personnel. Which of the following are appropriate delegation actions on the part of the nurse? (Select all that apply.) 1. The nurse directs the assistive personnel to obtain a set of vital signs on the patient returning from the diagnostic procedure. 2. The nurse directs the patient care technician to go to the patient in pain and to reposition and offer comfort measures until the nurse can bring an ordered analgesic to the patient. 3. The nurse directs the patient care technician to set up meal trays for patients. 4. The nurse directs the patient care technician to gather a history from the newly admitted patient about his medications. 5. The nurse directs the patient care technician to assist one of the stable patients up in a chair for his meal.

1. Answer: 2, 3, 5. The nurse can delegate repetitive, noninvasive tasks such as vital signs on a stable patient, assisting a mobile patient with ambulation, and setting up meal trays. It is inappropriate for the nurse to delegate aspects of the nursing process, such as collecting a medication history. The nurse also should not delegate vital signs if a patient might be unstable from returning from a diagnostic test

Chapter 40 1. What is the proper position to use for an unresponsive patient during oral care to prevent aspiration? (Select all that apply.) 1. Prone position 2. Sims' position 3. Semi-Fowler's position with head to side 4. Trendelenburg position 5. Supine position

1. Answer: 2, 3. Place the unconscious patient in semi-Fowler's position with head to the side or use the Sims' position to help avoid aspiration while performing oral care. The supine and Trendelenburg positions would make it easier for a patient to aspirate. The prone position would not be suitable for accessing the oral cavity.

9. Place the following steps for applying a wrist restraint in the correct order: 1. Pad the skin overlying the wrist. 2. Insert two fingers under the secured restraint to be sure that it is not too tight. 3. Be sure that the patient is comfortable and in correct anatomical alignment. 4. Secure restraint straps to bedframe with quick-release buckle. 5. Wrap limb restraint around wrist or ankle with soft part toward skin and secure snugly

9. Answer: 3, 1, 5, 2, 4

Chapter 15 1. A nurse enters a patient's room at the beginning of a shift to conduct an assessment of his condition following a blood transfusion. The nurse cared for the patient on the previous day as well. The patient has a number of issues he wishes to share with the nurse, who takes time to explore each issue. The nurse also assesses the patient and finds no signs or symptoms of a reaction to the blood product. The nurse observed the patient the prior day and sees a change in his behavior, a reluctance to get out of bed and ambulate. Which of the following actions improve the nurse's ability to make clinical decisions about this patient? (Select all that apply.) 1. Working the same shift each day 2. Spending time during the patient assessment 3. Knowing the early mobility protocol guidelines 4. Caring for the patient on consecutive days. 5. Knowing the pattern of patient behavior about ambulation

1. Answer: 2, 4, 5. Clinical decision making is enhanced by spending enough time during patient assessments and knowing a patient's typical behaviors (which is reinforced when a nurse is assigned to a patient on consecutive days). Working the same shift does not mean the nurse will care for the same patient. Knowledge about the mobility protocol is helpful, but clinical decisions center on the needs of an individual patient (e.g., whether a patient meets the criteria for a mobility protocol and how well he or she has advanced so far)

Chapter 27 1. Which of the following are safe practices to follow in the safe preparation and storage of food? (Select all that apply.) 1. Always use a single cutting board to prepare foods for cooking. 2. Refrigerate leftovers as soon as possible. 3. Always buy vegetables in packages marked "prewashed." 4. Cook meats to the proper temperature. 5. Wash hands thoroughly before food preparation.

1. Answer: 2, 4, 5. The Centers for Disease Control and Prevention (CDC) recommends washing hands thoroughly before food preparation and to wash cooking surfaces often. Keep raw meat, poultry, seafood, and their juices away from other foods, and use separate cutting boards for each. Rinse fruits and vegetables thoroughly, and always cook food to the proper temperature. Refrigerate leftovers promptly. A single cutting board can cause cross contamination. Even if packages show that vegetables have been prewashed, thoroughly wash when opening a package.

Chapter 47 1. Which nursing actions do you take when placing a bedpan under an immobilized patient? (Select all that apply.) 1. Lift the patient's hips off the bed and slide the bedpan under the patient. 2. After positioning the patient on the bedpan, elevate the head of the bed to a 45-degree angle. 3. Adjust the head of the bed so that it is lower than the feet, and use gentle but firm pressure to push the bedpan under the patient. 4. Have the patient stand beside the bed, and then have him or her sit on the bedpan on the edge of the bed. 5. Make sure the patient has a nurse call system in reach to notify the nurse when he or she is ready to have the bedpan removed.

1. Answer: 2, 5. Elevating the head of the bed allows the patient the most normal and comfortable position for defecation on a bedpan. Sitting on a bedpan for a prolonged time is uncomfortable and exerts pressure on the ischial bony prominences, so it is important for the patient to have privacy but to be able to let the nurse know when he or she is finished using the bedpan

Chapter 17 1. A nursing student is working with a faculty member to identify a nursing diagnosis for an assigned patient. The student has assessed that the patient is undergoing radiation treatment, has liquid stool, and the skin is clean and intact. The student selects the nursing diagnosis Impaired Skin Integrity. The faculty member explains that the student has made a diagnostic error for which of the following reasons? 1. Incorrect clustering of data 2. Wrong diagnosis 3. Condition is a collaborative problem 4. Premature ending assessment

1. Answer: 2. A standard for the nursing diagnosis of Impaired Skin Integrity is the actual alteration in skin integrity, not the skin being clean and intact. The student needs to review data and compare more closely with the standard assessment findings for a correct diagnosis

Chapter 26 1. The nurse contacts a provider about a change in a patient's condition and receives several new orders for the patient over the phone. When documenting telephone orders in the electronic health record, most hospitals require a nurse to do which of the following? 1. Print out a copy of all telephone orders entered into the electronic health record in order to keep them in personal records for legal purposes. 2. "Read back" all telephone orders to the provider over the phone to verify all orders were heard, understood, and transcribed correctly before entering the orders in the electronic health record. 3. Record telephone orders in the electronic health record, but wait to implement the order(s) until they are electronically signed by the health care provider who gave them. 4. Implement telephone order(s) immediately, but insist that the health care provider come to the patient care unit to personally enter the order(s) into the electronic health record within the next 24 hours

1. Answer: 2. Guidelines from TJC require a "read-back" on all telephone (and verbal) orders. The nurse reads a telephone order back word for word and receives confirmation that the order is correct from the health care provider who gave the order.

Chapter 44 1. Which of the following signs or symptoms in a patient who is opioid-naïve is of greatest concern to the nurse when assessing the patient 1 hour after administering an opioid? 1. Oxygen saturation of 95% 2. Difficulty arousing the patient 3. Respiratory rate of 12 breaths/min 4. Pain intensity rating of 5 on a scale of 0 to 10

1. Answer: 2. Sedation is a concern because it may indicate that the patient is experiencing opioid-related side effects. Advancing sedation may indicate that the patient may progress to respiratory depression.

Chapter 23 1. A nurse is planning care for a patient going to surgery. Who is responsible for informing the patient about the surgery along with possible risks, complications, and benefits? 1. Family member 2. Surgeon 3. Nurse 4. Nurse manager

1. Answer: 2. The person performing the procedure is responsible for informing the patient about the procedure and its risks, benefits, and possible complications

Chapter 31 1. It is important to take precautions to prevent medication errors. A nurse is administering an oral tablet to a patient. Which of the following steps is the second check for accuracy in determining the patient is receiving the right medication? 1. Logging on to automated dispensing system (ADS) or unlocking medicine drawer or cart. 2. Before going to patient's room, comparing patient's name and name of medication on label of prepared drugs with MAR. 3. Selecting correct medication from ADS, unit-dose drawer, or stock supply and comparing name of medication on label with MAR or computer printout. 4. Comparing MAR or computer printout with names of medications on medication labels and patient name at patient's bedside.

1. Answer: 2. This is the second check for accuracy

Chapter 37 1. The nurse is interviewing a patient in the community clinic and gathers the following information about her: she is intermittently homeless, a single parent with two children who have developmental delays. She has had asthma since she was a teenager. She does not laugh or smile, does not volunteer any information, and at times appears close to tears. She has no support system and does not work. She is experiencing an allostatic load. As a result, which of the following would be present during complete patient assessment? (Select all that apply.) 1. Post-traumatic stress disorder 2. Rising hormone levels 3. Chronic illness 4. Insomnia 5. Depression

1. Answer: 3, 4, 5. An increased allopathic load can result in longterm physiological and psychological problems such as chronic illness, depression, sleep deprivation, chronic fatigue syndrome, and autoimmune disorders. Post-traumatic stress disorder results from a single traumatic event. Hormone levels rise in the alarm stage.

Chapter 38 1. A nurse is instructing a patient who has decreased leg strength on the left side on how to use a cane. Which actions indicate proper cane use by the patient? (Select all that apply.) 1. The patient keeps the cane on the left side of the body. 2. The patient slightly leans to one side while walking. 3. The patient keeps two points of support on the floor at all times. 4. After the patient places the cane forward, he or she then moves the right leg forward to the cane. 5. The patient places the cane forward 15 to 25 cm (6 to 10 inches) with each step

1. Answer: 3, 5. Have a patient keep the cane on the stronger side of the body. For maximum support when walking, the patient places the cane forward 15 to 25 cm (6 to 10 inches), keeping body weight on both legs. The patient needs to learn that two points of support such as both feet or one foot and the cane are on the floor at all times. The patient moves the weaker leg forward to the cane, so body weight is divided between the cane and the stronger leg.

Chapter 34 1. A 16-year-old female tells the school nurse that she doesn't need the human papillomavirus (HPV) vaccine since her partner always uses condoms. The best response by the nurse to this statement is: 1. "Latex condoms are the most effective way to eliminate the risk of HPV transmission." 2. "Your parents may not want you to receive the HPV vaccine since it has been shown to increase sexual risk taking and sexual activity." 3. "The HPV 9-valent vaccine is recommended for males and females even if they use condoms because it targets the specific viruses that cause cancer and genital warts."

1. Answer: 3. An HPV vaccine that protects both men and women against the types of HPV that cause serious health issues is available and recommended for individuals ages 11 to 26. The use of latex condoms reduces the risk of contracting a sexually transmitted infection (STI), but abstinence is the only practice that eliminates the risk. Longitudinal research indicates that vaccination does not increase sexual risk-taking behaviors among youths and is safe.

Chapter 8 1. A health system upgraded its electronic health record across all its practice settings to enhance patient care and communication among health care providers. This is an example of which component of the Chronic Care Model? 1. Health systems 2. Decision support 3. Clinical information systems 4. Community

1. Answer: 3. Clinical information systems maintain and share patient health information among providers and patients to ensure effective communication and quality patient care

Chapter 10 1. A family includes a mother, a stepfather, two teenage biological daughters of the mother, and a biological daughter of the father. The father's daughter just moved home following the loss of her job in another city. The family is converting a study into a bedroom and is in the process of distributing household chores. Nursing assessment reveals all members of the family think that their family can adjust to lifestyle changes. This is an example of family: 1. Diversity. 2. Durability. 3. Resiliency. 4. Configuration

1. Answer: 3. Resiliency is the ability of the family to cope with the unexpected. This family used resources to provide some short-term solutions for the return home of an adult child.

Chapter 6 1. A patient discharged a week ago following a stroke is currently participating in rehabilitation sessions provided by nurses, physical therapists, and registered dietitians in an outpatient setting. In what level of prevention is the patient participating? 1. Primary prevention 2. Secondary prevention 3. Tertiary prevention 4. Transtheoretical prevention

1. Answer: 3. Tertiary prevention involves minimizing the effects of long-term disease or disability by interventions directed at preventing complications and deterioration following surgery. Tertiary prevention activities are directed at rehabilitation rather than diagnosis and treatment. Care at this level aims to help patients achieve as high a level of functioning as possible, despite the limitations caused by illness or impairment. This level of care is called preventive care because it involves preventing further disability or reduced functioning.

Chapter 32 1. When planning patient education, it is important to remember that patients with which of the following illnesses often find relief in complementary therapies? 1. Lupus and diabetes 2. Ulcers and hepatitis 3. Heart disease and pancreatitis 4. Chronic back pain and arthritis

1. Answer: 4. Evidence supports the use of many complementary therapies for chronic pain syndromes, particularly pain that is unremitting and unresponsive to conventional allopathic therapies

Chapter 11 1. The nurse is aware that preschoolers often display a developmental characteristic that makes them treat dolls or stuffed animals as if they have thoughts and feelings. This is an example of: 1. Logical reasoning. 2. Egocentrism. 3. Concrete thinking. 4. Animism

1. Answer: 4. This is the belief that inanimate objects have lifelike qualities; it is a component of magical thinking evident in preoperational thought

Chapter 7 1. An experienced nurse is explaining the use of touch from a caring perspective. What information does the nurse include in the discussion with the student about touch? 1. Nurses touch patients only while performing procedures or doing assessments. 2. Touch is a type of verbal communication. 3. Nurses use touch only when a patient is in pain. 4. Touch forms a connection between nurse and patient

1. Answer: 4. Touch is relational and leads to a connection between nurse and patient. It involves contact and noncontact touch. Contact touch involves obvious skin-to-skin contact, whereas noncontact touch refers to eye contact

Chapter 48 1. When repositioning an immobile patient, the nurse notices redness over the hip bone. What is indicated when a reddened area blanches on fingertip touch? 1. A local skin infection requiring antibiotics 2. Sensitive skin that requires special bed linen 3. A stage 3 pressure injury needing the appropriate dressing 4. Blanching hyperemia, indicating the attempt by the body to overcome the ischemic episode

1. Answer: 4. When repositioning an immobile patient, it is important to assess all bony prominences for the presence of redness, which can be the first sign of impaired skin integrity. Pressing over the area compresses the blood vessels in the area; if the integrity of the vessels is good, the area turns lighter in color and then returns to the red color. However, if the area does not blanch when pressure is applied, tissue damage is likel

Chapter 33 1. A 50-year-old woman is recovering from a bilateral mastectomy. She refuses to eat, discourages visitors, and pays little attention to her appearance. One morning the nurse enters the room to see the patient with her hair combed and makeup applied. Which of the following is the best response from the nurse? 1. "What's the special occasion?" 2. "You must be feeling better today." 3. "This is the first time I've seen you look this good." 4. "I see that you've combed your hair and put on makeup."

1. Answer: 4. When the nurse uses a matter-of-fact approach and acknowledges a change in the patient's behavior or appearance, it allows the patient to establish its meaning. Telling the patient she has never looked this good conveys criticism; making assumptions about it being a special occasion or about an obvious improvement in mood superimposes the nurse's opinion and limits the assessment

Chapter 41 1. The nurse is preparing to perform nasotracheal suctioning on a patient. Arrange the steps in order. 1. Apply suction. 2. Assist patient to semi-Fowler's or high Fowler's position, if able. 3. Advance catheter through nares and into trachea. 4. Have patient take deep breaths. 5. Lubricate catheter with water-soluble lubricant. 6. Apply sterile gloves. 7. Perform hand hygiene. 8. Withdraw catheter

1. Answer: 7, 2, 6, 4, 5, 3, 1, 8. Refer to Skill 41.1 for the steps to this procedure.

Chapter 5 1. Match the components of PICO using the question "Does the use of guided imagery compared with standard care decrease the postoperative pain in hospitalized adolescents?" _____ (P) Patient/population A. Adolescents receiving standard care _____ (I) Intervention B. Decreased postoperative pain _____ (C) Comparison C. Hospitalized adolescents _____ (O) Outcome D. Guided imagery

1. Answer: C, D, A, B. This question includes the four PICO elements of patient/population (hospitalized adolescents who had surgery), intervention (guided imagery), comparison group (adolescents receiving standard care), and outcome (decreased postoperative pain)

Chapter 9 1. Which of the following is an example of a patient with a health disparity? (Select all that apply.) 1. A patient who has a homosexual sexual preference 2. A patient unable to access primary care services 3. A patient living with a chronic disease 4. A family who relies on public transportation 5. A patient who has had a history of smoking for 10 years

1. Answers: 2, 3, 5. Poor health status (chronic disease), disease risk factors (smoking history), poor health outcomes, and limited access to health care (unable to access primary care) are types of health disparities.

10. A middle-aged adult patient has limited mobility following a total knee arthroplasty. During assessment, the nurse notes that the patient is having difficulty breathing while lying supine. Which assessment data support a pulmonary issue related to immobility? (Select all that apply.) 1. Oxygen saturation of 89% 2. Irregular radial pulse 3. Diminished breath sounds bilateral bases on auscultation 4. BP: 132/84 5. Pain reported at 3 on scale of 0 to 10 following medication 6. Respiratory rate of 26

10. Answer 1, 3, and 6. Patients who are immobile are at high risk of developing pulmonary complications. Pooling of sections secondary to atelectasis can cause decreased lung sounds. Tachypnea is a common response to dyspnea. Atelectasis can also negatively impact oxygen saturation as exhibited by the 89% oxygen saturation on this patient.

10. Family members have asked for a meeting with the nursing staff of an assisted-living residential center to discuss the feasibility of their mother using a walker. The family is worried that her health is declining; they wonder whether she can use the walker safely. Which of the following instructions should the nurse give the family after assessing that it is safe for the woman to use a walker? (Select all that apply.) 1. A walker is useful for patients who have impaired balance. 2. The patient uses a walker by pushing the device forward. 3. Leaning over the walker improves the patient's balance. 4. Walkers should not be used on stairs. 5. If the patient has difficulty advancing the walker, a walker with wheels is an option

10. Answer 1, 4, 5. A walker can be used by a patient who is weak or has problems with balance. Walkers with wheels are useful for patients who have difficulty lifting and advancing the walker as they walk because of limited balance or endurance. A patient uses a walker correctly by holding the handgrips on the upper bars, taking a step, moving the walker forward, and taking another step. A walker requires a patient to lift the device up and forward. The patient should not lean over the walker or walk behind it; otherwise he or she might lose balance and fall. Walkers should not be used on stairs

10. Which of the following statements correctly describe the evaluation process? (Select all that apply.) 1. Evaluation is an ongoing process. 2. Evaluation involves the gathering of data for recognizing errors or omissions in care. 3. Evaluation involves making clinical decisions. 4. Evaluation requires the use of assessment skills. 5. Evaluation is done only when a patient's condition changes.

10. Answer: 1, 2, 3, 4. Evaluation is ongoing and reveals whether outcomes are achieved, which includes examining in what way a patient's condition changes. Because evaluation is ongoing, never use evaluation only when a patient condition changes.

10. Which sleep-hygiene actions at bedtime can the nurse delegate to assistive personnel? (Select all that apply.) 1. Giving the patient a back rub 2. Turning on quiet music 3. Dimming the lights in the patient's room 4. Giving a patient a cup of coffee 5. Monitoring for the effect of the sleeping medication that was given

10. Answer: 1, 2, 3. Giving the patient a back rub, turning on quiet music, and dimming the lights are all appropriate sleep-hygiene measures. These activities are within the scope of practice for assistive personnel. Coffee, tea, cola, and chocolate act as stimulants, causing a person to stay awake or awaken throughout the night and should not be ingested before bedtime. Monitoring medication effect is a registered nurse activity

10. The public health nurse is working with the county health department on a task force to fully integrate the goals of Healthy People 2020. Most of the immigrant population do not have a primary care provider, nor do they participate in health promotion activities; the unemployment rate in the community is 25%. How does the nurse determine which goals need to be included or updated? (Select all that apply.) 1. Assess the health care resources within the community. 2. Assess the existing health care programs offered by the county health department. 3. Compare existing resources and programs with Healthy People 2020 goals. 4. Initiate new programs to meet Healthy People 2020 goals. 5. Implement educational sessions in the schools to focus on nutritional needs of the children

10. Answer: 1, 2, 3. The nurse must first assess for existing health care resources and educational programs. Then the nurse must compare these resources and programs with Healthy People 2020 goals. These processes determine whether any goals need to be added or updated.

10. The nurse manager of a community clinic arranges for staff in-services about various complementary therapies available in the community. What is the purpose of this training? (Select all that apply.) 1. Nurses play an essential role in the safe use of complementary therapies. 2. Nurses are often asked for recommendations and strategies that promote well-being and quality of life. 3. Nurses learn how to provide all of the complementary modalities during their basic education. 4. Nurses play an essential role in patient education to provide information about the safe use of these healing strategies. 5. Nurses appreciate the cultural aspects of care and recognize that many of these complementary strategies are part of a patient's life.

10. Answer: 1, 2, 4, 5. All of the statements are true except that nurses do not learn how to provide all of the complementary modalities during their basic education. Nurses play an essential role in the safe use of complementary therapies in our emerging health care system. They have an appreciation for many types of interventions and can understand the patient's need to become more involved in his or her health care decisions and choices. They also understand the patient's desire to take a more active role in his or her healing and health promotion processes. Culturally relevant care that uses a full complement of intervention strategies that are supported with evidence is a central tenet of contemporary nursing practice.

10. A patient who returned from surgery 3 hours ago following a kidney transplant is reporting pain at a 7 on a scale of 0 to 10. The nurse has tried repositioning with no improvement in the patient's pain report. Unmanaged surgical pain can lead to which of the following problems? (Select all that apply.) 1. Delayed ambulation 2. Reduced ventilation 3. Catheter-associated urinary tract infection 4. Retained pulmonary secretions 5. Reduced appetite

10. Answer: 1, 2, 4, 5. Unmanaged surgical pain can lead to delayed ambulation, reduced ventilation, retained pulmonary secretions, or reduced appetite. Unmanaged surgical pain is not associated with catheter-associated urinary tract infection

10. A nurse is conferring with another nurse about the care of a patient with a stage II pressure injury. The two decide to review the clinical practice guideline of the hospital for pressure injury care. The use of a clinical practice guideline achieves which of the following? (Select all that apply.) 1. Allows nurses to act more quickly and appropriately 2. Sets a level of clinical excellence for practice 3. Eliminates need to create an individualized care plan for the patient 4. Incorporates evidence-based interventions for stage II pressure injury 5. Provides for access to patient care information within the electronic health record

10. Answer: 1, 2, 4. A clinical practice guideline is a systematically developed set of statements about appropriate health care for specific health care problems or clinical situations. Evidence-based research provides the basis for sound clinical practice guidelines. A nurse individualizes how to apply nursing interventions for each unique patient. Standard interventions are developed for the more common health problems; thus standard interventions assist nurses to intervene more quickly and appropriately. An individualized plan of care is always necessary. The use of standard interventions aids in capturing sharable patient and care information within the electronic medical record.

10. Which of the following describes a nurse's application of a specific knowledge base during critical thinking? (Select all that apply.) 1. Initiative in reading current evidence from the literature 2. Application of nursing theory 3. Reviewing a policy and procedure manual 4. Considering a colleague's view of a patient's needs 5. Previous time caring for a specific group of patients

10. Answer: 1, 2. A nurse's specific knowledge base varies according to educational experience that includes basic nursing education, continuing education courses, and additional college degrees. A nurse also builds knowledge by reading the nursing literature (especially research-based literature) to maintain current knowledge of nursing science and theory.

10. Which actions by the nurse help grieving families? (Select all that apply.) 1. Encourage involvement in nonthreatening group social activities. 2. Follow up with the family in their home. 3. Remind them that feelings of sadness or pain can return around anniversaries. 4. Encourage survivors to ask for help. 5. Look for overuse of alcohol, sleeping aids, or street drugs

10. Answer: 1, 3, 4, 5. Providing education, encouraging, and monitoring for healthy and unhealthy coping responses during grief are ways to support and help families grieve. Encouraging survivors to seek available resources helps survivors cope with grief

10. After receiving an intramuscular (IM) injection in the deltoid, a patient states, "My arm really hurts. It's burning and tingling where I got my injection." What should the nurse do next? (Select all that apply.) 1. Assess the injection site. 2. Administer an oral medication for pain. 3. Notify the patient's health care provider of assessment findings. 4. Document assessment findings and related interventions in the patient's medical record. 5. This is a normal finding, so nothing needs to be done. 6. Apply ice to the site for relief of burning pain.

10. Answer: 1, 3, 4. Assessing the injection site may reveal a site reaction or induration from the injection. The health care provider needs to be notified in case there is an adverse effect from the injection. The nurse must always document adverse effects so that the site and patient can be monitored.

10. During a visit to a family clinic, a nurse teaches a mother about immunizations, the use of car seats, and home safety for an infant and toddler. Which type of nursing interventions are these? 1. Restorative 2. Health promotion 3. Acute care 4. Growth and development

10. Answer: 2. Health promotion activities focus on interventions designed to maintain the physical, social, emotional, and spiritual health of the family unit, including information about specific health behaviors and family coping techniques.

10. A crisis intervention nurse is working with a mother whose child with Down syndrome has been hospitalized with pneumonia and who has lost her child's disability payment while the child is hospitalized. The mother worries that her daughter will fall behind in her classes during hospitalization. Which strategies are effective in helping this mother cope with these stressors? (Select all that apply.) 1. Referral to social service process reestablishing the child's disability payment 2. Sending the child home in 72 hours and having the child return to school 3. Coordinating hospital-based and home-based schooling with the child's teacher 4. Teaching the mother signs and symptoms of a respiratory tract infection 5. Telling the mother that the stress will decrease in 6 weeks when everything is back to normal

10. Answer: 1, 3, 4. The stressors for this parent are her child's illness, missing school, and loss of disability payments. Obtaining resources to resolve these stressors will reduce the mother's stress load and allow her to focus on helping her child improve and on preventing another respiratory tract infection. Discharging the child in 72 hours with a return to school may not be best for the child's physical condition and may make the situation worse. Giving the mother a 6-week time frame is unrealistic because everyone's time frame is different. The mother may also need to adjust.

10. Which of the following approaches are recommended when gathering assessment data from an 82-year-old male patient entering a primary care clinic for the first time? (Select all that apply.) 1. Recognize normal changes associated with aging. 2. Avoid direct eye contact. 3. Lean forward and smile as you pose questions. 4. Allow for pauses as patient tells his story. 5. Use the list of questions from the clinic assessment form to complete all data.

10. Answer: 1, 3, 4. When assessing older adults, listen patiently and allow for pauses and time for patients to tell their story. Do not just focus on the list of questions on an assessment form. The questions might not be relevant to the patient's problems. Recognize normal changes associated with aging. Older-adult symptoms are often muted or less obvious, vague, or nonspecific compared with younger adults. Maintain a patient-directed gaze. Eye contact shows interest in what the patient is saying.

10. Which of the following are measures to reduce tissue damage from shear? (Select all that apply.) 1. Use a transfer device (e.g., transfer board) 2. Have head of bed elevated when transferring patient 3. Have head of bed flat when repositioning patient 4. Raise head of bed 60 degrees when patient positioned supine 5. Raise head of bed 30 degrees when patient positioned supine

10. Answer: 1, 3, 5. A transfer device can pick up a patient and prevent his or her skin from sticking to the bedsheet as he or she is repositioned. Positioning the patient flat when repositioning reduces shear. Positioning the patient with the head of the bed elevated at 30 degrees prevents him or her from sliding. The head of bed in higher position causes patient to slide down, causing shear.

10. The nurse is caring for a 50-year-old woman visiting the outpatient medicine clinic. The patient has had type 1 diabetes since age 13. She has numerous complications from her disease, including reduced vision, heart disease, and severe numbness and tingling of the extremities. Knowing that spirituality helps patients cope with chronic illness, which of the following principles should the nurse apply in practice? (Select all that apply.) 1. Pay attention to the patient's spiritual identity throughout the course of her illness. 2. Select interventions that you know scientifically support spiritual well-being. 3. Listen to the patient's story each visit to the clinic, and offer a compassionate presence. 4. When the patient questions the reason for her long-time suffering, try to provide answers. 5. Consult with a spiritual care adviser, and have the adviser recommend useful interventions.

10. Answer: 1, 3. A person's spiritual well-being can change over time; therefore, it is important to pay attention to it over the course of his or her illness. Listening is a powerful way to support a patient's spirituality. Evidence-based interventions are preferred, but they must be agreed on by the patient and tailored to his or her perspectives and not just those of the nurse. Patients are not looking for answers. What is spoken as a spiritual question is most often an expression of spiritual pain. Using spiritual care advisers is a valuable resource but should be selected by the patient, not independently by the nurse, and any interventions should be mutually agreed on among nurse, patient, and adviser

10. A patient is receiving total parenteral nutrition (TPN). What are the primary interventions the nurse should follow to prevent a central line infection? (Select all that apply.) 1. Change the dressing using sterile technique. 2. Change TPN containers every 48 hours. 3. Change the TPN tubing every 24 hours. 4. Monitor glucose levels to watch and assess for glucose intolerance. 5. Elevate head of the bed 45 degrees to prevent aspiration

10. Answer: 1, 3. The central line is inserted into a large vein that leads to the superior vena cava. This increases risk for infection. Therefore to prevent infection, change the TPN infusion tubing every 24 hours. Do not hang a single container of PN for more than 24 hours or lipids more than 12 hours. Use sterile technique during central line dressing changes (see Chapter 42). Monitoring glucose levels and elevating the head of bed are not interventions that will prevent central line infections

10. When assessing a patient's adjustment to the role changes brought about by a medical condition such as a stroke, the nurse asks about which of the following? (Select all that apply.) 1. What are your thoughts about returning to work? 2. What questions do you have about your medications? 3. How has your health affected your relationship with your partner? 4. What level of physical activity are you able to perform? 5. What concerns do you have about another stroke?

10. Answer: 1, 3. The nurse must assess role performance as related to professional identity (work) and personal relationships (partner). The other questions are important, but not related to self-concept.

10. There is no urine when a catheter is inserted 3 inches into a female's urethra. What should the nurse do next? 1. Remove the catheter and start all over with a new kit and catheter. 2. Leave the catheter there and start over with a new catheter. 3. Pull the catheter back and reinsert at a different angle. 4. Ask the patient to bear down and insert the catheter farther

10. Answer: 2. The catheter may be in the vagina; leave the catheter in the vagina as a landmark indicating where not to insert, and insert another sterile catheter. Pulling the catheter back and reinserting is poor technique, increasing the risk for catheter-associated urinary tract infection (CAUTI).

10. A 71-year-old patient enters the emergency department after falling down stairs at church. The nurse is conducting a fall history with the patient and his wife. They live in a one-level ranch home. He has had diabetes for over 15 years and experiences some numbness in his feet. He wears bifocal glasses. His blood pressure is stable at 130/70. The patient does not exercise regularly and states that he experiences weakness in his legs when climbing stairs. He is alert, oriented, and able to answer questions clearly. What are the fall risk factors for this patient? (Select all that apply.) 1. Impaired vision 2. Residence design 3. Blood pressure 4. Leg weakness 5. Exercise history

10. Answer: 1, 4, 5. Risk factors for falling include sensory changes such as visual loss, musculoskeletal conditions affecting mobility (in this case weakness), and deconditioning (from lack of exercise). The mere presence of a chronic disease is not a risk factor unless it is a condition such as a neurological disorder that alters mobility or cognitive function. The patient's blood pressure is stable, and there is no report of orthostatic hypotension. A one-floor residence should not pose risks.

10. You are floated to work on a nursing unit where you are given an assignment that is beyond your capability. Which is the best nursing action to take first? 1. Call the nursing supervisor to discuss the situation. 2. Discuss the problem with a colleague. 3. Leave the nursing unit and go home. 4. Say nothing and begin your work.

10. Answer: 1. Alerting the nursing supervisor as a representative of the hospital administration is the first step in providing notice that a problem may exist related to insufficient staffing. This notice serves to share the burden of knowledge of the staffing inequity issues that may create an unsafe patient situation for the hospital and nursing staff.

10. During an EBP committee meeting, a nurse discussed two systematic integrative reviews related to the use of prepackaged bath kits versus the standard use of bath basins. What level of evidence is the nurse presenting? 1. Level I 2. Level II 3. Level IV 4. Level VI

10. Answer: 1. Level 1 evidence is the strongest level of evidence. It is comprised of systematic integrative reviews or metaanalyses of randomized and unrandomized clinical trials

10. A nurse is taking care of a patient who has decided to stop smoking cigarettes. Which online resource would provide evidence-based information about smoking cessation? 1. The American Lung Association online toolkit for smoking cessation 2. An online blog led by a nurse for people discussing smoking cessation 3. A self-help website maintained by a hospital focusing on general wellness behaviors 4. A CDC website that discusses addictive behavior and risk factors

10. Answer: 1. The American Lung Association has an online toolkit that provides many resources for smoking cessation and would be the best online resource for patient referral. Online blogs may not be evidence-based. The resources in Options 3 and 4 do not provide specific information on how to stop smoking

10. Which number corresponds to the spot where you would assess for an air leak in the patient with a chest tube?

10. Answer: 1. This component is the air leak monitor. Option 2 is the dry suction regulator, where a nurse can regulate or titrate the amount of suction applied to the chest tube. Option 3 is the suction monitor bellows that the nurse assesses to ensure that suction is working and being applied. If the bellows is all the way over to the right, then the suction is working. Option 4 is the collection chamber, where the nurse will monitor the amount or volume of drainage coming from the patient's pleural space

10. Match the fall prevention intervention on the left with the scientific rationale on the right. 1. Prioritize nurse call system responses to patients at high risk. 2. Place patient in a wheelchair with wedge cushion. 3. Establish elimination schedule with bedside commode. 4. Use a low bed for patient. 5. Provide a hip protector. 6. Place nonskid floor mat on floor next to bed. A. Maintains comfort and makes exit difficult B. Makes it difficult for patients with lower extremity weakness to stand C. Reduces slipping when walking D. Reduces fall impact E. Ensures rapid response for help F. Reduces chance of patient trying to get out of bed on own

10. Answer: 1E, 2A, 3F, 4B, 5D, 6C.

10. Match the following caring behaviors with their definitions. 1. Knowing 2. Being with 3. Doing for 4. Maintaining belief a. Sustaining faith in the other's capacity to get through an event or transition and face a future with meaning b. Striving to understand an event as it has meaning in the life of the other c. Being emotionally present to the other d. Doing for the other as he or she would do for self if it were at all possible

10. Answer: 1b, 2c, 3d, 4a. These are from Swanson's theory (see Table 7.2). The theory describes caring as consisting of five categories or processes. Swanson defines caring as a nurturing way of relating to a valued other toward whom one feels a personal sense of commitment and responsibility. This theory supports the claim that caring is a central nursing phenomenon but not necessarily unique to nursing practice

10. An 82-year-old patient who resides in a nursing home has the following three nursing diagnoses: Risk for Fall, Impaired Physical Mobility related to pain, and Imbalanced Nutrition: Less Than Body Requirements related to reduced ability to feed self. The nursing staff identified several goals of care. Match the goals on the left with the appropriate outcome statements on the right. Goals Outcomes 1. _____ Patient will ambulate independently in 3 days. 2. _____ Patient will be injury free for 1 month. 3. _____ Patient will achieve 5-pound weight gain in 1 month. 4. _____ Patient will achieve pain relief by discharge. a. Patient expresses fewer nonverbal signs of discomfort within 24 hours. b. Patient increases caloric intake to 2500 calories daily. c. Patient walks 20 feet using a walker in 24 hours. d. Patient identifies barriers to remove in the home within 1 week.

10. Answer: 1c, 2d, 3b, 4a.

10. The faith community nurse is teaching the community center women's group about breast cancer risk factors. Which factors does the nurse include? (Select all that apply.) 1. First child at the age of 26 years 2. Menopause onset at the age of 49 years 3. Family history with BRCA1 inherited gene mutation 4. Age over 40 years 5. Onset of menses before the age of 12 6. Recent use of oral contraceptives

10. Answer: 3, 4, 5, 6. These are all risk factors for development of breast cancer. Onset of menopause after the age of 55, not at the age of 49, is a risk factor. First child after the age of 30, not birth of a child at 26, is a risk factor.

10. Match the following actions (1 through 4) with the terms (a through d) listed below: a. Advocacy b. Responsibility c. Accountability d. Confidentiality 1. You see an open medical record on the computer and close it so that no one else can read the record without proper access. 2. You administer a once-a-day cardiac medication at the wrong time, but nobody sees it. However, you contact the provider and your head nurse and follow agency procedure. 3. A patient at the end of life wants to go home to die, but the family wants every care possible. The nurse contacts the primary care provider about the patient's request. 4. You tell your patient that you will return in 30 minutes to give him his next pain medication.

10. Answer: 1d, 2c, 3a, 4b. Action 1 corresponds with option d. Preventing unnecessary access to a patient's health care information protects the patient's right to confidentiality. Action 2 corresponds to option c, accountability. Accountability refers to taking ownership of one's actions, which includes acknowledging errors. Action 3 corresponds to option a, advocacy. Sharing the patient's stated wish with other members of the health care team is an example of using your voice to benefit another person, in this case the patient. Action 4 corresponds with option b, responsibility. By following through on an established plan in caring for the patient, the nurse demonstrates responsibility.

10. Patient-to-patient transmission of infection cannot occur if gloves are routinely used. 1. True 2. False

10. Answer: 2 Although gloves are an additional tool to decrease the spread of infection from patient to patient, touching gloves with unclean hands as you put them on contaminates the gloves so that they are no longer clean.

10. A nurse is calling a patient's health care provider about a problem the patient is having following surgery. The health care organization uses the SBAR system in reporting patient problems. Put the statements in order according to the SBAR system. 1. Would it be possible to give the patient an antiemetic to help with the patient's nausea and comfort? 2. The patient is experiencing nausea right now. The nausea has worsened over the past hour. He states he feels as though he is going to get sick. 3. The patient had surgery earlier today to remove a tumor in the colon. He was admitted to the surgical unit 4 hours ago. He has a nasogastric (NG) tube in place. There is no postoperative order for an antiemetic

10. Answer: 2, 3, 4, 1. SBAR provides a consistent way to communicate patient problems. In this example, the Situation (S) is that the patient is experiencing nausea. Next the nurse provides the Background (B) about the patient's surgery and current orders. Then the nurse provides Assessment (A) data about the patient's current status. Finally, the nurse provides a Recommendation (R) to administer an antiemetic medication.

10. The nurse assesses the risk factors for coronary artery disease (CAD) in a female patient. Which of these factors are classified as genetic and physiological? (Select all that apply.) 1. Sedentary lifestyle 2. Mother died from CAD at age 48 3. History of hypertension 4. Eats diet high in sodium 5. Elevated cholesterol level

10. Answer: 2, 3, 5. Genetic and physiological risk factors include those related to heredity, genetic predisposition to an illness, or those that involve the physical functioning of the body. This patient's genetic and physiological risk factors include her family history of CAD as well as her history of hypertension and elevated serum cholesterol level. A sedentary lifestyle and dietary choices are behavioral factors that increase the risk for developing CAD.

10. A nurse newly hired at a community hospital learns about intentional hourly rounding during orientation. Which of the following are known evidence-based outcomes from intentional rounding? (Select all that apply.) 1. Reduction in nurse staffing requirements 2. Improved patient satisfaction 3. Reduction in patient falls 4. Increased costs 5. Reduction in patient call light use

10. Answer: 2, 3, 5. Intentional rounding is an evidence-based practice used in an increasing number of hospitals today. Studies have shown that intentional rounding can reduce patient falls and call light use and improve patient satisfaction scores. Proactive problem solving can occur when using intentional rounding. Education for patients helps them understand the importance of this practice.

10. The nurse uses silence as a therapeutic communication technique. What are the purposes of the nurse's silence? (Select all that apply.) 1. Allows the nurse time to focus and avoid saying the wrong thing 2. Prompts the patient to talk when he or she is ready 3. Allows the patient time to think and gain insight 4. Allows time for the patient to drift off to sleep 5. Determines whether the patient would prefer to talk with another staff member

10. Answer: 2, 3. Silence can provide that patient an opportunity to think and gain insight. Often the patient feels compelled to break the silence and is prompted to talk.

10. A patient is admitted for dehydration caused by pneumonia and shortness of breath. He has a history of heart disease and cardiac dysrhythmias. The assistive personnel reports his admitting vital signs to the nurse. Which measurements should the nurse reassess? (Select all that apply.) 1. Right arm BP: 118/72 2. Radial pulse rate: 72 and irregular 3. Temporal temperature: 37.4°C (99.3°F) 4. Respiratory rate: 28 5. Oxygen saturation: 99%

10. Answer: 2, 4, 5. Irregular pulse and elevated respiratory rate are outside expected values and require further assessment by the nurse. Pneumonia and shortness of breath can cause low oxygen saturation; an assessment of 99% may be a false-high value. Blood pressure and temperature are within expected values for the patient history

10. A 53-year-old female being treated for breast cancer tells the nurse that she has no interest in sex since her surgery 2 months ago. The nurse is aware that: (Select all that apply.) 1. Sexual issues are expected in a woman this age 2. Women experience sexual dysfunction more frequently than men. 3. Hypoactive sexual desire disorder (HSDD) occurs in women over 65 years of age. 4. Medical conditions such as cancer often contribute to HSDD. 5. Disturbances in self-concept affect sexual functioning

10. Answer: 2, 4, 5. Women of all ages (not just older women) can experience reduced sexual desire or libido. Biological, organic, or psychosocial factors; pain; depression; and body image concerns can result in sexual problems in men and women. Sexual dysfunction is common in men and women, but it occurs more frequently in women. Self-concept issues, including changes in body image, identity, and role performance, can impact self-esteem and sexual functioning.

10. A nurse is completing an assessment on a male patient, age 24. Following the assessment, the nurse notes that his family history is not significant for chronic illnesses, and his physical and laboratory findings are within normal limits. Because of these findings, nursing interventions are directed toward activities related to: (Select all that apply.) 1. Instructing him to return in 2 years. 2. Instructing him in secondary prevention. 3. Instructing him in health promotion activities. 4. Instructing him about routine screenings. 5. Instructing him about proper vaccinations

10. Answer: 3, 4, 5. While young adults generally have a minimum of major health problems, lifestyles such as tobacco or alcohol abuse, risky sexual activity, obesity, and lack of physical activity put them at risk for health problems. Instructing young adults in health promotion activities can decrease the risk for lifestyle-related health issues in the young adult. Proper vaccinations (e.g., flu vaccines and boosters to routine childhood vaccinations, such as tetanus) and regular health screenings are also important to maintain health.

10. The nurse observes an adult Middle Eastern patient attempting to bathe himself with only his left hand. The nurse recognizes that this behavior likely relates to: 1. Obsessive-compulsive behavior. 2. Personal preferences. 3. The patient's cultural norm. 4. Controlling behaviors.

10. Answer: 3. Cultural beliefs often influence patients' hygiene practices. Middle Eastern practices encourage one hand to be kept clean at all times

10. Which assessment does the nurse use as a clinical marker of vascular volume in a patient at high risk of extracellular fluid volume (ECV) deficit? 1. Dryness of mucous membranes 2. Skin turgor 3. Fullness of neck veins when supine 4. Fullness of neck veins when upright

10. Answer: 3. ECV deficit involves decreased vascular and interstitial volume. One way to assess vascular volume is to examine the fullness of neck veins when an individual is supine. With normal ECV, neck veins are full when the individual is supine. With ECV deficit, they are flat

10. Which of the following categories of shared theories would be most appropriate for a patient who is grieving the loss of a spouse? 1. Biomedical 2. Leadership 3. Psychosocial 4. Developmental

10. Answer: 3. Rationale: You can use various psychosocial theories to help patients with loss, death, and grief.

10. A 4-month-old infant has not been feeling well for 2 days. Which number on the image identifies the area of the infant's head where the nurse can assess for dehydration? 1. 1 2. 2 3. 3 4. 4 5. 5

10. Answer: 3. The anterior fontanel closes at 12 to 18 months, whereas the posterior fontanel closes by the end of the second or third month.

10. Dave reports being happy and satisfied with his life. What do we know about him? 1. He is in one of the later developmental periods, concerned with reviewing his life. 2. He is atypical, since most people in any of the developmental stages report significant dissatisfaction with their lives. 3. He is in one of the earlier developmental periods, concerned with establishing a career and satisfying long-term relationships. 4. It is difficult to determine Dave's developmental stage since most people report overall satisfaction with their lives in all stages

10. Answer: 4. Each of the life stages can be achieved successfully and result in satisfaction, including old age.

10. A nurse is taking a health history of a newly admitted patient with a diagnosis of possible fecal impaction. Which question is the priority to ask the patient or caregiver? 1. Have you eaten more high-fiber foods lately? 2. Have you taken antibiotics recently? 3. Do you have gluten intolerance? 4. Have you experienced frequent, small liquid stools recently?

10. Answer: 4. Frequent or continuous oozing of liquid stools occurs when liquid fecal matter above the impacted stool seeps around the fecal impaction.

10. The nurse is changing the dressing over the midline incision of a patient who had surgery. Assessment of the incision reveals changes from what was documented by the previous nurse. After documenting the current wound assessment, the nurse contacts the surgeon (Dr. Oakman) by telephone to discuss changes in the incision that are of concern. Which of the following illustrates the most appropriate way for the nurse to document this conversation? 1. Health care provider notified about change in assessment of abdominal incision. T. Wright, RN 2. 09-3-18: Notified Dr. Oakman by phone that there is a new area of redness around the patient's incision. T. Wright, RN 3. 1015: Contacted Dr. Oakman and notified about changes in abdominal incision. T. Wright, RN 4. 09-3-18 (1015): Dr. Oakman contacted by phone. Notified about new area of bright red erythema extending approximately 1 inch around circumference of midline abdominal incision and oral temperature of 101.5 F. No orders received. T. Wright, RN

10. Answer: 4. This statement includes the date and time the health care providerwas contacted, the specific name of the health care provider, descriptive details of the changes of concern noted in the patient assessment,whether any orderswere received, and the name and credentials of the nurse who contacted the health care provider

10. Match the characteristics on the left with the appropriate pain category on the right. Acute Pain Chronic Pain A. Has a protective effect B. Lasts more than 3 to 6 months C. Usually has identifiable cause D. Dramatically affects quality of life E. Viewed as a disease F. Eventually resolves with or without treatment

10. Answer: Acute pain: A, C, F; Chronic pain: B, D, E. Acute pain is protective, usually has an identifiable cause, and is of short duration. It eventually resolves, with or without treatment, after an injured area heals. Acute pain seriously threatens a patient's recovery by hampering his or her ability to become active and involved in self-care, and motivating a patient toward self-care can be hindered until the pain is managed successfully. Chronic pain has a dramatic effect on a person's quality of life. It is not protective. It typically is an ongoing or recurrent pain that lasts more than 3 to 6 months. Chronic pain does not always have an identifiable cause. It is viewed as a disease.

10. Fill in the Blank: A(n) __________________________ diagnosis is one that applies when there is an increased potential or vulnerability for a patient to develop a problem.

10. Answer: Risk

10. Identify the measures to ensure safety for a patient who has no sensation on one side of the body

10. Instruct the patient to obtain proper foot sizing and not to wear tight shoes; inspect feet to determine whether there are any pressure areas from shoes on the foot with limited sensation; test the temperature of bath water with the hand or foot on the side of the body with sensation; do not use heating pads or hot-water bottles on the affected side; lie on the back or unaffected side; when sitting in a chair readjust weight off affected side or stand up and move around frequently (e.g., every hour or every 90 minutes). In addition, teach the patient and family how to assess skin on the affected side for breakdown or tissue injury.

2. Theory is essential to nursing practice because it: (Select all that apply.) 1. Contributes to nursing knowledge. 2. Predicts patient behaviors in situations. 3. Provides a means of assessing patient vital signs. 4. Guides nursing practice. 5. Formulates health care legislation. 6. Explains relationships between concepts

2. Answer: 1, 2, 4, 6. A theory contains a set of concepts, definitions, and assumptions that explain a phenomenon. The theory explains how these elements are uniquely related in the phenomenon. These components provide a foundation of knowledge from which nurses can direct and deliver caring nursing practices. A theory helps explain an event by defining ideas or concepts, explaining relationships between the concepts, and predicting outcomes of nursing care. A nursing theory conceptualizes an aspect of nursing to describe, explain, predict, or prescribe nursing care.

2. A community health nurse is working in a clinic with a focus on asthma and allergies. What is the primary focus of the community health nurse in this clinic setting? (Select all that apply.) 1. Decrease the incidence of asthma attacks in the community 2. Increase patients' ability to self-manage their asthma 3. Treat acute asthma in the hospital 4. Provide asthma education programs for the teachers in the local schools 5. Provide scheduled immunizations to people who come to the clinic

2. Answer: 1, 2, 4. All of these activities improve the level of health and quality of life for patients in this community. Asthma self-management controls symptoms and improves a patient's quality of life. Assessing for and preventing risks, as well as educational programs, improve the level of health within a community. Managing chronic diseases in the community improves the overall level of health of that community

2. A nurse working the evening shift has five patients and is teamed up with an assistive personnel. One of the assigned patients has just returned from surgery, three others are stable and resting, and one has requested a pain medication. The patient in pain has two analgesics ordered prn for pain and has been using cold applications on his surgical site for pain relief. The last time an analgesic was given was 4 hours ago. The patient is scheduled for a physical therapy visit in 2 hours. Which of the following demonstrate good clinical decision making during intervention? (Select all that apply.) 1. The nurse reviews the options for pain relief for the patient. 2. The nurse assesses whether the prn medication, ordered every 4 to 6 hours and last given 4 hours ago, is effective and whether a new type of medication is needed. 3. The nurse reviews the policy and procedure for the cold application. 4. The nurse considers how the patient might react if the pain medication is held until an hour before physical therapy. 5. The nurse delegates vital sign assessment of the patient returning from surgery to the assistive personnel.

2. Answer: 1, 2, 4. The nurse exercises critical judgment in decision making by reviewing the set of all possible nursing interventions for a patient's problem, reviewing possible consequences associated with each possible nursing action, determining the probability of all possible consequences, and judging the value of the consequence to the patient. Clinical decision making is not in play simply by accessing a resource. Decision making would be applied if the nurse revises or adapts how to perform the cold application. Delegation of vital signs in a potentially unstable patient is not good clinical decision making

2. Which statements from a patient indicate an understanding of behaviors that will promote sleep? (Select all that apply.) 1. "I will not watch television in bed." 2. "I will not drink caffeine later in the day." 3. "A short nap late in the evening will lead to a more restful night of sleep." 4. "I am going to start eating dinner closer to my bedtime" 5. "I will start to exercise regularly during the day.

2. Answer: 1, 2, 5. To promote sleep, you should not watch television in bed. The noise of television can be disruptive and adds stimulation that is disruptive to sleep. Caffeine should not be consumed late in the day because it can cause wakefulness at bedtime. A regular exercise program completed in the morning is part of sleep hygiene practices and can help promote sleep. Exercise or eating a meal should not be done right before bed because sleep can be disrupted

2. The nurse is working in an agency that has recently implemented an electronic health record. Which of the following are acceptable practices for maintaining the security and confidentiality of electronic health record information? (Select all that apply.) 1. Using a strong password and changing your password frequently according to agency policy 2. Allowing a temporary staff member to use your computer user name and password to access the electronic record 3. Ensuring that work lists (and any other data that must be printed from the electronic health record) are protected throughout the shift and disposed of in a locked receptacle designated for documents that are to be shredded when no longer needed 4. Ensuring that the patient information that is displayed on the computer monitor that you are using is not visible to visitors and other health care providers who are not involved in that patient's care 5. Remaining logged in to a computer to save time if you only need to step away to administer a medication

2. Answer: 1, 3, 4. Mechanisms to protect the privacy and confidentiality of protected health information in the electronic health record include: not sharing passwords, not leaving computers with open electronic health records unattended, and preventing those not involved with a patient's care from seeing information displayed on a monitor.

2. An adolescent who is pregnant for the first time is at her initial prenatal visit. The women's health nurse practitioner (WHNP) informs the patient that she will be screening her for sexually transmitted infections (STIs). The patient replies, "I know I don't have an STI because I don't have any symptoms." Which responses by the WHNP would be appropriate? (Select all that apply.) 1. "Untreated STIs can cause serious complications in pregnancy, so we routinely screen pregnant women." 2. "Bacterial STIs don't usually cause symptoms, or you could have an asymptomatic viral STI." 3. "Chlamydia screening is recommended for all sexually active women up to age 25 even if asymptomatic." 4. "People between the ages of 15 and 24 are often asymptomatic and have the highest incidence of STIs." 5. "There is no need to screen for infection since you aren't having any problems or symptoms."

2. Answer: 1, 3, 4. Serious complications can result from untreated STIs in pregnancy, complications such as preterm labor, rupture of membranes, and premature delivery of the newborn. The risk of untreated STIs in any female is pelvic inflammatory disease, which, if untreated, can cause serious problems such as infertility. Routine screening for chlamydia is recommended for all sexually active women up to age 25. Many people do not know they are infected because they do not experience symptoms. Bacterial STIs are more likely to cause symptoms, whereas viral STIs are often asymptomatic

2. Which complementary therapies are most easily learned and applied by a nurse? (Select all that apply.) 1. Therapeutic massage therapy 2. Traditional Chinese medicine 3. Progressive relaxation 4. Breathwork and guided imagery 5. Therapeutic touch

2. Answer: 1, 3, 4. These are nurse-accessible complementary therapies. A simple back rub can be administered by nurses. Therapeutic massage therapists are licensed by local governmental agencies, and additional educational preparation is required to practice. Traditional Chinese medicine practitioners also attend training/educational programs, typically accredited by the Accreditation Commission for Acupuncture and Oriental Medicine

2. Before implementing touch, what does a nurse need to know about touch? (Select all that apply.) 1. Some cultures may have specific restrictions about non-skillbased touch. 2. Touch is a type of verbal communication. 3. Touch can successfully influence a patient's level of comfort. 4. There is never a problem with using touch at any time. 5. Touch only reduces physical pain

2. Answer: 1, 3. Some cultures have restrictions as to who can touch and when to touch. Assess your patient to determine whether there are any restrictions. Touch is a method of nonverbal communication. Touch can help improve a patient's sense of comfort and leads to a connection between nurse and patient. Not all patients perceive touch to be therapeutic. Know and understand whether your patients accept touch and how they interpret your intentions. Touching a patient can reduce physical pain and discomfort and decrease anxiety and fear

2. Which assessment questions should the nurse ask a preoperative patient preparing for surgery? (Select all that apply.) 1. "Are you experiencing any pain?" 2. "Do you exercise on a daily basis?" 3. "When do you regularly take your medications?" 4. "Do you have any medication allergies?" 5. "Do you use drugs and/or tobacco products

2. Answer: 1, 4, 5. Although regular exercise and adherence to the medication regimen are important, for the preoperative patient, the nurse needs to focus on factors that impact the surgical experience.

2. A 9-year-old child has a difficult time making friends at school and being chosen to play on the team. He also has trouble completing his homework and, as a result, receives little positive feedback from his parents or teacher. According to Erikson's theory, failure at this stage of development results in: (Select all that apply.) 1. Feelings of inadequacy. 2. A sense of guilt. 3. A poor sense of self. 4. Feelings of inferiority. 5. Mistrust.

2. Answer: 1, 4. School-age children need to feel real accomplishment and be accepted by peers to develop a sense of industry. Without proper support for learning new skills, or if skills are too difficult, they develop a sense of inadequacy and inferiority.

2. The nurse is evaluating the recent lab results for a patient. Which labs are the best indicators for malnutrition? (Select all that apply.) 1. Serum total protein 2. Potassium 3. Lipids 4. Albumin 5 Serum BUN

2. Answer: 1, 5. When a client is malnourished, he or she is in a state of negative nitrogen balance—meaning, the body is experiencing protein loss and requires more protein to maintain healing. Therefore, total protein will indicate the amount of muscle breakdown and protein loss. Albumin is a serum binding protein, and lower levels can be an indicator of malnutrition, but it is really more indicative of inflammation or kidney and liver disease. As a result, this is not the gold standard for diagnosing malnutrition. BUN is also an indicator because urea is the end product of protein metabolism, and when a patient is not getting enough protein, you will see a decreased BUN.

2. Match the assessment activity on the left with the type of assessment on the right. 1. Assessment conducted at beginning of a nurse's shift 2. Review of a patient's chief complaint 3. Completion of admitting history at time of patient admission to a hospital 4. Completion of the Long Term Care Minimum Data Set during an elderly patient admission to a nursing home A. Problem focused B. Comprehensive

2. Answer: 1. A, 2. A, 3. B, 4. B. Assessments can be either comprehensive or problem focused. A periodic assessment is a problem-focused approach that begins with a patient's presenting situation and specific problematic areas. A comprehensive assessment about a patient includes a detailed assessment of a patient's physical, psychosocial, cultural, spiritual, and lifestyle needs.

2. A patient who is having difficulty managing his diabetes mellitus responds to the news that his hemoglobin A1c, a measure of blood sugar control over the past 90 days, has increased by saying, "The hemoglobin A1c is wrong. My blood sugar levels have been excellent for the last 6 months." Which defense mechanism is the patient using? 1. Denial 2. Conversion 3. Dissociation 4. Displacement

2. Answer: 1. Denial is avoiding emotional stress by refusing to consciously acknowledge anything that causes intolerable anxiety. This patient's statements reflect denial about poorly controlled blood sugars

2. The licensed practical nurse (LPN) provides you with the changeof-shift vital signs on four of your patients. Which patient does the nurse need to assess first? 1. 84-year-old man recently admitted with pneumonia, RR 28, SpO2 89% 2. 54-year-old woman admitted after surgery for repair of a fractured arm, BP 160/86 mm Hg, HR 72 3. 63-year-old man with venous ulcers from diabetes, temperature 37.3° C (99.1° F), HR 84 4. 77-year-old woman with left mastectomy 2 days ago, RR 22, BP 148/62

2. Answer: 1. SpO2 89% is a critical value and requires immediate attention. Other values require attention but are not life threatening.

2. A health care provider writes the following order for a patient who is opioid-naïve who returned from the operating room following a total hip replacement: "Fentanyl patch 100 mcg, change every 3 days." On the basis of this order, the nurse takes the following action: 1. Calls the health care provider and questions the order 2. Applies the patch the third postoperative day 3. Applies the patch as soon as the patient reports pain 4. Places the patch as close to the hip dressing as possible

2. Answer: 1. The nurse needs to call the health care provider about the order because fentanyl patches are not indicated for acute pain. They are indicated for patients with chronic pain who are opioid tolerant.

2. During the administration of a warm tap-water enema, a patient complains of cramping abdominal pain that he rates 6 out of 10. What nursing intervention should the nurse do first? 1. Stop the instillation. 2. Ask the patient to take deep breaths to decrease the pain. 3. Tell the patient to bear down as he would when having a bowel movement. 4. Continue the instillation; then administer a pain medication

2. Answer: 1. When a patient complains of pain during an enema, you need to stop the instillation and conduct an assessment before discontinuing or resuming the procedure

2. The nurse is caring for a patient who is very depressed and decides to complete a spiritual assessment using the FICA tool. Using the FICA assessment tool, match the criteria on the left with the appropriate assessment question on the right. 1. F—Faith ___ 2. I—Importance of spirituality ___ 3. C—Community ___ 4. A—Interventions to address spiritual needs ___ a. Tell me if you have a higher power or authority that helps you act on your beliefs b. Describe which activities give you comfort spiritually c. To whom do you go for support in times of difficulty? d. Your illness has kept you from attending church. Is that a problem for you?

2. Answer: 1a, 2d, 3c, 4b.

2. Match the pressure injury stages with the correct definition. 1. Stage 1 2. Stage 2 3. Stage 3 4. Stage 4 a. Partial-thickness loss of skin with exposed dermis. The wound bed is viable, pink or red, moist, and may also present as an intact or ruptured serumfilled blister. Adipose (fat) is not visible, and deeper tissues are not visible. Granulation tissue, slough, and eschar are not present. These injuries commonly result from adverse microclimate and shear in the skin over the pelvis and shear in the heel. This stage should not be used to describe moisture-associated skin damage (MASD), including incontinence-associated dermatitis (IAD), intertriginous dermatitis (ITD), medical adhesive- related skin injury (MARSI), or traumatic wounds (skin tears, burns, abrasions). b. Intact skin with a localized area of nonblanchable erythema, which may appear differently in darkly pigmented skin. Presence of blanchable erythema or changes in sensation, temperature, or firmness may precede visual changes. Color changes do not include purple or maroon discoloration; these may indicate deep tissue pressure injury. c. Full-thickness skin and tissue loss with exposed or directly palpable fascia, muscle, tendon, ligament, cartilage, or bone in the ulcer. Slough and/ or eschar may be visible. Epibole (rolled edges), undermining, and/or tunneling often occurs. Depth varies by anatomical location. If slough or eschar obscures the extent of tissue loss, this is an Unstageable Pressure Injury. d. Full-thickness loss of skin, in which adipose (fat) is visible in the ulcer and granulation tissue and epibole (rolled wound edges) are often present. Slough and/or eschar may be visible. The depth of tissue damage varies by anatomical location; areas of significant adiposity can develop deep wounds. Undermining and tunneling may occur. Fascia, muscle, tendon, ligament, cartilage, and/ or bone are not exposed. If slough or eschar obscures the extent of tissue loss, this is an Unstageable Pressure Injury

2. Answer: 1b, 2a, 3d, 4c.

2. Match the concepts for a critical thinker on the right with the application of the term on the left. Term Application Concepts for Critical Thinkers a. Anticipate how a patient might respond to a treatment. ___ 1. Truth seeking b. Organize assessment on the basis of patient priorities. ___ 2. Open-mindedness c. Be objective in asking questions of a patient. ___ 3. Analyticity d. Be tolerant of the patient's views and beliefs. ___ 4. Systematicity

2. Answer: 1c, 2d, 3a, 4b.

2. Match the elements for correct identification of outcome statements with the SMART acronym terms below. 1. Specific 2. Measurable 3. Attainable 4. Realistic 5. Timed a. Mutually set an outcome that a patient agrees to meet. b. Set an outcome that a patient can meet based upon his or her physiological, emotional, economic, and sociocultural resources. c. Be sure an outcome addresses only one patient behavior or response. d. Include when an outcome is to be met. e. Use a term in an outcome statement that allows for observation as to whether a change takes place in a patient's status.

2. Answer: 1c; 2e; 3a; 4b; 5d.

2. A patient is diagnosed with meningitis. Which type of isolation precaution is most appropriate for this patient? 1. Reverse isolation 2. Droplet Precautions 3. Standard Precautions 4. Contact Precautions

2. Answer: 2 Because the patient is diagnosed with meningitis, which can be spread when the patient coughs or sneezes, droplet precautions are most appropriate.

2. Nurses must communicate effectively with the health care team for which of the following reasons? (Select all that apply.) 1. To improve the nurse's status with the health team members 2. To reduce the risk of errors to the patient 3. To provide an optimum level of patient care 4. To improve patient outcomes 5. To prevent issues that need to be reported to outside agencies

2. Answer: 2, 3, 4. Effective communication in health care has been linked to a decrease in medical errors and an improvement in quality of care and patient outcomes. The status of the nurse or the prevention of reportable issues is not the focus of communication with patients

2. The home care nurse is instructing an assistive personnel about interventions to facilitate location of items for patients with vision impairment. Which are effective strategies for enhancing a patient's impaired vision? (Select all that apply.) 1. Use of fluorescent lighting 2. Use of warm incandescent lighting 3. Use of yellow or amber lenses to decrease glare 4. Use of adjustable blinds, sheer curtains, or draperies 5. Indirect lighting to reduce glare

2. Answer: 2, 3, 4. Interventions to enhance vision include the use of warm, incandescent lighting; yellow or amber lenses to decrease glare; and adjustable blinds, sheer curtains, or draperies to allow for the adjustment of natural light. Fluorescent lighting can contribute to indirect and direct glare.

2. Which interventions does a nurse implement to help a patient at the end of life maintain autonomy while in a hospital? (Select all that apply.) 1. Use therapeutic techniques when communicating with the patient. 2. Allow the patient to determine timing and scheduling of interventions. 3. Allow patients to have visitors at any time. 4. Provide the patient with a private room close to the nurses' station. 5. Encourage the patient to eat whenever he or she is hungry

2. Answer: 2, 3, 5. Allowing patients to make choices about their care and end-of-life experience provides opportunities for them to maintain their autonomy

2. The student nurse is teaching a family member the importance of foot care for his or her mother, who has diabetes. Which safety precautions are important for the family member to know to prevent infection? (Select all that apply.) 1. Cut nails frequently. 2. Assess skin for redness, abrasions, and open areas daily. 3. Soak feet in water at least 10 minutes before nail care. 4. Apply lotion to feet daily. 5. Clean between toes after bathing

2. Answer: 2, 4, 5. Because of a patient's risk for infection, it is important to assess skin for redness, abrasions, and open areas daily. Apply lotion to feet daily to keep the skin hydrated, but do not leave excess lotion on the skin. Clean between toes carefully after bathing to avoid maceration. Do not cut nails or soak the feet of a patient with diabetes because this may create skin breakdown and open sores, leading to skin breakdown or infection.

2. A mother and her two children are homeless and enter a free health care clinic. Which statements most likely describe the effects of homelessness on this family? (Select all that apply.) 1. The children have stability in their education. 2. The family members may have symptoms of malnutrition, such as anemia. 3. The family is at a low risk for experiencing violence. 4. The children are at higher risk for developing ear infections. 5. All family members may have mental health issues.

2. Answer: 2, 4, 5. Families who are homeless often experience difficulty in accessing food and have a poor nutritional status. Children are at a higher risk for developing ear infections. Homelessness puts all family members at risk for developing mental health issues. Children have difficulty proving residency for school enrollment, so they are more likely not to be in school or to drop out. Families that are homeless are also at a higher risk for experiencing physical and emotional violence.

2. A nurse conducts an assessment of a 42-year-old woman at a health clinic. The woman is married and lives in a condo with her husband. She reports having frequent voiding and pain when she passes urine. The nurse asks whether she has to go to the bathroom at night, and the patient responds, "Yes, usually twice or more." The patient had an episode of diarrhea 1 week ago. She weighs 300 lb and reports having difficulty cleansing herself after voiding or passing stool. Which of the following demonstrate assessment findings that cluster to indicate the nursing diagnosis Impaired Urination. (Select all that apply.) 1. Age 42 2. Dysuria 3. Difficulty performing perineal hygiene 4. Nocturia 5. Episode of diarrhea

2. Answer: 2, 4. The assessment findings are individual data elements that form a pattern revealing a nursing diagnosis. The assessment findings include dysuria and nocturia. The patient's age is a descriptive characteristic of the patient. The episode of diarrhea is a historical finding that predisposed the patient to having perineal hygiene needs, but is not an assessment finding for the diagnosis.

2. When designing a plan for pain management for a patient following surgery, the nurse assesses that the patient's priority is to be as free of pain as possible. The nurse and patient work together to identify a plan to manage the pain. The nurse continually reviews the plan with the patient to ensure that the patient's priority is met. If the nurse's actions are driven by respect for autonomy, what aspect of this scenario best demonstrates that? 1. Assessing the patient's pain on a numeric scale every 2 hours 2. Asking the patient to establish the goal for pain control 3. Using alternative measures such as distraction or repositioning to relieve the pain 4. Monitoring the patient for oversedation as a side effect of his pain medication

2. Answer: 2. Asking the patient to establish the goal for pain control is a demonstration of respect for autonomy. Assessing, monitoring, and using alternative measures are interventions that address pain but that are not necessarily grounded in the principle of autonomy

2. A nurse conducted an assessment of a new patient who came to the medical clinic. The patient is 82 years old and has had osteoarthritis for 10 years and diabetes mellitus for 20 years. He is alert but becomes easily distracted during the assessment. He recently moved to a new apartment, and his pet beagle died just 2 months ago. He is most likely experiencing: 1. Dementia. 2. Depression. 3. Delirium. 4. Anxiety.

2. Answer: 2. Factors that often lead to depression include presence of a chronic disease or a recent change or life event (such as loss). Patients are alert but easily distracted in conversation.

2. A patient has labored breathing, a respiratory rate of 28 per minute, and lung sounds that reveal wheezing bilaterally. The nurse starts an ordered intravenous infusion to administer medication that will relax the patient's airways. When the nurse asks how the patient feels, he responds by saying, "I feel as if I can breathe better." The nurse auscultates the patient's lungs and notes decreased wheezing with a respiratory rate of 22 per minute. Which of the following evaluative measures may not reflect change in a patient's condition? 1. Counting respirations per minute 2. Asking the patient to describe how his breathing feels 3. Observing breathing pattern 4. Auscultating lung sounds

2. Answer: 2. Self-report may not truly reflect whether a patient's behavior or perception has changed.

2. A patient suddenly experiences a severe headache with numbness and decreased movement in the left arm. The emergency room physician suspects a stroke and is going to have the patient undergo an emergent angiogram to remove the clot. Which teaching approach is most appropriate? 1. Selling approach 2. Telling approach 3. Entrusting approach 4. Participating approach

2. Answer: 2. Telling is the best approach when there is limited time for teaching information

2. A nurse received change-of-shift report on these four patients and starts rounding. Which patient does the nurse need to focus on as a priority? 1. The patient who had abdominal surgery 2 days ago who is requesting pain medication 2. A patient admitted yesterday with atrial fibrillation who now has a decreased level of consciousness 3. A patient with a wound drain who needs teaching before discharge in the early afternoon 4. A patient going to surgery for a mastectomy in 3 hours who has a question about the surgery

2. Answer: 2. This patient is of high priority. The patient is experiencing the physiological problem of decreased level of consciousness that is an immediate threat to the patient's survival and safety. The nurse must intervene promptly and notify the health care provider of the life-threatening problem.

2. A patient is experiencing some problems with joint stability in the right leg. The doctor has prescribed crutches for the patient to use while being allowed to bear weight only on the left leg. Which of the following gaits should the patient be taught to use? 1. Four-point 2. Three-point 3. Two-point 4. Swing-through

2. Answer: 2. Three-point gait requires a patient to bear all of the weight on one foot. In a three-point gait the patient bears weight on both crutches and then on the uninvolved leg, repeating the sequence. The affected leg does not touch the ground during the early phase of the three-point gait. Gradually the patient progresses to touchdown and full weight-bearing on the affected leg.

2. A nurse enters the hospital room of a patient who had a total knee replacement the day before. Which of the following pose potential safety risks? (Select all that apply.) 1. A current safety inspection sticker is on the IV fluids pump. 2. A walker is positioned near the patient's bedside. 3. The hospital bed is in the high position. 4. There is no gait belt at the bedside. 5. The overbed table with the patient's glasses is positioned against the wall opposite the end of the bed

2. Answer: 3, 4, 5. All electrical equipment should be inspected routinely and have current safety inspection stickers. The patient has had knee surgery, so the presence of a walker is needed for him to ambulate. Safety risks include the absence of a gait belt; one should always be available for a patient who will need assistance in ambulation. The bed position is incorrect; it should be in low position. The position of the bedside table does not allow the patient to reach personal or care items easily

3. An older adult states that she cannot see her medication bottles clearly to determine when to take her prescription. What should the nurse do? (Select all that apply.) 1. Provide a dispensing system for each day of the week. 2. Provide larger, easier-to-read labels. 3. Tell the patient what is in each container. 4. Have a family caregiver administer the medication. 5. Use teach-back to ensure that the patient knows what medication to take and when.

3. Answer: 1, 2, 5. Larger print and a dispensing system can ensure safe medication administration in older adults. Medication pamphlets in larger print are also available. The use of teach-back ensures that the patient understands his or her medications and increases safety.

2. A 30-year-old patient diagnosed with major depressive disorder has a nursing diagnosis of Situational Low Self-Esteem related to negative view of self. Which of the following are appropriate interventions by the nurse? (Select all that apply.) 1. Encourage reconnecting with high school friends. 2. Role-play to increase assertiveness skills. 3. Focus on identifying strengths and accomplishments. 4. Provide time for journaling to explore underlying thoughts and feelings. 5. Explore new job opportunities.

2. Answer: 3, 4. Focusing on strengths and accomplishments to minimize the emphasis on failures helps the patient alter distorted and negative thinking. Journaling can allow a patient to explore thoughts and feelings that can promote insight and eventual behavioral change. The other interventions represent the nurse imposing ideas on what needs to occur for the patient to be healthier; allowing the patient to direct the change process is important.

2. What is a critical step when inserting an indwelling catheter into a male patient? 1. Slowly inflate the catheter balloon with sterile saline. 2. Secure the catheter drainage tubing to the bedsheets. 3. Advance the catheter to the bifurcation of the drainage and balloon ports. 4. Advance the catheter until urine flows, then insert ¼ inch more.

2. Answer: 3. Advancing the catheter to the bifurcation avoids inflating the catheter balloon in the prostatic urethra, causing trauma and pain. Catheter balloons are never inflated with saline. Securing the catheter drainage tubing to the bedsheets increases the risk for accidental pulling or tension on the catheter. Advancing the catheter until urine flows and then inserting it ¼ inch more is not unique to the male patient

2. Which factor affects a middle-age adult's adherence to a treatment plan? 1. Gender 2. Lifestyle 3. Motivation 4. Family history

2. Answer: 3. Motivation plays a key role in adherence with a prescribed course of therapy. Motivation can be influenced by a variety of factors, including age, experience, family history, social support, and pressure by health care providers

2. Based on the Transtheoretical Model of Change, what is the most appropriate response to a patient who states: "Me, stop smoking? I've been smoking since I was 16!" 1. "That's fine. Some people who smoke live a long life." 2. "OK. I want you to decrease the number of cigarettes you smoke by one each day, and I'll see you in 1 month." 3. "What do you think is the greatest reason why stopping smoking would be challenging for you?" 4. "I'd like you to attend a smoking-cessation class this week and use nicotine replacement patches as directed."

2. Answer: 3. The patient's response indicates that he is in the precontemplation stage and not intending to make a change in behavior in the next 6 months. In this stage the patient is not interested in information about the behavior and may be defensive when confronted with the information. Asking an open-ended question about smoking may stimulate the patient to identify a reason to begin a behavioral change. Nurses are challenged to motivate and facilitate health behavior change in working with individuals

2. A parent has brought her 6-month-old infant in for a well-child check. Which of her statements indicates a need for further teaching? 1. "I can start giving her whole milk at about 12 months." 2. "I can continue to breastfeed for another 6 months." 3. "I can give her plenty of fruit juice to increase her vitamin intake." 4. "I can start giving her solid food now, introducing one food at a time."

2. Answer: 3. The use of fruit juices and nonnutritive drinks such as fruit-flavored drinks or soda should be avoided since these do not provide sufficient and appropriate calories during this period.

2. A woman has severe life-threatening injuries, is unresponsive, and is hemorrhaging following a car accident. The health care provider ordered two units of packed red blood cells to treat the woman's anemia. The woman's husband refuses to allow the nurse to give his wife the blood for religious reasons. What is the nurse's responsibility? 1. Obtain a court order to give the blood. 2. Convince the husband to allow the nurse to give the blood. 3. Call security and have the husband removed from the hospital. 4. Gather more information about the wife's preferences and determine whether the husband is her power of attorney for health care.

2. Answer: 4. Adult patients such as those with specific religious objections are able to refuse treatment for personal religious reasons. Because this patient is unresponsive, it is important for the nurse to better understand the patient's preferences and know if the woman has a power of attorney for health care before following the husband's wishes. However, there needs to be clear directions on who can make the decision.

2. The nurse assesses pain and redness at a vascular access device (VAD) site. Which action is taken first? 1. Apply a warm, moist compress. 2. Aspirate the infusing fluid from the VAD. 3. Report the situation to the health care provider. 4. Discontinue the intravenous infusion

2. Answer: 4. Pain and redness at a VAD site are indicators of phlebitis. When phlebitis occurs, the infusion must be stopped and the VAD removed as the highest priority

2. An older-adult patient has been bedridden for 2 weeks. Which of these complaints by the patient indicates to the nurse that he or she is developing a complication of immobility? 1. Increase of appetite 2. Gum soreness 3. Difficulty in swallowing 4. Left ankle joint stiffness

2. Answer: 4. Temporary immobilization results in some muscle atrophy, loss of muscle tone, and joint stiffness. Two weeks of joint immobilization without ROM can quickly result in contractures. Patient's left ankle stiffness can be indicative of an early contracture

2. Which number corresponds to the area of the chest where you would auscultate for the tricuspid valve?

2. Answer: 4. The tricuspid area is located at the fourth or fifth intercostal space along the sternum

2. Place the steps of the EBP process in the appropriate order. 1. Critically appraise the evidence you gather. 2. Ask the clinical question in PICOT format. 3. Evaluate the outcomes of the practice decision or change. 4. Search for the most relevant and best evidence. 5. Cultivate a spirit of inquiry. 6. Integrate the evidence. 7. Communicate the outcomes of the EBP change.

2. Answer: 5, 2, 4, 1, 6, 3, 7. The process starts with a spirit of inquiry and the final step is sustaining the EBP change.

2. The health care provider has written the following orders. Which orders does the nurse need to clarify before administering the medication? Provide rationale for your answers, and rewrite the order so that it follows the ISMP current medication order safety guidelines. Timoptic .25% solution 1 drop OD BID Metoprolol 12.50 mg QD Insulin Glargine 6 u SC twice a day Enalapril 2.5 mg. PO three times a day, hold for systolic blood pressure <100

2. Answer: The nurse needs to clarify all the orders. Timoptic .25% solution 1 drop OD BID has a "naked" decimal point, and OD (right eye) could be mistaken for AD (right ear). Metoprolol 12.50 mg QD has a trailing zero, and the dosage could be mistaken for 1250 mg if the decimal point is not seen; it also has no route identified. Insulin Glargine 6 u SC twice a day includes the letter u, which means units but could be mistaken as the number 0 or 4, and SC could be mistaken as SL. Enalapril 2.5 mg. PO three times a day, hold for systolic blood pressure <100 has a period after mg, which could be mistaken as the number 1, and the < sign could be mistaken as greater than. The correctly written orders are "Timoptic 0.25% solution 1 drop right eye BID." Metoprolol 12.5 mg QD PO Insulin glargine 6 units subcutaneous twice a day Enalapril 2.5 mg PO three times a day, hold for systolic blood pressure less than 100

2. A nurse is providing education to a patient with type 2 diabetes. Which characteristics does the nurse include in her teaching to explain why type 2 diabetes is considered a chronic disease? (Select all that apply.) 1. Type 2 diabetes lasts throughout a person's life. 2. Genetic mutations drive the treatment for type 2 diabetes. 3. People with type 2 diabetes have to modify some of their daily activities. 4. Type 2 diabetes occurs in elderly people. 5. People with type 2 diabetes require ongoing medical care

2. Answer:1, 3, 5. A chronic disease is a pathophysiologic condition that lasts more than 1 year, requires ongoing medical care, and often limits a person's usual activities of daily living due to symptoms of the disease or self-care activities required to manage the disease.

2. A 35-year-old woman has Medicaid coverage for herself and two young children. She missed an appointment at the local health clinic to get an annual mammogram because she has no transportation. She gets the annual screening because her mother had breast cancer. Which of the following are social determinants of this woman's health? (Select all that apply.) 1. Medicaid insurance 2. Annual screening 3. Mother's history of breast cancer 4. Lack of transportation 5. Woman's age

2. Answers: 1, 4, 5. Social determinants of health are the conditions under which persons are born, grow, live, work, and age. The social determinants of health are mostly responsible for health disparities. Examples include age, race and ethnicity, socioeconomic status (as reflected by the woman's insurance), access to nutritious food, transportation resources, religion, sexual orientation, level of education, literacy level, disability (physical and cognitive), and geographic location (e.g., access to health care).

2. Which skills can the nurse delegate to assistive personnel (AP)? (Select all that apply.) 1. Initiate oxygen therapy via nasal cannula. 2. Perform nasotracheal suctioning of a patient. 3. Educate the patient about the use of an incentive spirometer. 4. Assist with care of an established tracheostomy tube. 5. Reposition a patient with a chest tube.

2. Answers: 4 and 5. Assistive personnel (AP) are not allowed to initiate oxygen therapy, provide education, or perform NT suctioning on a patient. They are allowed to assist the nurse in performing tracheostomy tube care and with repositioning patients.

4. A nurse is caring for a patient who is Muslim and has diabetes. Which of the following items does the nurse need to remove from the meal tray when it is delivered to the patient? 1. Small container of vanilla ice cream 2. A dozen red grapes 3. Bacon and eggs 4. Garden salad with ranch dressing

4. Answer: 3. Islam prohibits the consumption of pork.

3. A nurse working the evening shift has five patients and is teamed up with an assistive personnel. One of the assigned patients has just returned from surgery, one is newly admitted, and one has requested a pain medication. The patient who has returned from surgery just minutes ago has a large abdominal dressing, is still on oxygen by nasal cannula, and has an intravenous line. One of the other patients has just called out for assistance in setting up a meal tray. Another patient is stable and resting comfortably. Which patient is the nurse's current greatest priority? 1. Patient in pain 2. Patient newly admitted 3. Patient who returned from surgery 4. Patient requesting assistance with meal tray

3. Answer 3. The patient returning from surgery is likely the most physiologically unstable, requiring the nurse to perform an assessment and ensure the patient is managed appropriately. The patient in pain is likely to be the next priority, depending on the severity of the patient's reported pain. The newly admitted patient will require a nursing history, which takes time. The nurse can have the assistive personnel assist with the meal tray

3. The nurse recognizes that which factors influence a person's approach to death? (Select all that apply.) 1. Culture 2. Spirituality 3. Personal beliefs 4. Previous experiences with death 5. Gender 6. Level of education

3. Answer: 1, 2, 3, 4. Culture, spirituality, personal beliefs and values, and previous experiences with death influence how a person approaches death

3. A nurse asks an AP to help the patient in Room 418 walk to the bathroom right now. The nurse tells the AP that the patient needs the assistance of one person and the use of a walker. The nurse also tells the AP that the patient's oxygen can be removed while he goes to the bathroom but to make sure that when it is put back on the flowmeter is still at 2 L. The nurse also instructs the AP to make sure the side rails are up and the bed alarm is reset after the patient gets back in bed. Which of the following components of the "Five Rights of Delegation" were used by the nurse? (Select all that apply.) 1. Right task 2. Right circumstance 3. Right person 4. Right directions and communication 5. Right supervision and evaluation

3. Answer: 1, 2, 3, 4. The nurse provided 4 of the 5 components but did not provide the right supervision and evaluation. The nurse delegated the task of a patient to the bathroom to the AP, which is in the scope of an AP's duties and responsibilities and matched to the AP skill level. The nurse did provide clear directions by describing the task and the time period to complete the task. The nurse did not use "please" and "thank you" in the request. The nurse did not ask whether there were any questions, which would provide the AP an opportunity to get clarification if needed. The nurse did not ask the AP to follow up on how the patient did or whether there were any problems. The nurse did not provide appropriate monitoring, evaluation, intervention as needed, or feedback.

3. A 36-year-old patient newly diagnosed with type 1 diabetes shares with you that he is frustrated with the time it takes to prepare meals and monitor his exercise and blood sugar. He also is having trouble understanding his insulin schedule. Which of the following suggestions would be most appropriate? (Select all that apply.) 1. Provide patient education materials that are easy to read. 2. Refer this patient to a diabetes support group. 3. Refer the patient to his endocrinologist. 4. Suggest that the patient make an appointment with a registered dietitian. 5. Suggest ways to modify his schedule.

3. Answer: 1, 2, 4, 5. Adults are often impatient with the time and energy requirements that a chronic health problem requires for proper management. Patient-centered educational materials and support groups often help patients deal with these challenges. Working with a dietitian will help the patient identify ways to modify his eating habits. Helping the patient incorporate nutritional and exercise health behaviors into his schedule is also important.

3. The nurse caring for a refugee community identifies that the children are undervaccinated and the community is unaware of resources. The nurse assesses the community and determines that there is a health clinic within a 5-mile radius. The nurse meets with the community leaders and explains the need for immunizations, the location of the clinic, and the process of accessing health care resources. Which of the following practices is the nurse providing? (Select all that apply.) 1. Raising awareness about community resources for the children 2. Teaching the community about health promotion and illness prevention 3. Promoting autonomy in decision making about health practices 4. Improving the health care of the community's children 5. Participating in professional development activities to maintain nursing competency

3. Answer: 1, 2, 4. Raising awareness about community resources for the children will help the community identify potential clinics for vaccination and well-baby and child examinations. By teaching the community about relevant illnesses, the nurse increases the level of awareness not only about the disease but also about methods of treatment. As the community becomes more informed about the illness, prevention, and treatment methods, the health of the community will increase

3. A nurse working on a medicine unit in the hospital hears the fire alarm go off. As the nurse walks down the hallway, there is smoke coming from the family waiting area. Which of the following steps should the nurse take? (Select all that apply.) 1. Immediately phone in to the hospital alert system the exact location of the fire. 2. Direct the nurse technician to place empty stretchers behind the fire doors. 3. Go to each patient room, and direct ambulatory patients to walk themselves to a safe area. 4. Work with the nurse technician to help move patients requiring wheelchairs from their rooms. 5. Close the room doors of patients who cannot get out of bed, and keep them in their rooms

3. Answer: 1, 3, 4. If a fire occurs in a health care agency, protect patients from immediate injury, report the exact location of the fire, contain it, and extinguish it if possible. The nurse should alert the hospital about the fire immediately. The nurse technician should not place stretchers behind fire doors; the fire doors need to be able to close when a fire alarm sounds. It is important to keep equipment from blocking these doors. Patients who are close to a fire, regardless of its size, are at risk of injury and need to be moved to another area. Direct all ambulatory patients to walk by themselves to a safe area. In some cases, they can help move patients in wheelchairs. Move patients who cannot get out of bed from the scene of a fire by a stretcher, their bed, or a wheelchair. Do not leave them in their rooms.

3. A nurse is caring for a 66-year-old patient who lives alone and is receiving chemotherapy and radiation for a new cancer diagnosis. He is unable to care for himself because of severe pain and fatigue. He moves into his 68-year-old brother's home so his brother can help care for him. Which assessment findings indicate that this family caregiving situation will be successful? (Select all that apply.) 1. Both the patient and his brother attend church together regularly. 2. The brothers are living together and enjoy eating the same foods. 3. Other siblings live in the same city and are willing to help. 4. The patient and his brother have a close network of friends. 5. The patient has obsessive-compulsive disorder and has difficulty throwing away possessions.

3. Answer: 1, 3, 4. Many older adults use their faith and spirituality to cope with life changes. Having a close social network and other family members who are willing to assist with caregiving helps to alleviate the stress experienced by caregivers. Potential risk factors for caregiver distress and burden in this relationship include that the brothers are living together and that one of the brothers has a mental illness.

3. Motivational interviewing (MI) is a technique that applies understanding a patient's values and goals in helping the patient make behavioral changes. When using motivational interviewing, what outcomes does the nurse expect? (Select all that apply.) 1. Gaining an understanding of the patient's motivations 2. Directing the patient to avoid poor health choices 3. Recognizing the patient's strengths and supporting his or her efforts 4. Providing assessment data that can be shared with families to promote change 5. Identifying differences in patient's health goals and current behaviors

3. Answer: 1, 3, 5. Motivational interviewing is a technique used to promote an understanding of the patient's motivations, health goals, and current behaviors in a nonjudgmental environment while focusing on the patient's strengths and efforts. The nurse provides a supportive approach to assist the patient in establishing and promoting positive health care changes.

3. After surgery the patient with a closed abdominal wound reports a sudden "pop" after coughing. When the nurse examines the surgical wound site, the sutures are open, and pieces of small bowel are noted at the bottom of the now-opened wound. Which are the priority nursing interventions? (Select all that apply.) 1. Notify the health care provider. 2. Allow the area to be exposed to air until all drainage has stopped 3. Place several cold packs over the area, protecting the skin around the wound. 4. Cover the area with sterile, saline-soaked towels immediately. 5. Cover the area with sterile gauze and apply an abdominal binder

3. Answer: 1, 4. If a patient has an opening in the surgical incision and a part of the small bowel is noted, this is evisceration. The small bowel must be protected until an emergency surgical repair can be done. The small bowel and abdominal cavity should be maintained in a sterile environment; thus sterile towels that are moistened with sterile saline should be used over the exposed bowel for protection and to keep the bowel moist

3. Which instructions do you include when educating a person with chronic constipation? (Select all that apply.) 1. Increase fiber and fluids in the diet. 2. Use a low-volume enema daily. 3. Avoid gluten in the diet. 4. Take laxatives twice a day. 5. Exercise for 30 minutes every day. 6. Schedule time to use the toilet at the same time every day. 7. Take probiotics 5 times a week.

3. Answer: 1, 5, 6. These are the steps a patient needs to take to resolve chronic problems with constipation before considering regular laxative or enema use.

3. During a nursing assessment a patient displayed several behaviors. Which behavior suggests the patient may have a health literacy problem? 1. Patient has difficulty completing a registration form at a medical office 2. Patient asks for written information about a health topic 3. Patient speaks Spanish as primary language 4. Patient states unfamiliarity with a newly ordered medicine

3. Answer: 1. Behaviors that might reflect a health literacy deficit include having difficulty completing registration forms or health histories, failing to make follow-up appointments, and asking few questions during a nursing history or physical examination.

3. A patient who is depressed is crying and verbalizes feelings of low self-esteem and self-worth, such as "I'm such a failure ... I can't do anything right." What is the nurse's best response? 1. Remain with the patient until he or she validates feeling more stable. 2. Tell the patient that is not true and that every person has a purpose in life. 3. Review recent behaviors or accomplishments that demonstrate skill ability. 4. Reassure the patient that you know how he or she is feeling and that things will get better

3. Answer: 1. Demonstrating acceptance of the patient by supportively sitting with him or her builds a therapeutic nurse-patient relationship. The nurse's presence signals value and allows the patient to explore issues of self-concept and self-esteem. In contrast, giving false hope is neither therapeutic nor conveys acceptance, while focusing on skill ability signals conditional approval.

4. Which postoperative intervention best prevents atelectasis? 1. Use of intermittent compression stockings 2. Heel-toe flexion 3. Use of the incentive spirometer 4. Abdominal splinting when coughing

4. Answer: 3. Use of the incentive spirometer expands the lungs, thus preventing the onset of atelectasis. Heel-toe flexion and the use of intermittent compression stockings prevent the onset of deep vein thrombosis. Abdominal splinting keeps pressure on abdominal incisions to prevent pain during coughing and wound dehiscence

3. A nurse is caring for a young patient who has been told he has multiple sclerosis. The nurse has planned time to conduct a teaching session that will focus on the disease and principles of management. The nurse chooses to use the EDUCATE model to proceed with instruction. Which of the following are components of the model? (Select all that apply.) 1. State goals of the session for the patient. 2. Repeat the most important information. 3. Practice empathetic skills. 4. Be aware of nonverbal messages. 5. Use a standard question list for the chosen topic

3. Answer: 2, 3, 4. Repetition of important information, using empathetic skills, and being aware of nonverbal messages are all a part of the EDUCATE model. The nurse is not the source of the goals for a teaching session. Ask patients to state their goals of care to begin a discussion. A standard list of questions may not be relevant to a patient. A more patient-centered approach is to use a question list that includes questions patients can ask so that providers can answer them

3. A nurse is teaching the mother of a young infant about prevention of sudden infant death syndrome (SIDS). Which of the following statements indicates that the teaching has been effective? (Select all that apply.) 1. "I'll let the baby sleep in bed with me so I can watch her." 2. "I'll remove stuffed animals and pillows from the crib." 3. "I'll place my baby on her back for sleep." 4. "I'll be sure to keep my baby's room cool." 5. "I'll keep a crib bumper in the bed to prevent drafts."

3. Answer: 2, 3, 4. Safeguards that reduce the risk of SIDS include proper positioning on the back; removing stuffed animals, soft bedding, and pillows; and avoiding overheating the infant

3. Communication between a nurse caring for a patient in the preoperative holding area and the circulating nurse in the operating room (OR) can best be enhanced by which of the following? (Select all that apply.) 1. Documenting assessment findings in the medical record 2. Using a standardized SBAR tool 3. Being responsive in using nonverbal communication techniques 4. Giving specific information to a transport technician 5. Listening to the OR nurse's questions

3. Answer: 2, 3, 5. Documentation does not ensure clear communication of all findings and does not allow the OR nurse to raise questions. Giving information to another staff member to communicate important information is not acceptable in a hand-off. Using standardized tools designed for hand-offs and using communication skills will enhance communication

3. A nurse sends a text message to the oncoming nurse to report that a patient refuses to take medication as ordered. What should the oncoming nurse do? (Select all that apply). 1. Add this information to the board hanging at the patient's bedside. 2. Tell the nurse who sent the text that the text is a HIPAA violation. 3. Inform the nursing supervisor. 4. Forward the text to the charge nurse. 5. Thank the nurse for sending the information.

3. Answer: 2, 3. The Health Insurance Portability and Accountability Act (HIPAA) and Health Information Technology Act provide rules about how and with whom nurses can share patient health information. Sending a text message to another nurse about a patient is a violation of these acts. Report violations of the privacy of patient health information to your supervisor or manager

3. Which of the following motivates a patient to participate in an exercise program? (Select all that apply.) 1. Providing a patient with a pamphlet on exercise 2. Providing information to the patient when he or she is ready to change behavior 3. Explaining the importance of exercise at the time of diagnosis of a chronic disease 4. Having a structured daily plan that incorporates physical activity 5. Having support from significant other to engage in exercise

3. Answer: 2, 4, 5. Patients are more open to developing an exercise program when they are at a stage of readiness to change their behavior. In addition, social support and having a daily structure that incorporates physical activity motivate patients to exercise.

3. A nurse working on a medical patient care unit states, "I am having trouble sleeping, and I eat nonstop when I get home. All I can think of when I get to work is how I can't wait for my shift to be over. I wish I felt happy again." What are the best responses from the nurse manager? (Select all that apply.) 1. "I'm sure this is just a phase you are going through. Hang in there. You'll feel better soon." 2. "I know several nurses who feel this way every now and then. Tell me about the patients you have cared for recently. Did you find it difficult to care for them?" 3. "You can take diphenhydramine over the counter to help you sleep at night." 4. "Describe for me what you do with your time when you are not working." 5. "The hospital just started a group where nurses get together to talk about their feelings. Would you like for me to e-mail the schedule to you?"

3. Answer: 2, 4, 5. This nurse is experiencing symptoms of compassion fatigue. The nurse manager needs to establish a therapeutic relationship with the nurse. Acknowledging personal thoughts and feelings and talking with other nurses to identify coping strategies can help this nurse work through the feelings associated with compassion fatigue. Engaging in healthy behaviors and establishing a good work-life balance may also help

3. Review the following nursing diagnoses and identify the diagnoses that are stated correctly. (Select all that apply.) 1. Offer frequent skin care because of Impaired Skin Integrity 2. Risk of Infection 3. Chronic Pain related to osteoarthritis 4. Activity Intolerance related to physical deconditioning 5. Lack of Knowledge related to laser surgery

3. Answer: 2, 4. Option 2 is a "risk for" diagnosis and does not have related factors. Option 4 has related factors that a nurse can manage with etiology-specific interventions. Option 1 identifies a nursing intervention instead of a problem. Option 3 has a related factor of a medical diagnosis, which a nurse cannot treat directly. Option 5 describes a treatment procedure and not the patient's response to the procedure.

3. A nurse is completing a health history with the daughter of a newly admitted patient who is confused and agitated. The daughter reports that her mother was diagnosed with Alzheimer's disease 1 year ago but became extremely confused last evening and was hallucinating. She was unable to calm her, and her mother thought she was a stranger. On the basis of this history, the nurse suspects that the patient is experiencing: 1. Normal aging. 2. Delirium. 3. Depression. 4. Worsening dementia

3. Answer: 2. Hallmark characteristics of delirium are acute confusion, hallucinations, and agitation. These symptoms are not part of the normal aging process. As dementia worsens, there is a gradual rather than sudden change in memory, usually not accompanied by hallucinations. Depression does not present with acute confusion and agitation.

3. A 72-year-old patient asks the nurse about using an over-thecounter antihistamine as a sleeping pill to help her get to sleep. What is the nurse's best response? 1. "Antihistamines are better than prescription medications because prescription medications can cause a lot of problems." 2. "Antihistamines should not be used because they can cause confusion and increase your risk of falls." 3. "Antihistamines are effective sleep aids because they do not have many side effects." 4. "Over-the-counter medications when combined with sleephygiene measures are a good plan for sleep."

3. Answer: 2. Older adults should avoid the use of over-the-counter antihistamines. These medications have a long duration of action in older adults and can cause confusion, constipation, urinary retention, and an increased risk of falls.

3. The nurse is caring for a patient with pneumonia. On entering the room, the nurse finds the patient lying in bed, coughing, and unable to clear secretions. What should the nurse do first? 1. Start oxygen at 2 L/min via nasal cannula. 2. Elevate the head of the bed to 45 degrees. 3. Encourage the patient to use the incentive spirometer. 4. Notify the health care provider.

3. Answer: 2. The HOB needs to be elevated to help increase lung expansion and ease work of breathing. Also this makes it easier for the patient to expectorate.

3. Which of the family caregivers listed below will the nurse expect to be most at risk for experiencing poor health outcomes? 1. A 20-year-old daughter caring for a mother who needs help setting up her medications weekly 2. The 68-year-old spouse of a patient who is experiencing worsening dementia 3. A 32-year-old parent of a child who has an ear infection 4. A married couple who is sharing the caregiving responsibilities for a parent who was recently diagnosed with hypertension and coronary artery disease

3. Answer: 2. This family caregiver is elderly, has most likely cared for the patient who has significant needs for more than a year, lives with the patient, and is married to the patient. Factors that put the caregiver at greatest risk for poor health outcomes include providing care for more than a year, being 65 years of age or older, providing care for a family member who has significant needs, caring for someone with Alzheimer's disease or dementia, and living with the care recipient. Spouses report the largest impact on their health.

3. The nurse teaches parents how to have their children learn impulse control and cooperative behaviors. This would be during which of Erikson's stages of development? 1. Trust versus mistrust 2. Initiative versus guilt 3. Industry versus inferiority 4. Autonomy versus sense of shame and doubt

3. Answer: 2. Toddlers are learning that parents and society have expectations about behaviors and that they must learn to control their behavior

3. When delegating input and output (I&O) measurement to assistive personnel, the nurse instructs them to record what information for ice chips? 1. Two-thirds of the volume 2. One-half of the volume 3. One-quarter of the volume 4. Two times the volume

3. Answer: 2. When ice chips melt, their water volume is one-half the volume of the ice chips. The water volume should be recorded as intake

3. A patient is placed on Airborne Precautions for pulmonary tuberculosis. The nurse notes that the patient seems to be angry, but he knows that this is a normal response to isolation. Which is the best intervention? 1. Provide a dark, quiet room to calm the patient. 2. Reduce the level of precautions to keep the patient from becoming angry. 3. Explain the reasons for isolation procedures and provide meaningful stimulation. 4. Limit family and other caregiver visits to reduce the risk of spreading the infection

3. Answer: 3 By providing a rationale for the isolation, the patient is able to better understand the safety risks and cooperate with care. Providing reading material or other distractions for the patient will also help with times when alone in the room

3. A nurse initiates a brief interview with a patient who has come to the medical clinic because of self-reported hoarseness, sore throat, and chest congestion. The nurse observes that the patient has a slumped posture and is using intercostal muscles to breathe. The nurse auscultates the patient's lungs and hears crackles in the left lower lobe. The patient's respiratory rate is 20 per minute compared with an average of 16 per minute during previous clinic visits. The patient tells the nurse, "It is hard for me to get a breath." Which of the following data sets are examples of subjective data? (Select all that apply.) 1. Heart rate of 20 per minute and chest congestion 2. Lung sounds revealing crackles and use of intercostal muscles to breathe 3. Patient statement, "It's hard for me to get a breath" 4. Slumped posture and previous respiratory rate of 16 per minute 5. Patient report of sore throat and hoarseness

3. Answer: 3 and 5. Subjective data are your patients' verbal descriptions of their health problems, in this case hoarseness, sore throat, and the statement, "It is hard for me to get a breath." All other data are objective data.

3. A nurse who recently graduated from nursing school is providing discharge instructions to a patient who suffered a myocardial infarction (MI). The nurse knows that sexual issues are common after an MI but feels uncomfortable bringing up this topic. What is the best way for the nurse to handle this situation? (Select all that apply.) 1. Instruct the patient to discuss any sexual concerns with his or her partner after discharge. 2. Avoid discussing the topic unless the patient brings it up. 3. Ask a more experienced nurse to cover this with the patient and learn from the example. 4. Plan to attend conferences or training soon on how to discuss such issues. 5. Encourage the patient to discuss any personal concerns with the cardiologist

3. Answer: 3, 4. Nurses often avoid discussing sexual issues with patients because they are uncomfortable, lack knowledge, or have personal values in conflict with the patients' values. Nurses who have difficulty addressing sexual issues need to seek education and experiences to increase knowledge and explore their personal values

3. Integrity of the oral mucosa depends on salivary secretion. Which of the following factors impairs salivary secretion? (Select all that apply.) 1. Use of cough drops 2. Immunosuppression 3. Radiation therapy 4. Dehydration 5. Presence of oral airway

3. Answer: 3, 4. Radiation therapy reduces salivary flow. Dehydration impairs salivary secretion in the mouth. Cough drops increase sugar or acid content in the mouth, causing caries. Immunosuppression causes inflammation and bleeding of the gums. An oral airway irritates oral mucosa.

4. What is the correct sequence of steps when performing wound irrigation to a large open wound? 1. Use slow, continuous pressure to irrigate wound. 2. Attach 19-gauge angiocatheter to syringe. 3. Fill syringe with irrigation fluid. 4. Place biohazard bag near bed. 5. Position angiocatheter over wound

4. Answer: 4, 3, 2, 5, 1. Organized steps ensure a safe, effective irrigation of the wound.

3. An older adult patient with bilateral hearing loss wears a hearing aid in her left ear. Which of the following approaches best facilitates communication with her? (Select all that apply.) 1. Talk to the patient at a distance so he or she may read your lips. 2. Keep your arms at your side; speak directly into the patient's left ear. 3. Face the patient when speaking; demonstrate ideas you wish to convey. 4. Position the patient so that the light is on his or her face when speaking. 5. Verify that the information that has been given has been clearly understood

3. Answer: 3, 5. To facilitate communication, face the patient when speaking, speak slower and in a normal tone, talk toward the patient's best or normal ear, articulate clearly, and demonstrate ideas you wish to convey. You should also position yourself so the light is on your face when you speak and verify that the information that has been given has been clearly understood

3. When documenting an assessment of a patient's cardiac system in an electronic health record, the nurse uses the computer mouse to select the "WNL" statement to document the following findings: "Heart sounds S1 & S2 auscultated. Heart rate between 80-100 beats per minute, and regular. Denies chest pain." This is an example of using which of the following documentation formats? 1. Focus charting incorporating "Data, Action & Response" (DAR) 2. Problem-intervention-evaluation (PIE) 3. Charting-by-exception (CBE) 4. Narrative documentation

3. Answer: 3. Charting-by-exception (CBE) is a unique documentation format designed with the philosophy that all standards are met unless otherwise documented. Many computerized nursing documentation systems have incorporated a CBE design. Exception-based documentation systems incorporate clearly defined criteria for nursing assessment and documentation of "normal" findings. Predefined statements used to document "normal" assessment of body systems are called "within defined limits" (WDL) or "within normal limits" (WNL) definitions. They consist of written criteria for a "normal" assessment for each body system. Automated documentation within a computerized documentation system allows nurses to select a WNL (or WDL) statement or to choose other statements from a drop-down menu

3. A patient is being discharged home on an around-the-clock (ATC) opioid for postoperative pain. Because of this order, the nurse anticipates an additional order for which class of medication? 1. Opioid antagonists 2. Antiemetics 3. Stool softeners 4. Muscle relaxants

3. Answer: 3. Constipation is a common opioid-related side effect, and patients do not become tolerant to it.

3. The application of deontology does not always resolve an ethical problem. Which of the following statements best explains one of the limitations of deontology? 1. The emphasis on relationships feels uncomfortable to decision makers who want more structure in deciding the best action. 2. The single focus on power imbalances does not apply to all situations in which ethical problems occur. 3. In a diverse community it can be difficult to find agreement on which principles or rules are most important. 4. The focus on consequences rather than on the "goodness" of an action makes decision makers uncomfortable

3. Answer: 3. Deontology is an approach to ethics that identifies the correct action as that which is supported by fundamental principles and duties. The disadvantage of this approach is that its application relies on consensus around what the primary duties and principles are. Option 1 describes a limitation of the ethics of care. Option 2 describes a limitation of feminist ethics, while option 4 describes a limitation of utilitarianism

3. The nurse is assessing a patient who returned 1 hour ago from surgery for an abdominal hysterectomy. Which assessment finding would require immediate follow-up? 1. Auscultation of an apical heart rate of 76 2. Absence of bowel sounds on abdominal assessment 3. Respiratory rate of 8 breaths/min 4. Palpation of dorsalis pedis pulses with strength of +2

3. Answer: 3. In healthy adults the normal respiratory rate varies from 12 to 20 respirations per minute. A rate of 8 breaths/min is too low and could be caused by anesthesia or opioid sedation effects.

3. While planning care for a patient, a nurse understands that providing integrative care includes treating which of the following? 1. Disease, spirit, and family interactions 2. Desires and emotions of the patient 3. Mind-body-spirit of patients and their families 4. Muscles, nerves, and spine disorders

3. Answer: 3. Mind-body spirit is the focus of holistic nursing

3. A patient has been hospitalized for the past 48 hours with a fever of unknown origin. His medical record indicates tympanic temperatures of 38.7°C (101.6°F) at 0400, 36.6°C (97.9°F) at 0800, 36.9°C (98.4°F) at 1200, 37.6°C (99.6°F) at 1600, and 38.3°C (100.9°F) at 2000. How would the nurse describe this pattern of temperature measurements? 1. Usual range of circadian rhythm measurements 2. Sustained fever pattern 3. Intermittent fever pattern 4. Resolving fever pattern

3. Answer: 3. Temperature was elevated above acceptable range, returned to normal, and then elevated.

3. A nurse is reading a research article discussing a new practice to decrease the incidence of catheter-associated urinary tract infections. One section of the article describes who was studied and how the data were collected to answer the research questions and hypotheses. What section of the research article is currently being read? 1. The literature review 2. The data analysis 3. The methods 4. The implications for practice

3. Answer: 3. The methods section of a study describes the study design, subjects being studied, and how the researcher collects and organizes the data to answer the research question and hypotheses. The methods section also tells you where the study was conducted, how many subjects participated in the study, and what instruments were used to collect the data.

3. A nursing student is providing a hand-off report to the RN assuming her patient's care. She explains, "I ambulated him twice during the shift; he tolerated walking to end of hall each time and back with no shortness of breath. Heart rate was 88 and regular after exercise. The patient said he slept better last night after I closed his door and gave him a chance to have some uninterrupted sleep. I changed the dressing over his intravenous (IV) site and started a new bag of D5½NS. Which intervention is a dependent intervention? 1. Providing hand-off report at change of shift 2. Enhancing the patient's sleep hygiene 3. Administering IV fluids 4. Taking vital signs

3. Answer: 3. The only intervention that requires a physician or health care provider order is IV fluid administration. All other interventions are independent

4. A nurse is assigned to care for six patients at the beginning of the night shift. The nurse learns that the floor will be short by one registered nurse (RN) as a result of a call-in. A patient care technician from another area is coming to the nursing unit to assist. Because the unit requires hourly rounds on all patients, the nurse begins to make rounds on a patient who recently asked for a pain medication. The nurse is interrupted by another registered nurse who asks about another patient. Which factors in this nurse's unit environment will affect the ability to set priorities? (Select all that apply.) 1. Policy for conducting hourly rounds 2. Staffing level 3. Interruption by staff nurse colleague 4. Type of hospital unit 5. Competency of patient care technician

4. Answer 2, 3, 5. Many factors within the health care environment affect a nurse's ability to set priorities, including availability of resources (staffing), interruptions from care providers, and RN experience and technician competency. The type of hospital unit is not a factor, but the way a unit is organized and its model of care can be factors. A policy for conducting rounds in itself does not affect ongoing priority setting.

4. The nurse must take a verbal order during an emergency on the unit. Which of the following guidelines can be used for taking verbal or telephone orders? (Select all that apply). 1. Only authorized staff may receive and record verbal or telephone orders. The health care agency identifies in writing the staff who are authorized. 2. Clearly identify patient's name, room number, and diagnosis. 3. Read back all orders to health care provider. 4. Use clarification questions to avoid misunderstandings. 5. Write "VO" (verbal order) or "TO" (telephone order), including date and time, name of patient, and complete order; sign the name of the health care provider and nurse.

4. Answer: 1, 2, 3, 4, 5. These are all acceptable guidelines for taking verbal and telephone orders in a health care setting. All of the stated guidelines should be used by the nurse.

4. Which skills does the nurse teach a patient with a new colostomy before discharge from the hospital? (Select all that apply.) 1. How to change the pouch 2. How to empty the pouch 3. How to open and close the pouch 4. How to irrigate the colostomy 5. How to determine whether the ostomy is healing appropriately

4. Answer: 1, 2, 3, 5. The patient must be able to do these tasks to successfully manage his or her colostomy when going home. Irrigation is not done routinely for a colostomy

4. Chronic illness (e.g., diabetes mellitus, hypertension, rheumatoid arthritis) may affect a person's roles and responsibilities during middle adulthood. When assessing the health-related knowledge base of both the middle-age patient with a chronic illness and his or her family, the assessment should include which of the following? (Select all that apply.) 1. Medical course of the illness 2. Prognosis for the patient 3. Coping mechanisms of the patient and family 4. Socioeconomic status 5. Need for community and social services

4. Answer: 1, 2, 3, 5. When assessing the patient with a chronic illness, it is important that the nurse know how much the patient and his family know about how the illness has progressed and the long-term prognosis for the patient. This includes understanding the patient and families' ability and readiness to accept the illness and the outlook for the patient. Understanding the coping mechanisms used by the patient and family will assist the nurse in determining how to proceed to teach and counsel the patient and family regarding the patient's treatment regimen and whether there is a need and acceptance for community or social services to assist the patient and family

4. A family is facing job loss of the father, who is the major wage earner, and relocation to a new city where there is a new job. The children will have to switch schools, and his wife will have to resign from the job she enjoys. Which of the following contribute to this family's hardiness? (Select all that apply.) 1. Family meetings 2. Established family roles 3. New neighborhood 4. Willingness to change in time of stress 5. Passive orientation to life

4. Answer: 1, 2, 4. Family hardiness refers to the internal strengths and durability of the family unit. A sense of control over the outcome of life, a view of change as beneficial and growth producing, communication with family members (as in family meetings), well-established family roles among family members, and an active rather than passive orientation in adapting to stressful events all contribute to family hardiness.

4. A nurse is teaching an older adult patient about ways to detect a melanoma. Which of the following are age-appropriate teaching techniques for this patient? (Select all that apply.) 1. Speak in a low tone. 2. Begin and end the session with the most important information regarding melanoma. 3. Provide a pamphlet about melanoma with large font in blues and greens. 4. Provide specific information in frequent, small amounts for older adult patients. 5. Speak quickly so that you do not take up much of the patient's time.

4. Answer: 1, 2, 4. Lower tones are easier for patients with hearing deficits to hear. Reinforce important information at the beginning and end of each teaching session to enhance understanding. Providing information in small amounts helps the older adult understand information better. A pamphlet should be written in a color that contrasts with the background (e.g., black 14-point print on matte white paper). Blues and greens are hard to distinguish. Take your time with an older adult. Speaking quickly can easily lead to misunderstanding.

4. Which type of personal protective equipment should the nurse wear when caring for a pediatric patient who is placed on Airborne Precautions for confirmed chickenpox/herpes zoster? (Select all that apply.) 1. Disposable gown 2. N95 respirator mask 3. Face shield or goggles 4. Disposable mask 5. Gloves

4. Answer: 1, 2, 5 Chicken pox is an airborne organism that can travel great distances, so it is important that the air breathed by the nurse is filtered, and hands and clothes are covered, as required for airborne precautions.

4. While assessing an older woman who is recently widowed, the nurse suspects that this woman is experiencing a developmental crisis. Which questions provide information about the impact of this crisis? (Select all that apply.) 1. With whom do you talk on a routine basis? 2. What do you do when you feel lonely? 3. Tell me what your husband was like. 4. I know this must be hard for you. Let me tell you what might help. 5. Have you experienced any changes in lifestyle habits, such as sleeping, eating, smoking, or drinking?

4. Answer: 1, 2, 5. A developmental crisis occurs as a person moves through the stages of life, including widowhood. It is important to gather information about how this crisis affects the woman's interactions, how she is currently coping with loneliness, and any changes in her lifestyle habits. Although losing her husband is a source of stress, discussing him now does not focus on her current situation. Saying "I know this must be hard for you. Let me tell you what might help" is unacceptable because the purpose of assessment is to gather data and let the patient tell his or her story.

4. A 20-year-old patient diagnosed with an eating disorder has a nursing diagnosis of Situational Low Self-Esteem. Which of the following nursing interventions are appropriate to address self-esteem? (Select all that apply.) 1. Offer independent decision-making opportunities. 2. Review previously successful coping strategies. 3. Provide a quiet environment with minimal stimuli. 4. Support a dependent role throughout treatment. 5. Increase calorie intake to promote weight stabilization

4. Answer: 1, 2. Offering opportunities for decision making promotes a sense of control, which is essential for promoting independence and enhancing self-esteem. Reviewing successful coping strategies is also a priority intervention to signal previous mastery and promote effective coping in an individual with self-esteem issues. The amount of stimuli is unrelated to self-esteem. Promoting independence is an important part of treatment. Although weight stabilization may be needed, it is likely to have a negative effect on self-esteem early in treatment.

4. Which nursing intervention(s) best promote(s) effective sleep in an older adult? (Select all that apply.) 1. Limit fluids 2 to 4 hours before sleep. 2. Ensure that the room is completely dark. 3. Ensure that the room temperature is comfortably cool. 4. Provide warm covers. 5. Encourage walking an hour before going to bed

4. Answer: 1, 3, 4. Limiting fluids reduces incidence of nocturia. For safety reasons complete darkness should be avoided. A soft nightlight lessens the chance of a fall should the patient require ambulation to the bathroom during the night. Older adults sometimes require extra blankets or covers to achieve a comfortable sleeping temperature. Keeping the bedroom temperature at a cooler, comfortable temperature is conducive to sleep

4. A nurse in a community health clinic has been caring for a young female teenager with diabetes for several months. The nurse's goal of care for this patient is to achieve self-management of insulin medication. Identify appropriate evaluative measures for self-management for this patient. (Select all that apply.) 1. Quality of life 2. Patient satisfaction 3. Clinic follow-up visits 4. Adherence to self-administration of insulin 5. Description of side effects of medications

4. Answer: 1, 3, 4. Quality of life is a standard measure for patients with a chronic disease such as diabetes. Adherence to medication administration, demonstrated psychomotor skill (e.g., self-injection), and adherence to medical follow-up visits are common measures of health behavior. Patient satisfaction is not a measure as to whether the patient is managing her diabetes. Description of side effects measures knowledge, not self-management, although knowledge may assist a patient to better self-manage a problem

4. The ethics of care suggests that ethical dilemmas can best be solved by attention to relationships. How does this differ from other approaches to ethical problems? (Select all that apply.) 1. Ethics of care pays attention to the context in which caring occurs. 2. Ethics of care is used only by nurses because it is part of the Nursing Code of Ethics. 3. Ethics of care requires understanding the relationships between involved parties. 4. Ethics of care considers the decision maker's relationships with other involved parties. 5. Ethics of care is an approach that suggests a greater commitment to patient care. 6. Ethic of care considers the decision maker to be in a detached position outside the ethical problem

4. Answer: 1, 3, and 4. The ethics of care emphasizes attention to the context in which an ethical problem occurs and the relationships between involved parties, including relationships with the decision maker. No approach to ethical problems is exclusive to a single discipline, and no approach is superior to the others nor does any approach demonstrate a higher level of commitment to the patient, so options 2 and 5 are incorrect. Option 6 is true of principle-based approaches such as deontology but not true of the ethics of care.

4. What assessments does a nurse make before hanging an intravenous (IV) fluid that contains potassium? (Select all that apply.) 1. Urine output 2. Arterial blood gases 3. Fullness of neck veins 4. Serum potassium laboratory value in EHR 5. Level of consciousness

4. Answer: 1, 4. Increased potassium intake when potassium output is decreased or during hyperkalemia are major risks for hyperkalemia. Before increasing IV potassium intake, check to see that urine output is normal and that the serum potassium level in the health record is not above normal

4. Which of the following best describe a collaborative health problem? (Select all that apply.) 1. An actual or potential physiological complication that nurses monitor to detect the onset of changes in a patient's health status 2. The language medical practitioners use to communicate a patient's health problem and associated treatments and response 3. A diagnostic label that classifies a patient's response to illness so that all nurses can be familiar with a specific patient's health care needs 4. A language used by health care providers to communicate and consider each other's unique perspective, so they can better manage the multiple factors that influence the health of individuals 5. A diagnosis that provides clear direction as to the type of nursing interventions nurses are licensed to provide independently

4. Answer: 1, 4. Option 2 describes a medical diagnosis, and options 3 and 5 describe a nursing diagnosis. A collaborative problem is an actual or potential physiological complication that nurses monitor to detect the onset of changes in a patient's health status. Nurses intervene in collaboration with personnel from other health care disciplines to manage collaborative problems.

4. Which of the following actions, if performed by a registered nurse, could result in both criminal and administrative law sanctions against the nurse? (Select all that apply.) 1. Reviewing the electronic health record of a family member who is a patient in the same hospital on a different unit 2. Refusing to provide health care information to a patient's child 3. Reporting suspected abuse and neglect of children 4. Applying physical restraints without a written order 5. Completing an occurrence report on the uni

4. Answer: 1, 4. Viewing a family member's electronic health record violates the patient's rights provided by HIPAA. A physical restraint can be applied only on the written order of a health care provider based on The Joint Commission and Medicare guidelines.

4. Sequence the skills in the expected order of gross-motor development in an infant, beginning with the earliest skill. 1. Can lift head 45 degrees off table, when prone 2. Pulls self to standing position 3. Sits upright without support 4. Rolls from back to abdomen 5. Rolls from abdomen to back

4. Answer: 1, 5, 4, 3, 2. Although the pace of growth and development varies for each individual, it usually follows the same pattern. Lifting of head while prone occurs first, followed by turning and rolling, and then sitting, followed by standing.

4. A patient has been laid off from his construction job and has many unpaid bills. He is going through a divorce from his marriage of 15 years and has been praying daily to help him through this difficult time. He does not have a primary health care provider because he has never really been sick, and his parents never took him to a physician when he was a child. Which external variables influence the patient's health practices? (Select all that apply.) 1. Difficulty paying his bills 2. Praying daily 3. Age of patient (46 years) 4. Stress from the divorce and the loss of a job 5. Family practice of not routinely seeing a health care provider

4. Answer: 1, 5. External factors impacting health practices include family beliefs and economic impact. The way that patients' families use health care services generally affects their health practices. Their perceptions of the seriousness of diseases and their history of preventive care behaviors (or lack of them) influence how patients think about health. Economic variables may affect a patient's level of health by increasing the risk for disease and influencing how or at what point the patient enters the health care system.

4. Which of the following is the proper sequence for a four-point crutch gait?

4. Answer: 1. Four-point alternating, or four-point, gait gives stability to the patient but requires weight-bearing on both legs. Each leg is moved alternately with each opposing crutch, so three points of support are on the floor at all times

4. When Ryan was 3 months old, he had a toy train; when his view of the train was blocked, he did not search for it. Now that he is 9 months old, he looks for it, reflecting the presence of: 1. Object permanence. 2. Sensorimotor play. 3. Schemata. 4. Magical thinking.

4. Answer: 1. He is now in Piaget's later stage of sensorimotor thought and has learned that objects exist even though he cannot see or touch them.

4. The nurse administers a tube feeding via a patient's nasogastric tube. This is an example of which of the following? 1. Physical care technique 2. Activity of daily living 3. Indirect care measure 4. Lifesaving measure

4. Answer: 1. This is an example of a physical care technique.

4. A client who is receiving parenteral nutrition (PN) through a central venous catheter (CVC) has an air embolus. What should be the nurse's priority action? 1. Have the patient turn on the left side and perform a Valsalva maneuver. 2. Clamp the intravenous (IV) tubing to prevent more air from entering the line. 3. Have the patient take a deep breath and hold it. 4. Notify the health care provider immediately

4. Answer: 1. Turn the patient on his or her left side to prevent air from entering the left side of the heart. Then have the patient perform a Valsalva maneuver (holding the breath and "bearing down").

4. Match the threats to safety on the right to the category of risk factors on the left. A. Individual Risks B. Developmental Risks 1. An older adult has limited finances. 2. A young toddler likes to explore objects by placing them in his mouth. 3. A 55-year-old patient has a residual gait change due to a stroke. 4. A school-age child chooses to play ice hockey. 5. A patient newly diagnosed with diabetes has low health literacy.

4. Answer: 1A, 2B, 3A, 4B, 5A. Individual safety risk factors include lifestyle, impaired mobility, sensory or communication impairment, limited economic resources, and a lack of safety awareness. Developmental risks include those unique to each developmental age group, such as a toddler's desire to explore orally

4. A patient presents in the clinic with dizziness and fatigue. The assistive personnel (AP) reports a slow but regular radial pulse of 44. Place the following care activities in priority order. 1. Direct the AP to obtain a blood pressure. 2. Request that the patient lie on the clinical stretcher. 3. Assess the patient's apical pulse for a full minute. 4. Prepare to administer cardiac-stimulating medications

4. Answer: 2, 1, 3, 5, 4. The first priority is patient safety. Getting the patient to lie on a stretcher prevents falls. Directing the AP to obtain BP relates to the patient's symptom of dizziness while the nurse assesses apical pulse. If BP is abnormal, the nurse should recheck value. Oxygen saturation can be obtained quickly. The patient may require medications to increase heart rate.

4. The nurse therapeutically responds to an adult patient who is anxious by: (Select all that apply.) 1. Matching the rate of speech to be the same as that of the patient 2. Providing good eye contact 3. Demonstrating a calm presence 4. Spending time attentively with the patient 5. Assuring the patient that all will be well

4. Answer: 2, 3, 4. An adult patient who is anxious is reassured by the nurse who demonstrates good eye contact and a calm presence. Also, when the adult is anxious, remaining supportively present and calm assists the patient to begin to experience less anxiety. Telling the patient all will be well is false reassurance, and the nurse may escalate the patient's anxiety if the nurse's speech is speeded up to match the patient's speech.

4. A school nurse is planning a health fair for children in first, second, and third grade to promote healthy behaviors. The most appropriate health screening for this age-group would be: (Select all that apply.) 1. Providing information about eating fruits and vegetables 2. Taking the children's blood pressure 3. Recording the children's height and weight on a growth chart 4. Asking the students about their family history of cancer 5. Teaching the students about the risks of secondhand smoke

4. Answer: 2, 3. Age-appropriate screening for this group is assessing the children's blood pressure and height and weight. The children may not know about their family history of cancer as they are too young. Providing information about healthy nutrition and secondhand smoke are health promotion interventions and do not provide screening information.

4. A postoperative patient with a three-way indwelling urinary catheter and continuous bladder irrigation (CBI) complains of lower abdominal pain and distention. What should be the nurse's initial intervention(s)? (Select all that apply.) 1. Increase the rate of the CBI. 2. Assess the patency of the drainage system. 3. Measure urine output. 4. Assess vital signs. 5. Administer ordered pain medication

4. Answer: 2, 3. An appropriate first action would be to assess the patency of the drainage system. Urine output in the drainage bag should be more than the volume of the irritant solution infused. If the system is not draining urine and irrigant, the irrigant should be stopped immediately; the catheter may be occluded and the bladder distended. Pain medication should not be administered until after assessment is completed.

4. A patient is returning to an assisted-living apartment following a diagnosis of declining, progressive visual loss. Although she is familiar with her apartment and residence, she reports feeling a little uncertain about walking alone. There is one step into her apartment. Her children are scheduling themselves to be available to their mom for the next 2 weeks. Which of the following approaches will you teach the children to assist ambulation? (Select all that apply.) 1. Walk one-half step behind and slightly to her side. 2. Have her grasp your arm just above the elbow and walk at a comfortable pace. 3. Stand next to your mom at the top and bottom of stairs. 4. Stand one step ahead of mom at the top of the stairs. 5. Place yourself alongside your mom and hold onto her waist.

4. Answer: 2, 3. To help a person with a visual impairment ambulate, offer an elbow or arm. Instruct the patient to grasp your arm just above the elbow. If necessary, physically assist the person by guiding his or her hand to your arm or elbow. When assisting a person to ascend or descend stairs, stand next to the person.

4. A new medical resident writes an order for oxycodone CR 10 mg PO q2h prn. Which part of the order does the nurse question? 1. The drug 2. The time interval 3. The dose 4. The route

4. Answer: 2. Long-acting or sustained-release opioids are dosed on a scheduled basis, not prn, to provide a base of continuous opioid analgesia

4. In addition to a thorough patient assessment, when a nurse uses one of the nursing-accessible complementary therapies, he or she must ensure that which of the following has occurred? 1. The family has provided permission. 2. The patient has provided permission and consent. 3. The health care provider has given approval or provided orders for the therapy. 4. The nurse has received specialized training in the therapeutic technique

4. Answer: 2. Nurse-accessible therapies are independent nursing interventions. As long as the scope of practice identified by the nurse's State Board of Nursing permits this activity, you do not need to obtain permission from the patient's primary provider or his or her family members unless the patient is underage. An adult can provide consent. Specialized training is not required for nursing-accessible therapies.

4. The nurse is performing discharge teaching for a patient with chronic obstructive pulmonary disease (COPD). What statement, made by the patient, indicates the need for further teaching? 1. "Pursed-lip breathing is like exercise for my lungs and will help me strengthen my breathing muscles." 2. "When I am sick, I should limit the amount of fluids I drink so that I don't produce excess mucus." 3. "I will ensure that I receive an influenza vaccine every year, preferably in the fall." 4. "I will look for a smoking-cessation support group in my neighborhood."

4. Answer: 2. Patients need to make sure that they are adequately hydrated in order to liquefy secretions, making it easier to expectorate. Fluids should not be limited or else the mucus will become too thick. All the other answers indicate an understanding of the discharge plan.

4. The nurse is gathering a history from a 72-year-old male patient being admitted to a nursing home. The patient requests a private room. The nurse understands that: 1. The patient cannot be sexually active since he is moving into a nursing home. 2. The patient may be requesting a private room to facilitate an intimate relationship with his partner. 3. There is no need to take a sexual history since most older adults are uncomfortable discussing intimate details of their lives. 4. Older adults in nursing homes usually do not participate in sexual activity

4. Answer: 2. Studies have shown an increase in sexual dysfunction with aging but no decrease in sexual activity or interest. Sometimes sexual health is not addressed by the nurse, but it is important to include a sexual history as a routine aspect of assessment to communicate that sexual activity is normal. Long-term care facilities need to make arrangements to allow for continuation of sexual experiences of residents as long as no health risks are involved.

4. The nurse asks a patient the following series of questions: "Describe for me how much you exercise each day." "How do you tolerate the exercise?" "Is the amount of exercise you get each day the same, less, or more than what you did a year ago?" This series of questions would likely occur during which phase of a patient-centered interview? 1. Orientation 2. Working phase 3. Data interpretation 4. Termination

4. Answer: 2. The working phase of a relationship involves gathering accurate, relevant, and complete information about a patient's condition. It usually begins with open-ended questions

4. The nurse is caring for a patient admitted to the neurological unit with the diagnosis of a stroke and right-sided weakness. The nurse assumes responsibility for bathing and feeding the patient until the patient can begin performing these activities. The nurse in this situation is applying the theory developed by: 1. Johnson. 2. Orem. 3. Roy. 4. Peplau

4. Answer: 2. When applying Orem's self-care deficit theory, the nurse continually assesses the patient's ability to perform selfcare and intervenes as needed to ensure that physical, psychological, sociological, and developmental needs are being met. As the patient's condition improves, the nurse encourages the patient to begin doing these activities independently.

4. The nurse works at an agency where military time is used for documentation, and needs to document that a patient was transported to the operating room for an emergency procedure at 8 in the evening. Point to the area on the clockface below that indicates 8 in the evening in military time:

4. Answer: 2000. The military clock begins at 1 minute after midnight as 0001 and ends with midnight at 2400. Noon is 1200. 1 pm is 1300, 2 pm is 1400, 3 pm is 1500, and so on.

4. The REFLECT model can improve learning after providing patient care. Place the steps of this model in the correct order: 1. Think about your thoughts and actions at the time of a situation. 2. Review the knowledge you gained from the experience. 3. Review the facts of the situation. 4. Set a schedule for completing your plan of action. 5. Consider options for handling a similar situation in the future. 6. Recall any feelings you had at the time of the situation. 7. Create a plan for future situations.

4. Answer: 3, 1, 6, 2, 5, 7, 4. This is the correct order following the REFLECT model

4. While administering medications, a nurse realizes that a prescribed dose of a medication was not given. The nurse acts by completing an incident report and notifying the patient's health care provider. Which of the following is the nurse exercising? 1. Authority 2. Responsibility 3. Accountability 4. Decision making

4. Answer: 3. Accountability is nurses being answerable for their actions. It means nurses accept the commitment to provide excellent patient care and the responsibility for the outcomes of the actions in providing that. Following institutional policy for reporting medication errors demonstrates the nurse's commitment to safe patient care.

4. A nurse is assigned to care for the following patients. Which patient is most at risk for developing skin problems and thus requiring thorough bathing and skin care? 1. A 44-year-old female patient who has had removal of a breast lesion and is having her menstrual period 2. A 56-year-old male patient who is homeless and admitted to the emergency department with malnutrition and dehydration and who has an intravenous line 3. A 60-year-old female patient who experienced a stroke with rightsided paralysis and has an orthopedic brace applied to the left leg 4. A 70-year-old patient who has diabetes and dementia and has been incontinent of stool

4. Answer: 4. The 70-year-old patient has reduced circulation, which increases risk for infection, and is likely unaware of skin problems because of dementia. The presence of stool will also irritate the skin. The 44-year-old female patient needs good perineal hygiene. The 56-year-old patient is at risk for drying and fragility of the skin. The 60-year-old patient has reduced sensation and mobility and thus is unaware of skin problems or pressure areas.

4. A nurse implements an EBP change that teaches patients the importance of taking their diabetes medications correctly and regularly on time using videos streamed on the Internet. The nurse measures the patients' behavioral outcome from the practice change using which type of measurement? 1. Measuring the patient's weight 2. Chart auditing teaching sessions 3. Observing patients viewing the videos 4. Checking patients' blood sugars

4. Answer: 4. The desired behavioral outcome is the patients' blood sugars, which will show the patients' adherence to taking medications as prescribed. Measuring the patients' weight is a desirable physical outcome but not a measure of the effects of the teaching program. Charting auditing teaching sessions is a process measure to track teaching sessions. Observing the patients viewing the DVD is also a process measure.

4. A nurse has the responsibility of managing a patient's postmortem care. What is the proper order for postmortem care when there is no autopsy ordered? 1. Bathe the body of the deceased. 2. Collect any needed specimens. 3. Remove all tubes and indwelling lines. 4. Position the body for family viewing. 5. Speak to the family members about their possible participation. 6. Ensure that the request for organ/tissue donation and/or autopsy was completed. 7. Notify support person (e.g., spiritual care provider, bereavement specialist) for the family. 8. Accurately tag the body, including the identity of the deceased and safety issues regarding infection control. 9. Elevate the head of the bed

4. Answer: 6, 9, 2, 5, 7, 3, 1, 4, 8. This order provides dignity to the deceased and ensures that the nurse is adhering to all policies and laws concerning autopsies, organ donation, or an investigation

4. Place the following steps in the correct order for positioning a patient in the 30-degree lateral side-lying position. 1. Raise side rail and go to opposite side of bed. 2. Lower side rail and flex patient's knee that will not be next to mattress. Keep foot on mattress and place one hand on patient's upper bent leg near hip and other hand on shoulder. 3. Lower head of bed flat if patient can tolerate it. 4. Roll patient onto side toward you. 5. Lower side rail and position patient on side of bed opposite the direction toward which patient is to be turned. 6. Place hands under patient's dependent shoulder and bring shoulder blade forward. 7. Place hands under patient's dependent hip and bring hip slightly forward so that angle from hip to mattress is approximately 30 degrees

4. Answer: The correct order in the steps is 3, 5, 1, 2, 4, 6, 7

4. A nurse desires to communicate with a young woman who is Serbian and who has limited experience with being in a hospital. The nurse has 10 years of experience caring for Serbian women. The patient was admitted for a serious pregnancy complication. Apply the LEARN model and match the nurse's behaviors with each step of the model. 1. L ___________ a. The nurse notes that she has learned that fathers can visit mothers at any time in both Serbia and the United States. 2. E ___________ b. The nurse shares her perception of the woman's experiences as a patient. 3. A ___________ c. The nurse asks the patient how she can maintain bed rest when she returns home. 4. R __________ d. The nurse attends to the patient and listens to her story about hospitals in Serbia. 5. N __________ e. The nurse involves the patient in a discussion of the treatment options for her condition.

4. Answers: 1d, 2b, 3a, 4e, 5c. Listen with empathy and understanding to patient perception of the problem; Explain your perceptions of the problem (physiological, psychological, spiritual, and/or cultural; Acknowledge and discuss cultural differences and similarities between you and your patient; Recommend treatment (involving the patient); and Negotiate agreement (incorporate selected aspects of the patient's culture into patient-centered care).

5. A nurse working on a surgery floor is assigned four patients. The nurse assesses each patient, noting behaviors and physical signs and symptoms. Which of the following patients is more likely to be violent toward the nurse? 1. The first patient maintains eye contact with the nurse, is calm during the nurse's assessment, and asks questions frequently. 2. The second patient is very drowsy, loses attention span when the nurse asks questions, and mumbles when speaking. 3. The third patient moves nervously in bed, swears and grimaces when trying to cough, and speaks in a low volume. 4. The fourth patient speaks in a loud voice and becomes irritable when the nurse arrives to help walk the patient.

5. Answer 4. Patients who are most likely to enact violence include those who have an increased volume of speech, are irritable, demonstrate prolonged or intense glaring, mumble, use abusive language toward the nurse, and pace around the waiting area or bed

5. The infection control nurse has asked the staff to work on reducing the number of iatrogenic infections on the unit. Which of the following actions on your part would contribute to reducing health care-acquired infections? (Select all that apply.) 1. Teaching correct handwashing to assigned patients 2. Using correct procedures in starting and caring for an intravenous infusion 3. Providing perineal care to a patient with an indwelling urinary catheter 4. Isolating a patient on antibiotics who has been having loose stool for 24 hours 5. Decreasing a patient's environmental stimuli to decrease nausea

5. Answer: 1, 2, 3 Nausea is not typically associated with transmission of infection, and loose stools are a common side effect with antimicrobials. All the other interventions break the cycle of infection transmission.

5. A nurse is caring for a patient with chronic arthritis pain. The patient wants to add some complementary therapies to help with pain management. Which therapies might be most effective for controlling pain (Select all that apply): 1. Biofeedback 2. Acupuncture 3. Therapeutic touch 4. Chiropractic therapy 5. Herbal medicines

5. Answer: 1, 2, 3. Biofeedback is a mind-body technique that promotes relaxation and muscle tension and, in turn, reduces pain. Acupuncture modifies the body's response to pain and over time can reduce pain. Therapeutic touch is effective in reducing the pain response in patients with chronic illnesses. Chiropractic therapy realigns structure and reduces pain when the pain is the result of structural abnormality, not inflammation. Meditation assists in relaxing the body and stilling the mind. It allow a person to cope with stress, reduce anxiety, and feel at one with God and the universe. Herbal therapies are often used to prevent disease and promote health.

5. The nurse can increase a patient's self-awareness and self-concept through which of the following actions? (Select all that apply.) 1. Helping the patient define personal problems clearly 2. Allowing the patient to openly explore thoughts and feelings 3. Reframing the patient's thoughts and feelings in a more positive way 4. Having family members assume more responsibility during times of stress 5. Recommending self-help reading materials

5. Answer: 1, 2, 3. Helping a patient define problems clearly, allowing him or her to openly explore thoughts and feelings, and reframing his or her thoughts and feelings in a more positive way are designed to promote self-awareness and a positive self-concept. Having the family assume more responsibility does not help a patient achieve self-awareness; instead it is important to encourage the patient to assume more self-responsibility. The nurse should refrain from offering self-help reading materials unless directly asked; the nurse should then provide numerous options.

5. An 85-year-old patient returns to the inpatient surgical unit after leaving the PACU. Which of the following place the patient at risk during surgery? (Select all that apply.) 1. Stiffened lung tissue 2. Reduced diaphragmatic excursion 3. Increased laryngeal reflexes 4. Reduced blood flow to kidneys 5. Increased cholinergic transmission

5. Answer: 1, 2, 4. Older adults have stiffened lung tissue, reduced diaphragmatic excursion, and reduced blood flow to kidneys. Laryngeal reflexes are reduced, increasing risk for aspiration, and reduced cholinergic transmission puts them at risk for cognitive changes

5. A nurse sees a 76-year-old woman in the outpatient clinic. She states that she recently started to notice a glare in the lights at home. Her vision is blurred, and she is unable to play cards with her friends, read, or do her needlework. Which of the following nursing interventions are appropriate? (Select all that apply.) 1. Refer her to an ophthalmologist. 2. Suggest large-print books and playing cards. 3. Reassure her that this is part of normal aging. 4. Suggest lower-wattage light bulbs to decrease glare. 5. Assess her home environment for safety

5. Answer: 1, 2, 5. This patient most likely has cataracts and should be referred to an ophthalmologist. While common, cataracts are not considered to be part of normal aging. In the meantime, using large-print books or playing cards and reducing home safety hazards would be beneficial. Lower-wattage light bulbs would not be helpful.

5. One element of clinical decision making is knowing the patient. Which of the following activities affect a nurse's ability to know patients better? (Select all that apply.) 1. Caring for similar groups of patients over time 2. Reading the evidence-based practices appropriate to patients 3. Learning how patients typically respond to their clinical situations 4. Observing patients 5. Engaging with patients experiencing illness

5. Answer: 1, 3, 4, 5. Knowing the patient includes a nurse's understanding of a specific patient and his or her subsequent selection of interventions. Knowing the patient relates to a nurse's experience with caring for patients, time spent in a specific clinical area, and having a sense of closeness with patients. Reading evidence-based practices builds your knowledge about nursing science but does not provide the knowledge needed through engaging with and observing patients

5. The nurse is providing education on sexually transmitted infections (STIs) to a group of older adults. The nurse knows that further teaching is needed when the participants make which statements? (Select all that apply.) 1. "I don't need to use condoms since there is no risk for pregnancy." 2. "I should be screened for an STI each time I'm with a new partner." 3. "I know I'm not infected because I don't have discharge or sores." 4. "I was tested for STIs last year, so I know I'm not infected." 5. "The infection rate in older adults is low because most are not sexually active."

5. Answer: 1, 3, 4, 5. One of the challenges in reducing the incidence of STIs is that most STIs have few symptoms in males or females. Asymptomatic STIs can be diagnosed during a physical examination with appropriate laboratory tests. Older adults may engage in risky sexual behaviors because of lack of knowledge about STIs and condom usage. Research indicates that older adults are remaining sexually active longer than previously believed and the incidence of STI and human immunodeficiency virus (HIV) infections has steadily increased for the past 12 years. Screening after each new sex partner is the most effective method to detect and manage STIs, so this statement shows understanding of the teaching by the patient.

5. Which skin-care measures are used to manage a patient who is experiencing fecal and/or urinary incontinence? (Select all that apply.) 1. Frequent position changes 2. Keeping the buttocks exposed to air at all times 3. Using a large absorbent diaper, changing when saturated 4. Using an incontinence cleaner 5. Applying a moisture barrier ointment

5. Answer: 1, 4, 5. Skin that is in contact with stool and urine can become moist and soft, allowing it to become damaged. The stool contains bacteria and in some cases enzymes that can harm the skin if in contact for a prolonged period of time. The use of an incontinence cleaner provides a gentle removal of stool and urine, and the use of the moisture-barrier ointment provides a protective layer between the skin and the next incontinence episode. However, skin care and moisture barriers must also be used with frequent position changes to help reduce the risk for pressure injuries.

5. From the following list of indicators, determine which indicators are goals (G) and which indicators are outcomes (O). 1. _____ Will achieve pain relief 2. _____ Ambulates 10 feet down hallway 3. _____ Will remain free of infection 4. _____ Will be afebrile 5. _____ Reports pain severity reduced from 6 to a 4 on scale of 0 to 10 6. _____ Will gain improved mobility

5. Answer: 1-goal, 2-outcome, 3-goal, 4-goal, 5-outcome, 6-goal. A nursing plan of care includes mutually established goals and nurse-sensitive outcomes relevant to a patient's health status. Goals are broad statements that describe a desired change in a patient's condition, perceptions, or behavior. A nurse-sensitive outcome is a state, behavior, or perception that is measured along a continuum in response to a nursing intervention

5. Which of the following is a diagnostic error involving identification of a goal of care rather than a patient need? 1. Patient obtains social support care related to caregiver stress 2. Fear related to open-heart surgery 3. Acute Pain related to splinting of incision 4. Impaired Family Coping related to insufficient caregiver support

5. Answer: 1. A nursing diagnosis must identify a patient response to a health care problem and not the goal of care. Achieving social support would be a goal of care in a plan of care for relieving caregiver stress. Answers 2, and 3 are diagnostic errors: 2 makes the related factor a surgical procedure, 3 uses a symptom as a related factor. 4 is a correctly worded nursing diagnosis.

5. Which statement made by the patient indicates an understanding of sleep-hygiene practices? 1. "I usually drink a cup of warm milk in the evening to help me sleep." 2. "If I exercise right before bedtime, I will be tired and fall asleep faster." 3. "I know it does not matter what time I go to bed as long as I am tired." 4. "If I use hypnotics for a long time, my insomnia will be cured."

5. Answer: 1. Drinking a warm beverage such as milk in the evening can help promote sleep. Milk contains l-tryptophan, which helps promote sleep. Other snacks that contain l-tryptophan, such as cereal and cheese and crackers, may also promote sleep. Exercising right before bedtime may prevent sleep. Good sleep hygiene includes going to bed and getting up at the same time daily. Hypnotics can help with insomnia but are not curative.

5. A patient who is newly diagnosed with breast cancer states, "Although I am really scared about what is going to happen to me, I know my family will learn from this experience, and we will be stronger in the end." What term does the nurse use in the patient's medical record to describe the characteristic displayed in this statement? 1. Resiliency 2. End-of-life care 3. Family functioning 4. Family's culture

5. Answer: 1. Family resiliency helps families maintain a healthy response even when they are experiencing stressful events.

5. The nurse is caring for an older adult in a long-term care setting. The nurse reviews the medical record to find that the patient has progressive loss of total bone mass. The patient's history and tendency to take smaller steps with feet kept closer together will most likely result in which of the following? 1. Increase the patient's risk for falls and injuries 2. Result in less stress on the patient's joints 3. Decrease the amount of work required for patient movement 4. Allow for mobility in spite of the aging effects on the patient's joints

5. Answer: 1. The effect of bone loss is weaker bones, causing vertebrae to be softer and long shaft bones to be less resistant to bending, making an individual prone to fractures and muscle injuries. Older adults may walk more slowly and incorrectly and appear less coordinated. Many are afraid of falling. They often take smaller steps and keep their feet closer together, which decreases the base of support and thus alters body balance, increasing fall risk.

5. A nurse is conducting a home visit with a new mom and her three children. While in the home the nurse weighs each family member and reviews their 3-day food diary. She checks the mom's blood pressure and encourages the mom to take the children for a 15- to 30-minute walk every day. The nurse is addressing which level of need, according to Maslow? 1. Physiologica 2. Safety and security 3. Love and belonging 4. Self-actualization

5. Answer: 1. The nurse's actions address the basic physiological needs of nutrition, physical activity, and oxygen. According to Maslow, basic needs must be met before meeting higher level needs

5. Match the following types of theory with the appropriate description. 1. Middle-range theory 2. Shared theory 3. Grand theory 4. Practice theory a. Very abstract; attempts to describe nursing in a global context b. Specific to a particular situation; brings theory to the bedside c. Applies theory from other disciplines to nursing practice d. Addresses a specific phenomenon and reflects practice

5. Answer: 1d, 2c, 3a, 4b.

5. A nurse is administering ophthalmic ointment to a patient. Place the following steps in correct order for the administration of the ointment. 1. Clean eye, washing from inner to outer canthus. 2. Assess patient's level of consciousness and ability to follow instructions. 3. Apply thin ribbon of ointment evenly along inner edge of lower eyelid on conjunctiva. 4. Have patient close eye and rub lightly in a circular motion with a cotton ball. 5. Ask patient to look at ceiling, and explain the steps to patient.

5. Answer: 2, 1, 5, 3, 4. This is the correct order for safe administration of ophthalmic ointment

5. The nurse plans care for a 16-year-old male, taking into consideration that stressors experienced most commonly by adolescents include which of the following? (Select all that apply.) 1. Loss of autonomy caused by health problems 2. Physical appearance and body image 3. Accepting one's personal identity 4. Separation from family 5. Taking tests in school

5. Answer: 2, 3, 4, 5. As adolescents search for identity with peer groups and separate from their families, they also experience stress. In addition, they face stressful questions about sex, jobs, school, career choices, and using mind-altering substances. During this stage of development, stress can occur because of a preoccupation with appearance and body image. A loss of autonomy caused by health problems usually applies to the older adult.

5. The nurse is teaching a patient to prevent heart disease. Which information should the nurse include? (Select all that apply.) 1. Add salt to every meal. 2. Talk with your health care provider about taking a daily low dose of aspirin. 3. Work with your health care provider to develop a regular exercise program. 4. Limit daily intake of fats to less than 25% to 35% of total calories. 5. Review strategies to encourage the patient to quit smoking

5. Answer: 2, 3, 4, 5. Teaching about prevention of heart disease focuses on risk factor reduction. Smoking, lack of regular aerobic exercise, and a diet high in sodium and fats are three major risk factors that can be modified. Quitting smoking, regular exercise, and a diet with lower sodium and fat intake are preventive measures. Low-dose aspirin has been shown to be beneficial in reducing the risk of heart disease.

5. Which of the following are strategies for creating work environments that support nurse caring interventions? (Select all that apply.) 1. Increasing technological support 2. Improving flexibility for scheduling 3. Providing opportunities to discuss care 4. Promoting autonomy of practice 5. Encouraging increased input concerning nursing functions from health care providers

5. Answer: 2, 3, 4. These factors all affect nursing satisfaction. When nurses' job satisfaction is high, nurses are more connected with their patients and view their caring practices as part of the nursing culture. Increasing technology frequently removes the nurse from patient care and can have a negative impact on job satisfaction.

5. A 44-year-old male patient has just been told that his wife and child were killed in an auto accident while coming to visit him in the hospital. Which of the following statements are assessment findings that support a nursing diagnosis of Spiritual Distress related to loss of family members? (Select all that apply.) 1. "I need to call my sister for support." 2. "I have nothing to live for now." 3. "Why would my God do this to me?" 4. "I need to pray for a miracle." 5. "I want to be more involved in my church."

5. Answer: 2, 3. Patients most likely to have a diagnosis of Spiritual Distress are facing loss or terminal or serious illness and have poor personal relationships. Indicating that there is nothing to live for now and wondering why God would do this to him reflect dispiritedness (e.g., expressing lack of hope, meaning, or purpose in life; anger toward God). The other responses show a potential for enhancement of spiritual well-being

5. A young male patient enters the emergency department with fever and signs of a possible sexually transmitted infection. The nurse enters the patient's cubicle and begins to enter a history on the computer screen. Before beginning the nurse introduces himself and tells the patient all information will be held confidentially. The nurse starts data collection by establishing eye contact with the patient and then looks at the computer prompts to select a series of questions. As the nurse fills out questions on the computer, the patient asks a question about his treatment. The nurse states, "Let me get through these questions first." Which action interferes with the nurse's ability to use connection as a communication skill. 1. Introducing self to patient 2. Using the computer as a prompt for questions 3. Making the nurse's questions a priority 4. Assuring the patient all information is confidential

5. Answer: 3. Introducing self is a form of courtesy, as is informing the patient of the confidentiality of patient information. Using the computer as only a prompt and not solely focusing on the computer is appropriate. However, making your assessment questions more of a priority than those of the patient interferes with and prevents good connection.

5. A nurse is providing health promotion information at a health fair for female patients who are diagnosed with cancer. What information should the nurse include? (Select all that apply.) 1. Recommending that they avoid drinking alcohol to prevent alcohol misuse 2. Information from the local health department about smoking-cessation classes 3. The need to avoid physical activity while receiving cancer treatment to lessen fatigue 4. Strategies to talk with family and friends about the cancer diagnosis and the side effects from their treatment 5. How nutritional needs may change based on the diagnosis of cancer and its treatment

5. Answer: 2, 4, 5. Providing education about the benefits of smoking cessation and resources available to stop smoking promotes health. Friends and family may become tired of hearing about a patient's chronic illness experience. Helping the attendees learn effective ways to communicate with friends and family about their cancer and its treatment effects will promote understanding. Patients need information on how to make healthy dietary choices to promote their health. Diet choices often need to be modified when a patient is diagnosed with a chronic illness. Alcohol misuse occurs when a woman drinks more than 3 drinks a day or 7 drinks a week. People with chronic diseases need to participate in the same amount of activity and muscle-strengthening as healthy people whenever possible after consulting with their health care provider regarding the types and amounts of activity that are appropriate.

5. An ambulatory elderly woman with dementia is incontinent of urine. She has poor short-term memory and has not been seen toileting independently. What is the best nursing intervention for this patient? 1. Recommend that she be evaluated for an overactive bladder (OAB) medication. 2. Establish a toileting schedule. 3. Recommend that she be evaluated for an indwelling catheter. 4. Start a bladder-retraining program

5. Answer: 2. The first nursing intervention for any patient with incontinence who is able to toilet is to help him or her with toilet access. This patient has dementia; therefore a bladder-retraining program is inappropriate for her. There is nothing in the assessment to indicate that she may have an overactive bladder. A catheter increases risk for infection and is never the best intervention for incontinence.

5. A patient in the intensive care unit experiences a sentinel event related to central-line catheter care that resulted in serious injury. What performance improvement model should the unit use to identify errors that led to the sentinel event? 1. Six Sigma 2. Root cause analysis 3. PDSA 4. Balanced scorecard

5. Answer: 2. The unit should conduct a root cause analysis. The purpose of the root cause analysis is to gather data and information to identify active and latent errors that occurred leading to the sentinel event. The other options are quality improvement models that focus on process and performance measures leading to practice change. They may be used to improve processes or practice after the root cause analysis has identified the underlying errors that led to the sentinel event.

5. The health care provider's order is 500 mL 0.9% NaCl intravenously over 4 hours. Which rate does the nurse program into the infusion pump? 1. 100 mL/hr 2. 125 mL/hr 3. 167 mL/hr 4. 200 mL/hr

5. Answer: 2. To infuse 500 mL in 4 hours, set the rate at 125 mL/hr. (500 divided by 4 = 125)

5. A new nurse is going to help a patient walk down the corridor and sit in a chair. The patient has an eye patch over the left eye and poor vision in the right eye. What is the correct order of steps to help the patient safely walk down the hall and sit in the chair? 1. Tell patient when you are approaching the chair. 2. Walk at a relaxed pace. 3. Guide patient's hand to nurse's arm, resting just above the elbow. 4. Position yourself one-half step in front of patient. 5. Position patient's hand on back of chair

5. Answer: 3, 4, 2, 1, 5. These steps ensure safety when guiding a patient with impaired vision to walk and sit in a chair. In addition, they help to provide patient education and independence in carrying out activities of daily living.

5. When the nurse is assigned to a patient who has a reduced level of consciousness and requires mouth care, which physical assessment techniques should the nurse perform before the procedure? (Select all that apply.) 1. Oxygen saturation 2. Heart rate 3. Respirations 4. Gag reflex 5. Response to painful stimulus

5. Answer: 3, 4. Check a patient's respirations and whether there is a gag reflex present to determine risk for aspiration and to establish a baseline for the patient's condition.

5. A nursing student is providing a hand-off report to a registered nurse (RN) who is assuming her patient's care at the end of the clinical day. The student states, "The patient had a good day. His intravenous (IV) fluid is infusing at 124 mL/hr with D5½NS infusing in left forearm. The IV site is intact, and no complaints of tenderness. I ambulated him twice during the shift; he tolerated walking to the visitors lounge and back with no shortness of breath, respirations 14, heart rate 88 after exercise. He uses his walker without difficulty, gait normal. The patient ate ¾ of his dinner with no gastrointestinal complaints. For the goal of improving the patient's activity tolerance, which expected outcomes were shared in the hand-off? (Select all that apply.) 1. IV site not tender 2. Uses walker to walk 3. Walked to visitors lounge 4. No shortness of breath 5. Tolerated dinner meal

5. Answer: 3, 4. In this case, outcomes to determine whether a patient is tolerating activity include measures of exercise tolerance, including respiratory status and distance walked. Using a walker is an intervention; the fact that the walker is used without difficulty is a measure of mobility, not activity tolerance. The IV site being nontender is an outcome for Risk of Infection. Tolerating a dinner meal could be an outcome for appetite problem or nausea

5. A 50-year-old woman has elevated serum cholesterol levels that increase her risk for cardiovascular disease. One method to control this risk factor is to identify the patient's current diet trends and describe dietary changes to reduce the risk. This nursing activity is a form of: 1. Referral. 2. Counseling. 3. Health education. 4. Stress-management techniques.

5. Answer: 3. Health teaching offers dietary information to the woman to enable her to make decisions about her dietary health practices in an attempt to lower her cholesterol.

5. The following are steps in the process to help resolve an ethical problem. What is the best order of these steps to achieve resolution? 1. List all the possible actions that could be taken to resolve the problem. 2. Articulate a statement of the problem or dilemma that you are trying to resolve. 3. Develop and implement a plan to address the problem. 4. Gather all relevant information regarding the clinical, social, and spiritual aspects of the problem. 5. Take time to clarify values and identify the ethical elements, such as principles and key relationships involved. 6. Recognize that the problem requires ethics

5. Answer: 6, 4, 5, 2, 1, 3. This order reflects a systematic approach to ethical problems, similar to the nursing process

5. A nurse received bedside report at the change of shift with the night-shift nurse and the patient. The nursing student assigned to the patient asks to review the patient's medical record. The nurse lists patients' medical diagnoses on the message boards in the patients' rooms. Later in the day the nurse discusses the plan of care for a patient who is dying with the patient's family. Which of these actions describes a violation of the Health Insurance Portability and Accountability Act (HIPAA)? 1. Discussing patient conditions at the bedside at the change of shift 2. Allowing the nursing student to review the assigned patient's chart before providing care during the clinical experience 3. Posting medical information about the patient on a message board in the patient's room 4. Releasing patient information regarding terminal illness to family when the patient has given permission for information to be shared

5. Answer: 3. Posting the medical condition of a patient on a message board in the patient's room is not necessary for the patient's treatment. Doing so can result in this information being accessed by persons who are not involved in the patient's treatment.

5. The nurse reviews a patient's medical administration record (MAR) and finds that the patient has received oxycodone/acetaminophen (5/325), two tablets PO every 3 hours for the past 3 days. What concerns the nurse the most? 1. The patient's level of pain 2. The potential for addiction 3. The amount of daily acetaminophen 4. The risk for gastrointestinal bleeding

5. Answer: 3. The Food and Drug Administration (FDA) recommends a maximum daily dose of 4 g of acetaminophen

5. Which principle is most important for a nurse to follow when using a clinical practice guideline for an assigned patient? 1. Knowing the source of the guideline 2. Reviewing the evidence used to develop the guideline 3. Individualizing how to apply the clinical guideline for a patient 4. Explaining to a patient the purpose of the guideline

5. Answer: 3. Whenever a nurse applies a practice guideline, it is essential to consider the patient's unique needs and how to adapt and deliver the guideline. Knowing the source and reviewing the evidence are helpful in knowing the strength of any guideline. Explaining the purpose keeps a patient informed but is not the most important principle.

5. A 55-year-old adult male has been in the hospital over a week following surgical complications. The patient has had limited activity but is now finally ordered to begin a mobility program. The patient just returned from several diagnostic tests and tells the nurse he is feeling quite fatigued. The nurse prepares to instruct the patient on the mobility program protocol. Which of the following learning principles will likely be affected by this patient's condition? 1. Motivation to learn 2. Developmental stage 3. Stage of grief 4. Readiness to learn

5. Answer: 4. A patient's readiness to learn is affected by his or her attentional set. Physical discomfort, fatigue, anxiety, confusion, and environmental distractions influence the ability to concentrate and learn.

5. Which task is appropriate for a registered nurse (RN) to delegate to an AP? 1. Explaining to the patient the preoperative preparation before the surgery in the morning 2. Administering the ordered antibiotic to the patient before surgery 3. Obtaining the patient's signature on the surgical informed consent 4. Helping the patient to the bathroom before leaving for the operating room

5. Answer: 4. Assisting the patient with toileting activities is within the scope of an AP's duties. The other activities require the skill and knowledge of the RN

5. Parents are concerned about their toddler's negativism. To avoid a negative response, which of the following is the best way for a nurse to demonstrate asking the toddler to eat lunch? 1. Would you like to eat your lunch now? 2. Would you like to sit at the big table to eat? 3. When would you like to eat your lunch with your friends? 4. Would you like apple slices or applesauce with your sandwich?

5. Answer: 4. Nurses and parents can deal with the negativism by limiting the opportunities for a "no" answer.

5. When preparing a 4-year-old child for a procedure, which method is developmentally most appropriate for the nurse to use? 1. Allowing the child to watch another child undergoing the same procedure 2. Showing the child pictures of what he or she will experience 3. Talking to the child in simple terms about what will happen 4. Preparing the child through play with a doll and toy medical equipment

5. Answer: 4. Preschoolers are in the preoperational stage of cognitive development and learn more easily when play is used to teach.

5. A patient is receiving both parenteral (PN) and enteral nutrition (EN). When would the nurse collaborate with the health care provider and request a discontinuation of parenteral nutrition? 1. When 25% of the patient's nutritional needs are met by the tube feedings 2. When bowel sounds return 3. When the central line has been in for 10 days 4. When 75% of the patient's nutritional needs are met by the tube feedings

5. Answer: 4. When meeting 75% of nutritional needs by enteral feedings or reliable dietary intake, it is usually safe to discontinue PN therapy.

5. A nurse prepares to contact a patient's physician about a change in the patient's condition. Put the following statements in the correct order using SBAR (Situation, Background, Assessment, and Recommendation) communication. 1. "She is a 53-year-old female who was admitted 2 days ago with pneumonia and was started on levofloxacin at 5 pm yesterday. She states she has a poor appetite; her weight has remained stable over the past 2 days." 2. "The patient reported feeling very nauseated after her dose of levofloxacin an hour ago." 3. "Is it possible to make a change in antibiotics, or could we give her a nutritional supplement before her medication?" 4. "The patient started to complain of nausea yesterday evening and has vomited several times during the night.

5. Answer: 4S, 1B, 2A, 3R. The nurse describes the patient's complaint of nausea and vomiting to the physician (Situation). Specific patient demographic information and reason for admission with current symptomology are provided (Background). The physician is informed of the patient's complaint of nausea after receiving levofloxacin (Assessment). Physician is asked if he or she would like to make a change in the antibiotic or provide a nutritional supplement before medication administration (Recommendation).

5. Place the steps for an ileostomy pouch change in the correct order. 1. Close the end of the pouch. 2. Measure the stoma. 3. Cut the hole in the wafer to fit around the stoma and not leave skin exposed to the effluent. 4. Press the pouch in place over the stoma. 5. Remove the old pouch. 6. Trace the correct measurement onto the back of the wafer 7. Assess the stoma and the skin around it. 8. Cleanse and dry the peristomal skin

5. Answer: 5, 8, 7, 2, 6, 3, 4, 1. This order of tasks describes the correct way to change an ostomy pouch

5. Health care organizations must provide which of the following based on federal civil rights laws? (Select all that apply.) 1. Provide language assistance services at all points of contact free of charge. 2. Provide auxiliary aids and services, such as interpreters, note takers, and computer-aided transcription services. 3. Use patients' family members to interpret difficult topics. 4. Ensure that interpreters are competent in medical terminology. 5. Provide language assistance to all patients who speak limited English or are deaf.

5. Answers: 1, 2, 4, 5. The CLAS standards include standards for communication and language assistance, including providing language assistance free of charge, auxiliary aids and services, interpreters competent in medical terminology, and language assistance for patients with limited English or who are deaf

5. The nurse who works at the local hospital is transferring a patient to an acute rehabilitation center in another town. To complete the transfer, information from the patient's electronic health record must be printed and faxed to the acute rehabilitation center. Which of the following actions is most appropriate for the nurse to take to maintain privacy and confidentiality of the patient's information when faxing this information? (Select all that apply.) 1. Confirm that the fax number for the acute rehabilitation center is correct before sending the fax. 2. Use the encryption feature on the fax machine to encode the information and make it impossible for staff at the acute rehabilitation center to read the information unless they have the encryption key. 3. Fax the patient's information without a cover sheet so that the person receiving the information at the acute rehabilitation center can identify it more quickly. 4. After sending the fax, place the information that was printed out in a standard trash can after ripping it into several pieces. 5. After sending the fax, place the information that was printed out in a secure canister marked for shredding

5. Answers: 1, 2, 5. Nurses have the legal and ethical obligation to safeguard any patient information that is printed or extracted from the electronic (or paper) health record. Best practice is to use all measures to fax information securely, and to shred any printed health record material after it has been used for the purpose intended.

6. When using ice massage for pain relief, which of the following is correct? (Select all that apply.) 1. Apply ice using firm pressure over the skin. 2. Apply ice for 5 minutes or until numbness occurs. 3. Apply ice no more than 3 times a day. 4. Limit application of ice to no longer than 10 minutes. 5. Use a slow, circular steady massage

6. Answer: 1, 2, 5. Apply the ice with firm pressure to the skin, which is covered with a lightweight cloth. Then use a slow, steady circular massage over the area. Apply cold within a 6-inch circular area near the pain site or on the opposite side of the body corresponding to the pain site. Limit application to 5 minutes or when the patient feels numbness. Application near the actual site of pain tends to work best, and it can be applied several times each hour to help reduce pain

6. Which symptoms are warning signs of possible colorectal cancer according to the American Cancer Society guidelines? (Select all that apply.) 1. Change in bowel habits 2. Blood in the stool 3. A larger-than-normal bowel movement 4. Fecal impaction 5. Muscle aches 6. Incomplete emptying of the colon 7. Food particles in the stool 8. Unexplained abdominal or back pain

6. Answer: 1, 2, 6, 8. According to the American Cancer Society current guidelines, persons with these symptoms should seek medical evaluation because they may have colon cancer. Other conditions may also cause these symptoms, but if colon cancer is present, early diagnosis is important

6. A nurse has been caring for a patient over 2 consecutive days. During that time the patient had an intravenous (IV) catheter in the right forearm. At the end of shift on the second day the nurse inspects the catheter site, observes for redness, and asks whether the patient feels tenderness when the site is palpated. The nurse reviews the medical record from 24 hours ago and finds the catheter site was without redness or tenderness. Which of the activities below reflect the nurse's ability to perform patient evaluation? (Select all that apply.) 1. Comparing patient response with previous response 2. Examining results of clinical data 3. Recognizing error 4. Self-reflection 5. Checking medical record for when IV was inserted

6. Answer: 1, 2. In this situation the nurse performs evaluation by initially examining the patient and gathering subjective and objective data to compare with previous findings. The data give information about the status of the IV site. At this time no error has been identified and no self-reflection has thus occurred. Checking the medical record for IV insertion is not an evaluation of the effect the IV has on the patient.

6. Intimate partner violence (IPV) is linked to which of the following factors? (Select all that apply.) 1. Alcohol abuse 2. Marriage 3. Pregnancy 4. Unemployment 5. Drug use

6. Answer: 1, 3, 4, 5. IPV is linked to harmful health behaviors such as alcohol abuse and drug use. Other risk factors include unemployment and pregnancy.

6. Which of the following factors should be considered when choosing an intervention for a patient's plan of care? (Select all that apply.) 1. The specific patient outcome against which to judge effectiveness of interventions 2. The timing of care activities routinely conducted on the care unit 3. The scientific evidence available in support of an intervention 4. The amount of time required for implementation in consideration of patient's condition 5. The patient's values and beliefs regarding the intervention

6. Answer: 1, 3, 4, 5. When choosing interventions, consider six important factors: (1) desired patient outcomes, (2) characteristics of the nursing diagnosis, (3) research base knowledge for the intervention, (4) feasibility for doing the intervention, (5) acceptability to the patient, and (6) your own competency

6. What should the nurse teach a young woman with a history of urinary tract infections (UTIs) about UTI prevention? (Select all that apply.) 1. Maintain regular bowel elimination. 2. Limit water intake to 1 to 2 glasses a day. 3. Wear cotton underwear. 4. Cleanse the perineum from front to back. 5. Practice pelvic muscle exercise (Kegel) daily.

6. Answer: 1, 3, 4. Maintaining regular bowel elimination prevents the rectum from filling with stool, which can irritate the bladder. Adequate hydration will ensure that the bladder is regularly flushed and will help prevent a UTI. Cotton undergarments are recommended. Pelvic muscle exercises promote pelvic health but do not necessarily prevent UTIs

6. Which of the following are outcomes measurements? (Select all that apply.) 1. A nurse teaches a patient how to administer an injection and then observes the patient do a return demonstration. 2. A nurse implements a new pain-management protocol and checks patients' charts to confirm whether interventions are being provided. 3. A nursing unit adopts a set of strategies for reducing pressure injuries, and the UPC members use direct observation of the skin to measure incidence of pressure injuries. 4. A nursing unit implements a new fall-prevention protocol and checks the monthly performance data for incidence of falls on the unit. 5. A nursing unit implements a patient rounding program, and the charge nurse watches the assistive personnel to see whether hourly rounding is being done on patients

6. Answer: 1, 3, 4. Outcomes measurements are the observable or measurable effects of health care interventions. A nurse observing a patient's return demonstration, direct observation of patients' skin to measure incidence of pressure injuries, and checking the monthly performance data for incidence of falls are all outcome measures. The other options are examples of process measurements.

6. The American Dental Association suggests that patients who are at risk for poor hygiene use the following interventions for oral care: (Select all that apply.) 1. Use antimicrobial toothpaste. 2. Brush teeth 4 times a day. 3. Use 0.12% chlorhexidine gluconate (CHG) oral rinses. 4. Use a soft toothbrush for oral care. 5. Avoid cleaning the gums and tongue

6. Answer: 1, 3, 4. The American Dental Association guidelines (2018) for effective oral hygiene include brushing the teeth at least twice a day with an American Dental Association-approved fluoride toothpaste. Use antimicrobial toothpastes and 0.12% CHG oral rinses for patients at increased risk for poor oral hygiene (e.g., older adults and patients with cognitive impairments and who are immunocompromised). Rounded soft bristles stimulate the gums without causing abrasion and bleeding. Patients should clean the gums and the surface of the tongue.

6. Which of the following describe characteristics of an integrated health care system? (Select all that apply.) 1. The focus is holistic. 2. Participating hospitals follow the same model of health care delivery. 3. The system coordinates a continuum of services. 4. The focus of health care providers is finding a cure for patients. 5. Members of the health care team link electronically to use the EMR to share the patient's health care record.

6. Answer: 1, 3, 5. Integrated health care systems are shifting to more holistic approaches to health care. At the core of this shift is provision of a coordinated continuum of services for enhancing the health status of defined populations. There is no single model for an integrated health care system. Two types of integrated health care delivery systems are common: an organizational structure that follows economic imperatives and a system that supports an organized care delivery approach. Patient-centered medical home care is an example; members of the care team are linked by information technology, electronic health records, and system-best practices to ensure that patients receive care when and where they need it, and how they want it

6. The patient states, "I don't have confidence in my doctor. She looks so young." The nurse therapeutically responds: (Select all that apply.) 1. Tell me more about your concern. 2. You have nothing to worry about. Your doctor is perfectly competent. 3. You are worried about your care? 4. You can go online and see how others have rated your doctor. I do that. 5. You should ask your doctor to tell you her background

6. Answer: 1, 3.The nurse responding to the patient's concern about the physician's age would not disagree with the patient by simply claiming the doctor was competent but would rather ask questions to elicit more information about the area of concern, such as asking a broader question about concerns. Telling the patient to look the physician up online or advising the patient to query the physician directly are ways that the nurse unhelpfully gives advice to the patient.

6. A patient with progressive vision impairments had to surrender his driver's license 6 months ago. He comes to the medical clinic for a routine checkup. He is accompanied by his son. His wife died 2 years ago, and he admits to feeling lonely much of the time. Which of the following interventions reduce loneliness? (Select all that apply.) 1. Sharing information about senior transportation services 2. Reassuring the patient that loneliness is a normal part of aging 3. Maintaining distance while talking to avoid overstimulating the patient 4. Providing information about local social groups in the patient's neighborhood 5. Recommending that the patient consider making living arrangements that will put him closer to family or friends

6. Answer: 1, 4, 5. Loneliness is not a normal part of aging. Principles for reducing loneliness include providing information about local social groups and recommending alterations in living arrangements if physical isolation occurs. Access to senior transportation services is important, since transportation challenges can lead to social isolation

6. A 63-year-old patient is retiring from his job at an accounting firm where he was in a management role for the past 20 years. He has been with the same company for 42 years and was a dedicated employee. His wife is a homemaker. She raised their five children, babysits for her grandchildren as needed, and belongs to numerous church committees. What are the major concerns for this patient? (Select all that apply.) 1. The loss of his work role 2. The risk of social isolation 3. A determination on whether the wife will need to start working 4. How the wife expects household tasks to be divided in the home in retirement 5. The age the patient chose to retire

6. Answer: 1, 4. The psychosocial stresses of retirement are usually related to role changes with a spouse or within the family and to loss of the work role. Often there are new expectations of the retired person. This patient is not likely to become socially isolated because of the size of the family. Whether the wife will have to work is not a major concern at this time, nor is the age of the patient.

6. A patient is in skeletal traction and has a plaster cast due to a fractured femur. The patient experiences decreased sensation and a cold feeling in the toes of the affected leg. The nurse observes that the patient's toes have become pale and cold but forgets to document this because one of the nurse's other patients experienced cardiac arrest at the same time. Two days later the patient in skeletal traction has an elevated temperature, and he is prepared for surgery to amputate the leg below the knee. Which of the following statements regarding a breach of duty apply to this situation? (Select all that apply.) 1. Failure to document a change in assessment data 2. Failure to provide discharge instructions 3. Failure to provide patient education about cast care. 4. Failure to use proper medical equipment ordered for patient monitoring 5. Failure to notify a health care provider about a change in the patient's condition

6. Answer: 1, 5. The failure to document a change in assessment data and the failure to notify a health care provider about a change in patient status reflect a breach of duty to the patient.

6. A postoperative patient experiences tachypnea during the first hour of recovery. Which nursing intervention is a priority? 1. Elevate the head of the patient's bed. 2. Give ordered oxygen through a mask at 4 L/min. 3. Ask the patient to use an incentive spirometer. 4. Position the patient on one side with the face down and the neck slightly extended so that the tongue falls forward.

6. Answer: 1. Elevating the head of the patient's bed is a quick intervention that does not require a prescription, but it will promote lung expansion and allow secretions to move via gravity. Administration of oxygen requires a prescription. While using the incentive spirometer expands the lungs, it would not be the first action as positioning the patient to breath effectively is necessary

6. A nurse is caring for a patient newly diagnosed with testicular cancer. He asked the nurse to help him find the meaning of cancer by supporting beliefs about life. This is an example of: 1. Instilling hope and faith. 2. Forming a human-altruistic value system. 3. Cultural caring. 4. Being with.

6. Answer: 1. Instilling hope and faith helps increase an individual's capacity to get through an event or transition and face a future with meaning.

6. The nurse is providing community education about how the sexual response changes with age. Which statement made by one of the adults indicates the need for further information? 1. "Health problems such as diabetes, chronic obstructive pulmonary disease, and hypertension have little effect on sexual functioning and desire." 2. "It usually takes longer for both sexes to reach an orgasm." 3. "Most of the normal changes in function are related to alteration in circulation and hormone levels." 4. "Many medications can interfere with sexual function."

6. Answer: 1. Pathological processes can interfere with sexual function and desire. Changes in circulation, neurological pathways, and hormone levels account for many of the normal physiological changes that occur with the aging process. Common medications such as diuretics, antihypertensives, antianxiety medications, and antidepressants can contribute to sexual dysfunction. Older males and females take longer to reach orgasm, and the refractory period lengthens

6. A nurse is preparing medications for a patient. The nurse checks the name of the medication on the label with the name of the medication on the doctor's order. At the bedside the nurse checks the patient's name against the medication order as well. The nurse is following which critical thinking attitude? 1. Responsibility 2. Humility 3. Accurate 4. Fairness

6. Answer: 1. Responsibility involves performing procedures correctly by following standards of care. Accurate, broad, and fair are intellectual standards. Humility is the recognition of when you need more information to make a decision

6. When taking care of patients, a nurse routinely asks whether they take any vitamins or herbal medications, encourages family members to bring in music that the patient likes to help the patient relax, and frequently prays with her patients if that is important to them. The nurse is practicing which model? 1. Holistic 2. Health belief 3. Transtheoretical 4. Health promotion

6. Answer: 1. The nurse is using a holistic model of care that takes a more holistic view of health by considering emotional and spiritual well-being and other dimensions of an individual to be important aspects of physical wellness. The holistic health model of nursing attempts to create conditions that promote optimal health. Nurses using the holistic nursing model recognize the natural healing abilities of the body and incorporate complementary and alternative interventions such as music therapy, reminiscence, relaxation therapy, therapeutic touch, and guided imagery because they are effective, economical, noninvasive, nonpharmacological complements to traditional medical care.

6. Which of the following actions by the nurse demonstrate the practice of core principles of surgical asepsis? (Select all that apply.) 1. The front and sides of the sterile gown are considered sterile from the waist up. 2. Keep the sterile field in view at all times. 3. Consider the outer 2.5 cm (1 inch) of the sterile field as contaminated. 4. Only health care personnel within the sterile field must wear personal protective equipment. 5. After cleansing the hands with antiseptic rub, apply clean disposable gloves.

6. Answer: 2, 3 Maintaining sterility throughout the procedure requires constant vigilance and strict rules to ensure sterility, such as keeping the sterile field in sight at all times, making sure everyone in the room is in protective clothing like gowns, masks, eyewear and gloves, and considering anything beyond the front or below the waist of the gown to be contaminated. To make sure the sides of the sterile field are not contaminated, there is an outer one-inch border not considered sterile.

6. A nurse is assigned to a new patient admitted to the medical unit. The nurse collects a nursing history and interviews the patient. Place the following steps for making a nursing diagnosis in the correct order 1. Consider the context of patient's health problem and select a related factor. 2. Review assessment data, noting objective and subjective clinical information. 3. Cluster clinical data elements that form a pattern. 4. Identify appropriate assessment findings for diagnosis. 5. Identify a nursing diagnosis.

6. Answer: 2, 3, 5, 1, 4.

6. A nurse working the night shift is assigned a patient who has a history of having fallen in the hospital during a previous admission. The nurse wants to review the admission assessment completed by the nurse on the day shift. Which of the following sections in the assessment are most likely to provide information about the patient's current fall risks? (Select all that apply.) 1. Allergy history 2. Medication history 3. Patient age 4. Patient's occupation 5. Physical exam of neuromuscular function

6. Answer: 2, 3, 5. A patient's age will reveal his or her developmental status. The medication history is important to determine whether the patient is taking medications that typically predispose patients to falling. The examination of neuromuscular function will reveal whether the patient has any problems with cognitive status, muscle strength and coordination, balance, and gait—all of which can predispose to falls. The allergy history and occupational history will not reveal risk factors for patients to fall.

6. A 46-year-old patient is admitted to the emergency department following an automobile accident. The patient has a pelvic fracture and is ordered on bed rest and placed in an immobilization device to limit further injury until the fracture can safely be repaired Which measures would be appropriate for this patient to prevent complications of bed rest? (Select all that apply.) 1. Administer intravenous analgesic as ordered. 2. Have patient perform incentive spirometry. 3. Support patient in active assistive ROM exercises of upper extremities. 4. Provide patient a low-calorie diet. 5. Apply sequential compression devices to legs

6. Answer: 2, 3, 5. Because the patient is immobilized and has an unstable pelvic fracture, administration of IV analgesics is appropriate for pain, but is not focused on prevention of immobility complications. Use of incentive spirometry prevents pulmonary atelectasis. The patient has no upper extremity limitations, so active assistive ROM is appropriate and preventive. The patient should have compression devices on the legs to reduce risk of DVT. A low-calorie diet would be improper. Immobilized patients require a high-calorie intake.

6. Which of the following assessment findings suggest an altered self-concept? (Select all that apply.) 1. Uneven gait 2. Slumped posture and poor personal hygiene 3. Avoidance of eye contact when answering a question 4. Requests for visits from the chaplain 5. Frequent use of the call light

6. Answer: 2, 3. Common assessment findings for an individualwith altered self-concept can mirror depressive symptoms, such as slumped posture, poor hygiene, and avoiding intermittent eye contact.An individualwith an unsteady or uneven gait may have successfully adjusted to an underlying condition; this does not automatically signal an altered self-concept. Requests for spiritual support and nursing care should be honored and are not related to an altered self-concept.

6. A 10-year-old girl was playing on a slide at a playground during a summer camp. She fell and broke her arm. The camp notified the parents and took the child to the emergency department according to the camp protocol for injuries. The parents arrive at the emergency department and are stressed and frantic. The 10-yearold is happy in the treatment room, eating a Popsicle and picking out the color of her cast. List in order of priority what the nurse should say to the parents 1. "Can I contact someone to help you?" 2. "Your daughter is happy in the treatment room, eating a Popsicle and picking out the color of her cast." 3. "I'll have the doctor come out and talk to you as soon as possible." 4. "I want to be sure you are ok. Let's talk about what your concerns are about your daughter before we go see her."

6. Answer: 2, 4, 3, 1. First and most important the parents need to know the immediate status of their daughter. Letting them know the situation will help to relieve their immediate stress. Second, helping the parents discuss their concerns will reduce their stress and will allow them to see their daughter without increasing the 10-year-old's anxiety. Third, let the parents know that you recognize their need to talk to the doctor as soon as possible and that you will act as their advocate to get that accomplished. Last, but also important, you want to ask whether there is anyone you can call to help. There may be children who need to be picked up from camp/ day care, for example, and a neighbor or grandparent may be able to assist.

6. Which nursing interventions are appropriate to include in a plan of care to promote sleep for patients who are hospitalized? (Select all that apply.) 1. Give patients a cup of coffee 1 hour before bedtime. 2. Plan vital signs to be taken before the patients are asleep. 3. Turn television on 15 minutes before bedtime. 4. Have patients follow at-home bedtime schedule. 5. Close the door to patients' rooms at bedtime

6. Answer: 2, 4, 5. Bedtime routines relax patients in preparation for sleep. Patients in the hospital should follow their at-home bedtime routine. Taking vital signs before sleep onset prevents disruption of sleep and improves sleep duration and quality. Closing the door to patients' rooms decreases noise that can disrupt sleep. Noise is one of the main factors contributing to poor sleep in hospitalized patients. Excessive stimulation, such as watching television, should be avoided close to bedtime.

6. A nurse is visiting a patient who lives alone at home. The nurse is assessing the patient's adherence to medications. While talking with the family caregiver, the nurse learns that the patient has been missing doses. The nurse wants to perform interventions to improve the patient's adherence. Which of the following will affect how this nurse will make clinical decisions about how to implement care for this patient? (Select all that apply.) 1. Reviewing the family caregiver's availability during medication administration times 2. Determining the value the patient places on taking medications 3. Reviewing the number of medications and time each is to be taken 4. Determining all consequences associated with the patient missing specific medicines 5. Reviewing the therapeutic actions of the medications

6. Answer: 2, 4. Adherence is related to how a patient values a particular treatment and whether negative consequences are perceived if the treatment is not followed. Reviewing the family caregiver availability will be useful if the nurse cannot help the patient adhere better. Reviewing the number of medications will not change adherence unless the nurse consults with the physician about simplifying the regimen. Reviewing therapeutic actions of medication ensures safer administration but not better adherence.

6. A nurse is caring for a patient experiencing a stress response. The nurse plans care with the knowledge that systems respond to stress in what manner? (Select all that apply.) 1. Always fail and cause illness and disease 2. Cause negative responses over time 3. React the same way for all individuals 4. Protect an individual from harm in the short term 5. Tolerate the stress response indefinitely

6. Answer: 2, 4. In the beginning stress responses serve as a warning and physiological "alarm" of sorts, preparing the person to respond to harm. In this way they can be a protective mechanism. However, stress that continues unmitigated for long periods of time creates states of "exhaustion" that translate ultimately into negative physiological and psychological events.

6. A patient is receiving palliative care for symptom management related to anxiety and pain. A family member asks whether the patient is dying and now in "hospice." What does the nurse tell the family member about palliative care? (Select all that apply.) 1. Palliative care and hospice are the same thing. 2. Palliative care is for any patient, any time, any disease, in any setting. 3. Palliative care strategies are primarily designed to treat the patient's illness. 4. Palliative care relieves the symptoms of illness and treatment. 5. Palliative care selects home health care services

6. Answer: 2, 4. Palliative care and hospice care are different. Palliative care is available to all patients regardless of age, diagnosis, and prognosis. The focus of palliative care is on management of symptoms

6. The nurse is administering an IV push medication to a patient who has a compatible IV fluid running through intravenous tubing. Place the following steps in the appropriate order 1. Release tubing and inject medication within amount of time recommended by agency policy, pharmacist, or medication reference manual. Use watch to time administration. 2. Select injection port of IV tubing closest to patient. Whenever possible, injection port should accept a needleless syringe. Use IV filter if required by medication reference or agency policy. 3. After injecting medication, release tubing, withdraw syringe, and recheck fluid infusion rate. 4. Connect syringe to port of IV line. Insert needleless tip or small-gauge needle of syringe containing prepared drug through center of injection port 5. Clean injection port with antiseptic swab. Allow to dry. 6. Occlude IV line by pinching tubing just above injection port. Pull back gently on syringe plunger to aspirate blood return.

6. Answer: 2, 5, 4, 6, 1, 3. These are the correct steps to administer an IV push medication in an existing line with compatible fluid running

6. When nurses are communicating with adolescents, they should: 1. Ask closed-ended questions to get straight answers. 2. Ask the adolescent to collaborate on plan of care. 3. Avoid looking for meaning behind adolescents' words or actions. 4. Avoid discussing sensitive issues such as sex and drugs

6. Answer: 2. Do not avoid discussing sensitive issues. Asking questions about sex, drugs, and school opens the channels for further discussion. Ask open-ended questions. Look for the meaning behind their words or actions. Be alert to clues to their emotional state. Adolescents should be involved in their plan of care.

6. A patient has just learned she has been diagnosed with a malignant brain tumor. She is alone; her family will not be arriving from out of town for an hour. The nurse has been caring for her for only 2 hours but has a good relationship with her. What is the most appropriate intervention for support of her spiritual well-being at this time? 1. Make a referral to a professional spiritual care adviser. 2. Sit down and talk with the patient; have her discuss her feelings and listen attentively 3. Move the patient's Bible from her bedside cabinet drawer to the top of the over-bed table. 4. Ask the patient whether she would like to learn more about the implications of having this type of tumor.

6. Answer: 2. Establishing presence contributes to a patient's sense of well-being. It helps to prevent emotional and environmental isolation. Automatically making a referral to a spiritual care adviser might not be the patient's wish. She may not see an adviser as a resource. Reading a Bible can be an important ritual, but at this time the patient needs to make a connection with someone who can help minimize loneliness and powerlessness. Providing instruction will be important, but the patient is unlikely to be receptive at this time.

6. The nurse is caring for a family where there is a strong family history of breast cancer. One of the family members says, "I am afraid of having genetic testing. If it is positive, I know I have cancer." What is the nurse's best response? 1. "It will help diagnose the cancer early if you have it." 2. "If the results are positive, it means you have a higher risk for breast cancer, not that you have cancer. 3. "If it is going to cause you to worry, don't have the screening done." 4. "I am sure you will be fine. You are a healthy woman."

6. Answer: 2. This patient does not understand genetic testing and needs further information. A positive genetic screen for breast cancer indicates the genetic mutation exists in a woman's DNA, placing her at an increased risk for developing one of these cancers. It does not mean that the woman will develop one of these cancers

6. Which of the following describes a hydrocolloid dressing? 1. A seaweed derivative that is highly absorptive 2. Premoistened gauze placed over a granulating wound 3. A debriding enzyme that is used to remove necrotic tissue 4. A dressing that forms a gel that interacts with the wound surface

6. Answer: 4. A hydrocolloid dressing is made of materials that are adhesive and can form a gel over the open area of the wound. Since moisture enhances wound healing, the gel that forms places the wound in the proper environment for healing

6. A community health nurse conducts a community assessment focused on adolescent health behaviors. The nurse determines that a large number of adolescents smoke. Designing a smoking cessation program at the youth community center is an example of which nursing role? 1. Educator 2. Counselor 3. Collaborator 4. Case manager

6. Answer: 2. To engage a patient to participate in a smoking-cessation program or any program that requires changing a behavior requires the nurse to act as a counselor to support the patient in changing that behavior (e.g., in this case a smoking habit). The nurse does not educate the patient about the dangers of smoking but first must actively counsel him or her to decide to change the behavior. Without support and counseling, the smoking-cessation education may not be used effectively by the patient

6. An older-adult patient is receiving intravenous (IV) 0.9% NaCl. The nurse detects new onset of crackles in the lung bases. What is the priority action? 1. Notify a health care provider 2. Decrease the IV flow rate. 3. Lower the head of the bed. 4. Discontinue the IV site.

6. Answer: 2. When an IV fluid is infusing, monitor for excess infusion. Crackles in the lung bases are an indication of ECV excess. For patient safety, the IV flow rate must be decreased immediately. Then notify the health care provider

6. A hospice nurse is caring for a family that is providing end-of-life care for their grandmother, who has terminal breast cancer. The nurse focuses on symptom management for the grandmother and on helping the family with developing coping skills. This approach is an example of which of the following? 1. Family as context 2. Family as patient 3. Family as a system 4. Family as structure

6. Answer: 2. When the family as patient is the approach, the family's needs, processes, and relationships (e.g., parenting or family caregiving) are the primary focuses of nursing care.

6. A nurse observes a patient walking down the hall with a shuffling gait. When the patient returns to bed, the nurse checks the strength in both of the patient's legs. The nurse applies the information gained to suspect that the patient has a mobility problem. This conclusion is an example of: 1. Reflection. 2. Clinical inference. 3. Cue. 4. Validation.

6. Answer: 2. You begin to cluster cues that seem to relate together, make inferences, and identify emerging patterns. Clinical inference is part of the clinical decision-making process and precedes any judgment or decision about what are a patient's problems. It is the interpretation of the cues.

6. A nurse performs the following four steps in delegating a task to an AP. Place the steps in the order of appropriate delegation. 1. Do you have any questions about walking Mr. Malone? 2. Before you take him for his walk to the end of the hallway and back, please take and record his pulse rate 3. In the next 30 minutes please assist Mr. Malone in Room 418 with his afternoon walk. 4. I will make sure that I check with you in about 40 minutes to see how the patient did

6. Answer: 3, 2, 4, 1. This is the sequence of effective delegation. The nurse delegated the task of walking a patient to the AP, which is in the scope of the AP's duties and responsibilities and matched to the AP's skill level. The nurse provided clear directions by describing the task (the walk, taking and recording the pulse), the desired outcome (walk to the end of the hallway and back), and the time period (within the next 30 minutes). The nurse explains the process of follow-up with the AP to check how the patient did. The nurse asks whether the AP has any questions to provide the AP the opportunity to ask questions for clarification

6. A client is receiving an enteral feeding at 65 mL/hr. The gastric residual volume in 4 hours was 125 mL. What is the priority nursing intervention? 1. Assess bowel sounds. 2. Raise the head of the bed to at least 45 degrees. 3. Continue the feedings; this is normal gastric residual for this feeding. 4. Hold the feeding until you talk to the primary care provider

6. Answer: 3. Delayed gastric emptying is a concern if 250 mL or more remains in a patient's stomach on two consecutive assessments (1 hour apart) or if a single GRV measurement exceeds 500 mL. Therefore the best action is to continue the tube feedings at this time.

6. A nurse working in a large occupational health clinic knows that many of the workers at her company are marginalized and at risk for poor health outcomes. Which of the following individuals are most likely to be marginalized? 1. Wives of the employees 2. The head supervisors of the company 3. Workers who have a high school education 4. Workers employed for less than a year at the company

6. Answer: 3. Marginalized groups are more likely to have poor health outcomes and die earlier because of a complex interaction among their individual behaviors, environment of the communities in which they live, the policies and practices of health care and governmental systems, and the clinical care they receive. Examples of marginalized groups include people who are gay, lesbian, bisexual, or transgender; people of color; people who are physically and/ or mentally challenged; and people who are not college educated.

6. A patient recovering from open heart surgery is taught how to cough and deep breathe using a pillow to support or splint the chest incision. Following the teaching session, which of the following is the best way for the nurse to evaluate whether learning has taken place? 1. Verbalization of steps to use in splinting 2. Selecting from a series of flash cards the images showing the correct technique 3. Return demonstration 4. Cloze test

6. Answer: 3. Return demonstration permits a patient to perform a skill as the nurse observes. It provides excellent feedback and reinforcemen

6. The nurse is administering a dose of metoprolol to a patient, and is completing the steps of bar code medication administration within the EHR. As the bar code information on the medication is scanned, an alert that states "Do not administer dose if apical heart rate (HR) is <60 beats/minute or systolic blood pressure (SBP) is <90 mm Hg" appears on the computer screen. The alert that appeared on the computer screen is an example of what type of system? 1. Electronic health record (EHR) 2. Charting by exception 3. Clinical decision support system (CDSS) 4. Computerized physician order entry (CPOE)

6. Answer: 3.Computer decision supportsystems(CDSS) are computerized programs that prompt health care providers with clinical knowledge and relevant patient information that assistswith clinical decision making. A nursing CDSS uses a complex system of rules to analyze data and provide alertsto support clinical decisions made by nurses.

8. Match the category of direct care on the left with the specific direct care activity on the right. 1. Counseling ___ 2. Lifesaving measure ____ 3. Physical care technique ___ 4. Activity of daily living ____ a. Assisting patient with oral care b. Discussing a patient's options in choosing palliative care c. Protecting a violent patient from injury d. Using safe patient handling during positioning of a patient

8. Answer: 1 b, 2 c, 3 d, 4 a

7. A nurse asks how a patient's condition from a serious infection changed since yesterday while receiving a hand-off report. The nurse leaving the shift reports the patient has two priority nursing diagnoses—fluid imbalance and fever. The receiving nurse begins to provide care by measuring the patient's body temperature, inspecting the condition of the skin, reviewing the intake and output record, and checking the summary notes describing the patient's progress since the day before. The nurse asks a technician to measure intake and output during the shift. What critical thinking indicators reflect the nurse's ability to perform evaluation? (Select all that apply.) 1. Checking the summary notes 2. Asking the leaving RN about the patient's condition. 3. Assigning the technician to measure intake and output 4. Comparing current outcomes with those set for the patient's goals 5. Reflecting on patient's progress

7. Answer: 1, 2, 4, 5. Critical thinking indicators reflecting evaluation include examining the results of care according to clinical data collected, comparing achieved effects or outcomes with goals and expected outcomes, recognizing errors or omissions, and reflecting on the patient situation. Delegating a task such as intake and output measurement to a technician is appropriate but not a critical thinking indicator for evaluation.

7. Which of the following statements related to theory-based nursing practice are correct? (Select all that apply.) 1. Nursing theory differentiates nursing from other disciplines. 2. Nursing theories are standardized and do not change over time. 3. Integrating theory into practice promotes coordinated care delivery. 4. Nursing knowledge is generated by theory. 5. The theory of nursing process is used in planning patient care. 6. Evidence-based practice results from theory-testing research.

7. Answer: 1, 3, 4, 6. The overall goal of nursing knowledge is to explain the practice of nursing as different and distinct from the practice of medicine, psychology, and other health care disciplines. Theory generates nursing knowledge for use in practice, thus supporting evidence-based practice. The integration of theory into practice leads to coordinated care delivery and therefore serves as the basis for nursing. Although the nursing process is central to nursing, it is not a theory. Nursing theories are not stagnant and continue to evolve over time

7. Which are examples of positive health habits that may prevent the development of chronic illness later in life? (Select all that apply.) 1. Routine screening and diagnostic tests 2. Unprotected sexual activity 3. Regular exercise 4. Consistent seat belt use 5. Excess alcohol consumption

7. Answer: 1, 3, 4. Routine screening and diagnostic tests (i.e., laboratory screening for serum cholesterol or serum glucose levels, mammography or colonoscopy) will provide early detection of health issues. Regular exercise helps maintain weight and improve musculoskeletal functioning. Seat belt use saves lives and reduces the extent of injury in motor vehicle accidents

7. Put the following steps for removal of protective barriers after leaving an isolation room in order. 1. Remove and dispose of gloves. 2. Perform hand hygiene. 3. Remove eyewear or goggles. 4. Untie top and then bottom mask strings and remove from face. 5. Untie waist and neck strings of gown. Remove gown, rolling it onto itself without touching the contaminated side

7. Answer: 1, 3, 5, 4, 2 Removing isolation PPE correctly decreases the risk of self-contamination. The gloves are considered the most contaminated pieces of PPE and are therefore removed first. The face shield or goggles are next because they interfere with removal of other PPE. The gown is third, followed by the mask or respirator

7. A nurse in a community health clinic reviews screening results from students in a local high school during the most recent academic year. The nurse discovers a 10% increase in the number of positive tuberculosis (TB) skin tests when comparing these numbers to the previous year. The nurse contacts the school nurse and the director of the health department. Together they begin to expand their assessment to all students and employees of the school district. The community nurse was acting in which nursing role(s)? (Select all that apply.) 1. Epidemiologist 2. Counselor 3. Collaborator 4. Case manager 5. Caregiver

7. Answer: 1, 3. When the community health nurse initially noticed an increase in the number of positive TB skin tests, the nurse was comparing current data with previous data to track positive skin test rates. Once the increase was noted, the community health nurse collaborated with the school nurse and other members of the health department to determine the impact of the increased positive TB skin tests.

7. The nurse is gathering a sexual health history on a patient being admitted to the hospital for surgery. Which question demonstrates a nonjudgmental attitude? 1. Can you tell me your sexual orientation? 2. How do you and your wife feel about intimacy? 3. Do you have sex with men, women, or both? 4. Do you have sexual intercourse at your age?

7. Answer: 1. A nonjudgmental attitude facilitates trust and open communication between the nurse and patient. Using a term such as sexual orientation and asking about preferred pronouns allows the patient to identify his or her unique sexuality and sexual health needs.

7. Which is the best intervention the nurse should implement to promote bowel function? 1. Early ambulation 2. Deep-breathing exercises 3. Repositioning on the left side 4. Lowering the head of the patient's bed

7. Answer: 1. Early ambulation promotes peristalsis and thus the return of bowel function. Deep-breathing activities prevent the onset of respiratory complications. Positioning on the left side and lowering the head of the patient's bed do not promote peristalsis.

7. The nurse is caring for a patient with a chest tube for treatment of a right pneumothorax. Which assessment finding necessitates immediate notification of the health care provider? 1. New, vigorous bubbling in the water seal chamber. 2. Scant amount of sanguineous drainage noted on the dressing. 3. Clear but slightly diminished breath sounds on the right side of the chest. 4. Pain score of 2 one hour after the administration of the prescribed analgesic.

7. Answer: 1. The bubbling in the water seal chamber can mean a new pneumothorax or tube dislodgment. The drainage could be related to the insertion procedure and is scant, so it does not require an immediate phone call to the provider. Answers 3 and 4 are expected findings for a patient with a chest tube

7. Resolution of an ethical problem involves discussion with the patient, the patient's family, and participants from appropriate health care disciplines. Which statement best describes the role of the nurse in the resolution of ethical problems? 1. To articulate the nurse's unique point of view, including knowledge based on clinical and psychosocial observations 2. To study the literature on current research about the possible clinical interventions available for the patient in question 3. To hold a point of view but realize that respect for the authority of administrators and physicians takes precedence over personal views 4. To allow the patient and the physician private time to resolve the dilemma on the basis of ethical principles

7. Answer: 1. The ideal process for resolving ethical problems engages the perspectives of all involved, and nurses, as members of the health care team, have a valuable and unique point of view to share. Option 2 is a strategy that assists in answering a clinical question but does not address ethics. Options 3 and 4 are incorrect because both suggest that the nurse disengage from her own values and critical thinking and follow an action driven by the points of views of others.

7. Which action can a nurse delegate to assistive personnel (AP)? 1. Performing glucose monitoring every 6 hours on a patient 2. Teaching the client about the need for enteral feeding 3. Administering enteral feeding bolus after tube placement has been verified 4. Evaluating the client's tolerance of the enteral feeding

7. Answer: 1. The skills of measuring blood glucose level after skin puncture (capillary puncture) can be delegated to AP. The nurse needs to administer enteral feeding because of the risk of aspiration. The nurse is responsible for teaching the client and evaluating the tolerance to the enteral feeding.

7. A nurse on a hospital unit is preparing to hand off care of a patient being discharged to a home health nurse. Match the activities on the left with the hand-off report categories on the right. Activities Categories 1. Use a standard checklist for the report. 2. Encourage questions and clarification. 3. Offer specific information on how to reduce patient's risks. 4. Give report at time when shift has ended and other nurses are requesting information. 5. Explain how patient's discharge was delayed by insufficient numbers of staff. 6. Organize time by preparing in advance what to report.

7. Answer: 1A, 2A, 3A, 4B, 5B, 6A.

7. A patient's cultural background affects the motivation for learning. Using the ACCESS model, match the nursing approach with the correct model component. ACCESS model component Nursing approach 1. Assessment 2. Communication 3. Cultural 4. Establishment 5. Sensitivity 6. Safety A. Help patients feel culturally secure and able to maintain their cultural identity. B. Remain aware of verbal and nonverbal responses. C. Be aware of how patients from diverse backgrounds perceive their care needs. D. Become aware of your patient's culture and your own cultural biases. E. Learn about the patient's health beliefs and practices. F. Show respect by creating a caring rapport.

7. Answer: 1E, 2B, 3D, 4F, 5C, 6A

7. The nurse is contacting the health care provider about a patient's sleep problem. Place the steps of the SBAR (situation, background, assessment, recommendation) in the correct order. 1. Mrs. Dodd, 46 years old, was admitted 3 days ago following a motor vehicle accident. She is in balanced skeletal traction for a fractured left femur. She is having difficulty falling asleep. 2. "Dr. Smithson, this is Pam, the nurse caring for Mrs. Dodd. I'm calling because Mrs. Dodd is having difficulty sleeping." 3. "I'm calling to ask if you would order a hypnotic such as zolpidem to use on a prn basis." 4. Mrs. Dodd is taking her pain medication every 4 hours as ordered and rates her pain as 2 out of 10. Last night she was still awake at 0100. She states that she is comfortable but just can't fall asleep. Her vital signs are BP 124/76, P 78, R 12 and T 37.1°C (98.8°F)

7. Answer: 2, 1, 4, 3. SBAR is Situation, Background, Assessment, and Recommendation. This is the correct sequence of steps in SBAR for the patient and sleep problem

7. A nurse is preparing to teach an older adult who has chronic arthritis how to practice meditation. Which of the following strategies are appropriate? (Select all that apply.) 1. Encourage family members to participate in the exercise. 2. Have patient identify a quiet room in the home that has minimal interruptions. 3. Suggest the use of a quiet fan running in the room. 4. Explain that it is best to meditate about 5 minutes 4 times a day. 5. Show the patient how to sit comfortably with the limitation of his arthritis and focus on a prayer

7. Answer: 2, 3, 5. A quiet room with no distractions is conducive to meditation. The low buzz of a fan also blocks distractive noises. A patient should relax comfortably during meditation. Meditation is usually recommended 10 to 20 minutes twice a day. The activity should be conducted alone without distraction.

7. The school nurse has been following a 9-year-old student who has shown behavioral problems in class. The student acts out and does not follow teacher instructions. The nurse plans to meet with the student's family to learn more about social determinants of health that might be affecting the student. Which of the following factors would be appropriate for this type of assessment? (Select all that apply.) 1. The student's seating placement in the classroom 2. The level of support parents offer when the student completes homework 3. The level of violence in the family's neighborhood 4. The age at which the child first began having behavioral problems 5. The cultural values about education held by family

7. Answer: 2, 3, 5. Social determinants include social support, exposure to crime and violence, and culture. The nurse should learn the child's age at which behavioral problems appeared, but this is not a social determinant. Seating placement is not a social determinant but could b

7. A nurse is administering a metered-dose inhaler (MDI) with a spacer to a patient with chronic obstructive pulmonary disease. Place the steps of the procedure in the correct order. 1. Insert MDI into end of spacer. 2. Perform a respiratory assessment. 3. Remove mouthpiece from MDI and spacer device. 4. Place the spacer mouthpiece into patient's mouth, and instruct patient to close lips around the mouthpiece. 5. Depress medication canister, spraying 1 puff into spacer device. 6. Shake inhaler for 2-5 seconds. 7. Instruct patient to hold breath for 10 seconds. 8. Instruct patient to breathe in slowly through mouth for 3 to 5 seconds.

7. Answer: 2, 3, 6, 1, 4, 5, 8, 7. Obtains baseline respiratory assessment before medication. Ensures optimal delivery of medication using a metered-dose inhaler.

7. The nurse enters a patient's room and finds that the patient was incontinent of liquid stool. Because the patient has recurrent redness in the perineal area, the nurse worries about the risk of the patient developing a pressure injury. The nurse cleanses the patient, inspects the skin, and applies a skin barrier ointment to the perineal area. The nurse consults the ostomy and wound care nurse specialist for recommended skin care measures. Which of the following correctly describe the nurse's actions? (Select all that apply.) 1. The application of the skin barrier is a dependent care measure. 2. The call to the ostomy and wound care specialist is an indirect care measure. 3. The cleansing of the skin is a direct care measure. 4. The application of the skin barrier is an instrumental activity of daily living. 5. Inspecting the skin is a direct care activity

7. Answer: 2, 3. Application of the skin barrier is a direct, independent care measure. It is not an instrumental activity of daily living. Inspecting the skin is an assessment step. The other two interventions are described correctly.

7. A patient has an order for application of compression stockings. Place the following steps for application of the stockings in the correct order: 1. Place patient's toes into foot of stocking up to the heel; keep smooth. 2. Use tape measure to measure patient's leg for proper stocking size. 3. Slide stocking up over patient's calf until sock is completely extended. 4. Turn elastic stocking inside out, keeping hand inside holding heel. Take other hand and pull stocking inside out until reaching the heel. 5. Slide remaining portion of stocking over patient's foot, covering toes. Be sure foot fits into toe and heel of stocking

7. Answer: 2, 4, 1, 5, 3 is the correct sequence

7. Which of the following is an indication for a binder to be placed around a surgical patient with a new abdominal wound? (Select all that apply.) 1. Collection of wound drainage 2. Providing support to abdominal tissues when coughing or walking 3. Reduction of abdominal swelling 4. Reduction of stress on the abdominal incision 5. Stimulation of peristalsis (return of bowel function) from direct pressure

7. Answer: 2, 4. A binder placed over the abdomen can provide protection to the abdominal incision by offering support and decreasing stress from coughing and movement.

7. According to Piaget's cognitive theory, a 12-year-old child is most likely to engage in which of the following activities? (Select all that apply.) 1. Using building blocks to determine how houses are constructed 2. Writing a story about a clown who wants to leave the circus 3. Drawing pictures of a family using stick figures 4. Writing an essay about patriotism 5. Hanging out with a best friend

7. Answer: 2, 5. As adolescents mature, their thinking moves to abstract and theoretical subjects. They have the capacity to reason with respect to possibilities. They typically have a close friend and enjoy spending time with friends

7. During admission of an obese patient with heart failure the assistive personnel (AP) reports to the nurse that the blood pressure (BP) is 140/76 on the left arm and 128/72 on the right arm. What actions do you take on the basis of this information? (Select all that apply.) 1. Notify the health care provider immediately. 2. Repeat the measurements on both arms using a stethoscope. 3. Ask the patient if she has taken her blood pressure medications recently. 4. Obtain blood pressure measurements on lower extremities. 5. Verify that the correct cuff size was used during the measurements. 6. Review the patient's record for her baseline vital signs. 7. Compare right and left radial pulses for strength

7. Answer: 2, 6. The systolic BP measurements are significantly different and may reflect some vascular abnormalities. However, unexpected findings require reassessment by the nurse with a comparison to previous values. It is premature to notify the provider without further assessment. The differences are not caused by medications. An inappropriate cuff size would reflect similar systolic pressures; pulse strength would be similar for these BP measurements

7. You are caring for a 4-year-old child who is hospitalized for an infection. He tells you that he is sick because he was "bad." Which is the most correct interpretation of his comment? 1. Indicative of maladaptive stress response 2. Representative of his cognitive development 3. Suggestive of excessive discipline at home 4. Indicative of his developing sense of inferiority

7. Answer: 2. If two events are related in time or space, preschool children link them in a causal fashion. For example, the hospitalized child reasons, "I cried last night, and that's why the nurse gave me the shot."

7. A nurse is assessing an older adult brought to the emergency department following a fall and wrist fracture. The patient is very thin and unkempt, has a stage 3 pressure injury on her coccyx, and has old bruising to the extremities in addition to her new bruises from the fall. She defers all of the questions to her caregiver son, who accompanied her to the hospital. What is the nurse's next step? 1. Call social services to begin nursing home placement. 2. Ask the son to step out of the room so that she can complete her assessment. 3. Call adult protective services because you suspect elder mistreatment.

7. Answer: 2. The assessment leads you to suspect elder mistreatment, but the nurse needs more information directly from the patient before calling social services or the adult protective services or telling the patient she cannot return home. The nurse will best get this information by asking the son to leave so that she can privately ask the patient direct questions. If the son refuses to leave, this will be another indication that elder mistreatment may be occurring. Cognitive testing will be important but is not the priority

7. A patient with a 3-day history of a stroke that left her confused and unable to communicate returns from interventional radiology following placement of a gastrostomy tube. The patient had been taking hydrocodone/APAP 5/325 up to four tablets/day before her stroke for the past year to manage her arthritic pain. The health care provider's order reads as follows: "Hydrocodone/ APAP 5/325 1 tab, per gastrostomy tube, q4h, prn." Which action by the nurse is most appropriate? 1. No action is required by the nurse because the order is appropriate. 2. Request to have the order changed to around the clock (ATC) for the first 48 hours. 3. Ask for a change of medication to meperidine (Demerol) 50 mg IVP, q3h, prn. 4. Begin the hydrocodone/APAP when the patient shows nonverbal symptoms of pain.

7. Answer: 2. The patient can be expected to have acute pain related to the G-tube insertion; in addition, she has a history of chronic pain. Her pain should be treated with ATC medication to match the timing of her pain

7. Using the Transtheoretical Model of Change, order the steps that a patient goes through to make a lifestyle change related to physical activity. 1. The individual recognizes that he is out of shape when his daughter asks him to walk with her after school. 2. Eight months after beginning walking, the individual participates with his wife in a local 5K race. 3. The individual becomes angry when the physician tells him that he needs to increase his activity to lose 30 lb. 4. The individual walks 2 to 3 miles, 5 nights a week, with his wife. 5. The individual visits the local running store to purchase walking shoes and obtain advice on a walking plan

7. Answer: 3, 1, 5, 4, 2. This sequence follows the order of the steps of transtheoretical model of change: precontemplation, contemplation, preparation, action, and maintenance.

7. A nurse has been caring for a patient with a chronic wound that has not been healing. The nurse talks with a nurse specialist in wound care to find alternative approaches from what the health care provider ordered for dressing the wound. The two decide that because of the patient's allergy to tape a nonallergenic dressing will be used. The nurse obtains an order from the health care provider for the new dressing. After two days there is improvement in the wound. This is an example of which critical thinking standards? (Select all that apply.) 1. Clear 2. Broad 3. Relevant 4. Risk taking 5. Creativity

7. Answer: 3, 4, 5. In this situation the nurse is seeking a different approach and being creative in pursuing the wound care specialist and asking for wound care options. The two nurses show relevance in selecting an option crucial to the patient's unique situation (an allergy). The nurse takes a risk by considering an option different from what the health care provider initially ordered

7. A nurse is performing an assessment on a patient admitted to the unit following treatment in the emergency department for severe bilateral eye trauma. During patient admission the nurse's priority interventions include which of the following? (Select all that apply.) 1. Conducting a home-safety assessment and identifying hazards in the patient's living environment 2. Reinforcing eye safety at work and in activities that place the patient at risk for eye injury 3. Placing necessary objects such as the nurse call system and water in front of the patient to prevent falls caused by reaching 4. Orienting the patient to the environment to reduce anxiety and prevent further injury to the eye 5. Alerting other nurses and health care providers about patient's visual status during hand-off reports

7. Answer: 3, 4, 5. Safety is a top concern when setting priorities for patients who experience eye trauma. Patients with eye trauma may experience serious visual impairments. The patient needs to be oriented to the environment and necessary objects placed in front of him or her to reduce anxiety and prevent further injury. Communicate about the patient's visual impairment to other staff who will be caring for the patient

7. Place the following steps of the assessment process in the correct order. 1. Compare data with another source to determine data accuracy. 2. As a pattern forms, probe and frame further questions. 3. Interview a patient, observe behavior, and gather physical assessment findings. 4. Cluster cues that relate together, make inferences, and identify emerging patterns. 5. Differentiate important data from the total data you collect

7. Answer: 3, 5, 4, 2, 1. The assessment process begins with thorough and appropriate data collection, gained through patient interview, observation, and physical examination. Once all data are collected, you then differentiate important data from the total data you collect; emerging signs of a problem are important to focus on compared with normal findings for a body function. Cluster the cues that relate together and begin to identify a pattern for a problem area. Once a pattern forms, probe further with the interview or observations. Finally, during validation compare data with other sources to determine accuracy

7. Meditation may intensify the effects of which of these medications? (Select all that apply.) 1. Steroid medications 2. Insulin 3. Thyroid-regulating medications 4. Cough syrups 5. Antihypertensive medications

7. Answer: 3, 5. Mind-body techniques, including meditation, create physiological responses in the cardiovascular and respiratory systems. These responses may include decreased blood pressure, reduced heart rate, and slowed respirations. They decrease the need for antihypertensive and other cardiac regulators and thyroid-regulating medications.

7. Which example demonstrates a nurse performing the skill of evaluation? 1. The nurse explains the side effects of the new blood pressure medication ordered for the patient. 2. The nurse asks a patient to rate pain on a scale of 0 to 10 before administering a pain medication. 3. After completing a teaching session, the nurse observes a patient drawing up and administering an insulin injection. 4. The nurse changes a patient's leg ulcer dressing using aseptic technique.

7. Answer: 3. Evaluation is one of the most important aspects of clinical care coordination, involving the determination of patient outcomes. Observing a patient do a return demonstration of teaching is evaluation to ensure that patient has understood teaching. Option 2 is not evaluation because it occurs before administering a pain medication. The other options are interventions

7. A nurse is teaching a patient to obtain a specimen for fecal occult blood testing using fecal immunochemical testing (FIT) at home. How does the nurse instruct the patient to collect the specimen? 1. Get three fecal smears from one bowel movement. 2. Obtain one fecal smear from an early-morning bowel movement. 3. Collect one fecal smear from three separate bowel movements. 4. Get three fecal smears when you see blood in your bowel movement.

7. Answer: 3. Samples from three separate bowel movements decrease the risk of a false-negative or a false-positive result

7. When assessing an older adult who is showing symptoms of anxiety, insomnia, anorexia, and mild confusion, what is the first assessment the nurse conducts? 1. The amount of family support 2. A 3-day diet recall 3. A thorough physical assessment 4. Threats to safety in her home

7. Answer: 3. Stress often causes symptoms similar to physical illnesses. Physical causes for problems need to be investigated and treated before treatment for stress-related symptoms can be initiated.

8. A nurse is participating in a health and wellness event at the local community center. A woman approaches and relates that she is worried that her widowed father is becoming more functionally impaired and may need to move in with her. The nurse asks about his ability to complete activities of daily living (ADLs). ADLs include independence with: (Select all that apply.) 1. Driving. 2. Toileting. 3. Bathing. 4. Daily exercise. 5. Eating.

8. Answer: 2, 3, 5. Activities of daily living are self-care tasks that measure function and are markers for the ability to live independently. Although driving and daily exercise are important to quality of life and health maintenance, they would not necessarily impact a person's ability to live independently.

7. A patient tells the nurse, "My doctor told me to lose weight, exercise, stop smoking, and eat better. I am not sick at all. Why would he tell me that?" The nurse's best response would be: 1. "Since I was not there to hear the conversation, I am not exactly sure." 2. "All of these things are behaviors that are good for you and your family. Why not just give it a try?" 3. "I believe he is trying to get you to think about ways that you can be healthier. All these things help to prevent future health problems." 4. "Eating a balanced, healthy diet and exercising regularly will help you lose weight. I know stopping smoking is really hard to do."

7. Answer: 3. This response is based on therapeutic communication and provides basic rationale for recommendations improving general health and preventing chronic disease. It is appropriate for a nurse to provide clarification in this situation

7. While planning morning care, which of the following patients would have the highest priority to receive his or her bath first? 1. A patient who just returned to the nursing unit from a diagnostic test 2. A patient who prefers a bath in the evening when his wife visits and can help him 3. A patient who is experiencing frequent incontinent diarrheal stools and urine 4. A patient who has been awake all night because of pain 8/10

7. Answer: 3. Urine and fecal material contain substances that can injure a patient's skin and increase the risk for pressure injury and skin damage. Prompt and frequent perineal hygiene is a priority in incontinent patients.

7. A nurse interviews and conducts a physical examination of a patient that includes the following findings: reduced movement of lower leg, reduced range of motion in left knee, and difficulty turning in bed without assistance. This data set is an example of: 1. Collaborative data set. 2. Diagnostic label. 3. Related factors. 4. Data cluster.

7. Answer: 4. A data cluster is the data elements about a patient that form a meaningful pattern, in this case a pattern of impaired mobility.

7. An example of a nurse caring behavior that families of acutely ill patients perceive as important to patients' well-being is: 1. Making health care decisions for patients. 2. Having family members provide a patient's total personal hygiene. 3. Injecting the nurse's perceptions about the level of care provided. 4. Asking permission before performing a procedure on a patient

7. Answer: 4. Asking permission demonstrates to the patient and family that the nurse respects the patient's rights. Respecting and protecting patient rights is part of "doing for" and "being with" the patient

7. The home health nurse is visiting a 90-year-old man who lives with his 89-year-old wife. He is legally blind and is 3 weeks' post right hip replacement. He ambulates with difficulty with a walker. He comments that he is saddened now that his wife has to do more for him and he is doing less for her. Which of the following is the priority nursing diagnosis? 1. Impaired Self Toileting 2. Lack of Knowledge Regarding Resources for the Visually Impaired 3. Disturbed Body Image 4. Risk for Situational Low Self-Estee

7. Answer: 4. Blindness coupled with difficulty ambulating places him at risk for situational low self-esteem. He and his wife most likely have adapted to the blindness, but his difficulty with ambulation affects many aspects of his life, including self-esteem. However, this low self-esteem is situational; as his mobility improves, his low self-esteem will also resolve. Nothing in the question itself suggests that the other diagnoses are true.

7. A mother is concerned about her child's flulike symptoms. You learn from the health assessment that the mother practices use of "hot" and "cold" foods to treat ailments. Which of the following foods do you expect the mother to use to treat her child? 1. Chicken 2. Yogurt 3. Fresh fruits 4. Eggs

7. Answer: 4. Certain cultures believe in the importance of balance and harmony in health. Natural or holistic balance is believed to be achieved by using "hot" and "cold" foods as remedies to treat illness

7. A 7-year-old child was recently diagnosed with asthma. A nurse is providing education to the child and her parents about the treatment and management of asthma and changes they need to make in their home environment to promote her health. Which statement made by the parents requires follow-up by the nurse? 1. "We have made an appointment to talk with the school nurse about the change in our child's health." 2. "We forgot to give our daughter her medications before bedtime, so we made a list of her medications to help us remember." 3. "We have worked out a schedule to check on her before and after school." 4. "We have not been spending time with our parents because we are so busy taking care of our daughter."

7. Answer: 4. Changes in family relationships indicate possible family caregiver stress.

7. The nurses on a medical unit have seen an increase in the number of pressure injuries developing in their patients. The nurses decide to initiate a performance improvement project using the PDSA model. Which of the following is an example of "Plan" from that model? 1. Orienting patients to the unit's practice of hourly rounding on patients 2. Reviewing the incidence of pressure injuries on patients cared for using the protocol 3. Based on findings from patients who developed injuries, implementing an evidence-based skin care protocol on all units 4. Meeting with all disciplines to develop a multidisciplinary approach for reducing pressure injuries

7. Answer: 4. Meeting with all disciplines to develop a multidisciplinary approach for reducing pressure injuries is developing a plan to test the change (Plan).The implementation of the new protocol is the "Do," or carrying out the test. Reviewing the incidence of pressure injuries on patients cared for using the protocol is observing and learning the outcomes (study). Orienting the patients on the unit to the practice of hourly rounding is part of the protocol being implemented.

7. A man who is homeless enters the emergency department seeking health care. The health care provider indicates that the patient needs to be transferred to the city hospital for care before assessing the patient. This action is most likely a violation of which of the following laws? 1. Health Insurance Portability and Accountability Act (HIPAA) 2. Americans with Disabilities Act (ADA) 3. Patient Self-Determination Act (PSDA) 4. Emergency Medical Treatment and Active Labor Act (EMTALA)

7. Answer: 4. The EMTALA requires that an emergency situation needs to be established and that the patient needs to be stabilized before a transfer is appropriate.

7. Place the following steps for insertion of an indwelling catheter in a female patient in appropriate order. 1. Insert and advance catheter. 2. Lubricate catheter. 3. Inflate catheter balloon. 4. Cleanse urethral meatus with antiseptic solution. 5. Drape patient with the sterile square and fenestrated drapes. 6. When urine appears, advance another 2.5 to 5 cm. 7. Prepare sterile field and supplies. 8. Gently pull catheter until resistance is felt. 9. Attach drainage tubing.

7. Answer: 5, 7, 2, 4, 1, 6, 3, 8, 9.

7. Place the following steps for discontinuing intravenous (IV) access in the correct order: 1. Perform hand hygiene and apply gloves. 2. Explain procedure to patient. 3. Remove IV site dressing and tape. 4. Use two identifiers to ensure correct patient. 5. Stop the infusion and clamp the tubing. 6. Carefully check the health care provider's order. 7. Clean the site, withdraw the catheter, and apply pressure

7. Answer: 6, 4, 2, 1, 5, 3, 7. A health care provider's order is necessary before discontinuing IV access, unless there is a complication such as infiltration or phlebitis. Identifying the patient and explaining the procedure are performed before hand hygiene and glove application in order to maintain clean gloves. Removing the site dressing before stopping the infusion and then withdrawing the catheter keeps the VAD patent without forming a clot that could embolize during catheter withdrawal.

7. The nurse is writing a narrative progress note. Identify each of the following statements as subjective data (S) or objective data (O): 1. April 24, 2019 (0900) 2. Repositioned patient on left side. 3. Medicated with hydrocodone-acetaminophen 5/325 mg, 2 tablets PO. 4. "The pain in my incision increases every time I try to turn on my right side." 5. S. Eastman, RN 6. Surgical incision right lower quadrant, 3 inches in length, well approximated, sutures intact, no drainage 7. Rates pain 7/10 at location of surgical incision

7. Answer: O: 1, 2, 3, 5, 6, 7. S: 4. Statement 4 is the only example of a subjective statement. All other statements in the list are objective data. Logical order for placement of these statements includes "1, 4, 6, 7, 3, 2, and 5." The date and time of a narrative note are recorded first, followed by information from the patient that informs clinical decisions, followed by assessment data, and interventions made. A narrative note is closed with the nurse's signature (first initial, last name, credentials).

8. A nurse works with a patient using therapeutic communication and the phases of the therapeutic relationship. Place the nurse's statements in order according to these phases. 1. The nurse states, "Let's work on learning injection techniques." 2. The nurse is mindful of his/her own biases and knowledge in working with the patient with B12 deficiency. 3. The nurse summarizes progress made during the nursing relationship. 4. After providing introductions, the nurse defines the scope and purpose of the nurse-patient relationship

8. 2, 4, 1, 3. In the therapeutic relationship the nurse begins by understanding the self (preinteraction), then provides introductions, followed by a working phase and finally termination and summarization

8. A family consisting of a grandparent, two adults, and three schoolage children just immigrated to the United States. They come to a community wellness center to establish health care. Which of the following questions does the nurse ask to assess the family's function? (Select all that apply.) 1. "What does your family do to keep members healthy?" 2. "How does your family usually make decisions?" 3. "What health services are available in your neighborhood?" 4. "Which rituals or celebrations are important for your family?" 5. "Is there a lot of crime in your neighborhood?" 6. "How many parks are there in your community?"

8. Answer: 1, 2, 4. Cultural practices, decision making, and rituals and celebrations describe what is important to a family. This set of assessment data helps you gain insight into how the family functions and what they consider important to their health and family functioning

8. The nurse manager from the surgical unit was awarded the nursing leadership award for practice of transformational leadership. Which of the following are characteristics or traits of transformational leadership displayed by the award winner? (Select all that apply.) 1. The nurse manager regularly rounds on staff to gather input on unit decisions. 2. The nurse manager sends thank-you notes to staff in recognition of a job well done. 3. The nurse manager sends memos to staff about decisions that the manager has made regarding unit policies. 4. The nurse manager has an "innovation idea box" to which staff are encouraged to submit ideas for unit improvements. 5. The nurse manager develops a philosophy of care for the staf

8. Answer: 1, 2, 4. Nurse managers who practice transformational leadership are focused on change and innovation. They motivate and empower their staff with the focus on team development. The manager will spend time on the unit with the staff sharing ideas and listening to staff input. The manager is enthusiastic about opportunities to enhance the team and shows appreciation and recognizes team members for good work. The manager holds the team accountable and provides support for the team members in the stressful health care environment. The manager shares the philosophy of care developed by the nurse executive of the organization.

8. Which statements reflect the difficulty that can occur for agreement on a common definition of the word quality when it comes to quality of life? (Select all that apply.) 1. Community values influence definitions of quality, and they are subject to change over time. 2. Individual experiences influence perceptions of quality in different ways, making consensus difficult. 3. The value of elements such as cognitive skills, ability to perform meaningful work, and relationship to family is difficult to quantify using objective measures. 4. Statistical analysis is difficult to apply when the outcome cannot be quantified. 5. Whether a person has a job is an objective measure, but it does not play a role in understanding quality of life.

8. Answer: 1, 2, and 4. These statements describe why a single definition for the term quality of life is challenging. Options 3 and 5 are true statements, but they do not explain why the definition of quality of life is difficult to agree on, which is what the question asks for.

8. A 45-year-old woman who is obese tells a nurse that she wants to lose weight. Which assessment findings may be contributing factors to the woman's obesity? (Select all that apply.) 1. The woman works in an executive position that is very demanding. 2. The woman says that she has little time to prepare meals at home and eats out at least four nights a week. 3. The woman works out at the corporate gym at 5 am three mornings per week. 4. The woman says that she tries to eat "low-cholesterol" foods to help lose weight. 5. The woman says that she vacations annually to reduce stress

8. Answer: 1, 2. Demanding and stressful work environments can lead to frequent "stress" eating of non-nutritious foods. Frequently eating away from home and eating fast food have been identified as contributing factors to obesity.

8. When is the application of a warm compress to an ankle muscle sprain indicated? (Select all that apply.) 1. To relieve edema 2. To reduce shivering 3. To improve blood flow to an injured part 4. To protect bony prominences from pressure injuries 5. To immobilize area

8. Answer: 1, 3. Warm compresses can improve circulation by dilating blood vessels, and they reduce edema. The moisture of the compress conducts heat

8. A nurse demonstrates caring by helping family members to: (Select all that apply.) 1. Become active participants in care. 2. Remove themselves from personal care. 3. Make health care decisions for the patient. 4. Plan uninterrupted time for family and patient to be together. 5. Discuss their concerns

8. Answer: 1, 4, 5. Caring for the family takes into consideration the context of the patient's illness and the stress that it imposes on all members. Encouraging family members to provide some care and discuss concerns helps the family to feel involved. Last, providing time for the family to just be together without any "care tasks" or interruptions encourages "presence" for the patient and family

8. Two nurses are revising a self-management education program to help patients better manage their asthma. What strategies are most important for them to include in the program? 1. Have patients list the medications they are prescribed to take and describe any problems they are having with their medications. 2. Create a common set of patient goals that the patients will work toward as a group. 3. Look for group leaders who are health care providers that are respected by the community. 4. Provide information on how to balance activities during the day. 5. Ask patients to discuss how other people in their family react to them now that they have asthma

8. Answer: 1, 4, 5. Self-management education (SME) programs provide evidence-based interventions to help patients with chronic diseases manage their illness and its related symptoms. Current evidence shows that patients need to identify and work on their individual goals. Also offering small, highly interactive group workshops that are facilitated by highly trained leaders who have health challenges of their own is very effective; respected health care providers may not have the personal experience with asthma needed to effectively lead group discussions. Providing strategies, such as how to deal with fatigue, isolation, medication management, and effective communication with family, friends, and health care professionals, helps engage patients in making healthy lifestyle changes.

8. A patient has hypokalemia with stable cardiac function. What are the priority nursing interventions? (Select all that apply.) 1. Fall prevention interventions 2. Teaching regarding sodium restriction 3. Encouraging increased fluid intake 4. Monitoring for constipation 5. Explaining how to take daily weights

8. Answer: 1, 4. Hypokalemia causes bilateral skeletal muscle weakness, especially in the quadriceps, which creates a risk for falling. Hypokalemia also causes gastrointestinal smooth muscle weakness, which produces constipation.

8. At a well-child examination, the mother comments that her toddler eats little at mealtime, will sit only briefly at the table, and wants snacks all the time. Which of the following should the nurse recommend? (Select all the apply.) 1. Provide nutritious snacks for a healthy diet. 2. Offer rewards for eating at mealtimes. 3. Avoid snacks so she is hungry at mealtime. 4. Offer finger foods so she can eat as she walks. 5. Explain to her why eating at mealtime is important.

8. Answer: 1, 4. Mealtime has psychosocial and physical significance. If the parents struggle to control toddlers' dietary intake, problem behavior and conflicts can result. Toddlers often develop "food jags," or the desire to eat one food repeatedly. Rather than becoming disturbed by this behavior, encourage parents to offer a variety of nutritious foods at meals and to provide only nutritious snacks between meals. Toddlers are unlikely to sit down at the table for extended meals, so finger foods are recommended.

8. A nurse is working with an older adult who recently moved to an assisted-living center because of declining physical capabilities associated with the normal aging process. Which nursing interventions are directed at promoting self-esteem in this patient? 1. Commending the patient's efforts at completing self-care tasks 2. Assuming that the patient's physical complaints are attention-seeking measures 3. Minimizing time discussing memories and past achievements spent with the patient 4. Limiting decision-making opportunities for the patient to reduce stress

8. Answer: 1. Reinforce efforts to complete tasks, allowing additional time to complete tasks if needed and support efforts directed at independence. This fosters self-esteem and confidence. It is important for the nurse to refrain from assumptions, as in assuming that physical complaints are attention seeking or that limiting decisions will reduce stress. Time should be allocated to review of past accomplishments and memories

8. Which of the following statements best explains therapeutic touch (TT)? 1. Intentionally mobilizes energy to balance, harmonize, and repattern the recipient's biofield 2. Intentionally heals tissue damage or corrects certain disease symptoms 3. Is overwhelmingly effective in many conditions 4. Is completely safe and does not warrant any special precautions

8. Answer: 1. TT is focused on healing the whole person and providing energy to the body that supports innate healing responses. The research literature is questionable; systematic analyses claim that the research designs are too weak for any conclusive evidence to be identified with confidence. Although TT is relatively safe and there have been very few negative events associated with its use, all therapies (complementary or conventional) should be used with caution in certain populations.

8. Which explanation provided by the nurse is the most accurate meaning for "providing culturally congruent care"? 1. It fits the patient's valued life patterns and set of meanings. 2. It is the same set of values as those of the health care team member providing daily care. 3. It holds one's own way of life as superior to those of others. 4. It redirects the patient to a more socially expected set of values

8. Answer: 1. The ongoing process of cultural competence refers to the need of the health care professional to be culturally sensitive in providing culturally appropriate care specific to the needs of the patient, family, and community

8. A nurse is caring for a patient who recently lost a leg in a motor vehicle accident. The nurse best assists the patient to cope with this situation by applying which of the following theories? 1. Roy 2. Watson 3. Johnson 4. Benner

8. Answer: 1. When applying Roy's adaptation model, the nurse helps the patient cope with/adapt to changes in physiological, self-concept, role function, and interdependence domains.

8. A nurse is assigned to care for a woman who is expecting her first child. The nurse organizes herself and plans to gather data about the patient by applying Pender's health promotion model, including the patient's characteristics and experiences and situational influences. She plans to observe patient behavior and consider the patient's psychosocial issues. Such data will offer a clear understanding to help the nurse identify the patient's needs. This is an example of which of the following concepts? (Select all that apply.) 1. Diagnostic reasoning 2. Deductive reasoning 3. Inductive reasoning 4. Assessment 5. Problem solving

8. Answer: 2 and 4. Assessment involves data gathering to improve knowledge about patients and to recognize patterns of data that reveal problems. Deductive reasoning moves from the general to the specific. A nurse will start analysis of the facts and observations about a patient from a conceptual viewpoint, such as Nola Pender's health promotion model. Then the nurse forms an inference and eventually interprets the patient's condition with respect to the conceptual view

8. A nurse reviews data gathered regarding a patient's response to a diagnosis of cancer. The nurse notes that the patient is restless, avoids eye contact, has increased blood pressure, and expresses a sense of helplessness. The nurse compares the pattern of assessment findings for Anxiety with those of Fear and selects Anxiety as the correct diagnosis. This is an example of the nurse avoiding an error in? (Select all that apply.) 1. Data collection 2. Data clustering 3. Data interpretation 4. Making a diagnostic statement 5. Goal setting

8. Answer: 2, 3 and 4. Data interpretation involves analyzing assessment findings and then involves placing a label on a data pattern or cluster to clearly identify the patient's response to health problems. Critical thinking is necessary. The interpretation of data clusters or patterns leads to the selection of various nursing diagnoses that may apply to a patient. It is important to compare the data in a cluster with the data standards for a diagnosis to come to a reasoned conclusion in making an accurate nursing diagnosis. In this case the nurse did not gather additional data, so avoidance of an error in data collection is not a correct response. Goal setting is not part of the nursing diagnosis process.

8. Which of the following are symptoms of secondary traumatic stress and burnout that commonly affect nurses? (Select all that apply.) 1. Regular participation in a book club 2. Lack of interest in exercise 3. Difficulty falling asleep 4. Lack of desire to go to work 5. Anxiety while working

8. Answer: 2, 3, 4, 5. Nurses are particularly susceptible to the development of secondary traumatic stress and burnout—the components of compassion fatigue. Symptoms include decline in health, emotional exhaustion, irritability, restlessness, impaired ability to focus and engage with patients, feelings of hopelessness, inability to take pleasure from activities, and anxiety.

8. A patient diagnosed with colon cancer has been receiving chemotherapy for 6 weeks. The patient visits the outpatient infusion center twice a week for infusions. The nurse assigned to the patient is having difficulty accessing the patient's intravenous (IV) port used to administer the chemotherapy. Despite attempts to flush the port, it is obstructed. This also occurred 2 weeks earlier. What steps should the nurse follow to make a consultation with a member of the IV infusion team? (Select all that apply.) 1. Ask the IV nurse to come to the infusion center at a time when the nurse starts care for a second patient. 2. Specifically identify the problem of port obstruction, and attempt to flush the port to resolve the problem. 3. Explain to the IV nurse the frequency in which this port has obstructed in the past. 4. Tell the IV nurse the problem is probably related to the physician who inserted the port. 5. Describe to the IV nurse the type and condition of the port currently in use

8. Answer: 2, 3, 5. When making a consult, identify the general problem area (obstructed port). Provide a consultant with relevant information about the problem area (type and condition of port). Provide a summary of the problem, the methods used to resolve the problem so far, and outcomes of these methods (port flushing, port remains obstructed). Do not prejudice or influence consultants (physician to blame). Be available to discuss a consultant's findings and recommendations. The consultant is not there to take over the problem but to help you resolve it.

8. Which nursing interventions should a nurse implement when removing an indwelling urinary catheter in an adult patient? (Select all that apply.) 1. Attach a 3-mL syringe to the inflation port. 2. Allow the balloon to drain into the syringe by gravity. 3. Initiate a voiding record/bladder diary. 4. Pull the catheter quickly. 5. Clamp the catheter before removal

8. Answer: 2, 3. By allowing the balloon to drain by gravity, it is possible to avoid the development of creases or ridges in the balloon and thus minimize trauma to the urethra during withdrawal. All patients who have a catheter removed should have their voiding monitored. The best way to do this is with a voiding record or bladder diary. The size syringe used to deflate the balloon is dictated by the size of the balloon. In the adult patient balloon sizes are either 10 mL or 30 mL. Catheters should be pulled out slowly and smoothly. There is no evidence to support clamping catheters before removal.

8. A patient is diagnosed with a multidrug-resistant organism (MDRO) in his surgical wound and asks the nurse what this means. What is the nurse's best response? (Select all that apply.) 1. There is more than one organism in the wound that is causing the infection. 2. The antibiotics the patient has received are not strong enough to kill the organism. 3. The patient will need more than one type of antibiotic to kill the organism. 4. The organism has developed a resistance to one or more broad-spectrum antibiotics, indicating that the organism will be hard to treat effectively. 5. There are no longer any antibiotic options available to treat the patient's infection.

8. Answer: 2, 4 An MDRO is a single organism that is resistant to one or more classes of antibiotics, which makes it harder to treat, but there is treatment available.

8. The nurse is using the QSEN competency of EBP when working with the unit council to initiate a change related to pain management. Which behaviors demonstrate the nurse practicing behaviors associated with EBP? (Select all that apply.) 1. Initiating plan for self-development as a team member 2. Reading original research related to pain management 3. Demonstrating effective use of strategies to reduce risk of harm to self or others 4. Valuing EBP as critical to the development of pain management guidelines for the unit 5. Describing to the unit council reliable sources for locating clinical guidelines 6. Applying technology and information management tools to support safe processes of care

8. Answer: 2, 4, 5. "Reading original research related to pain management" is an EBP behavior as the nurse searches for and locates the best evidence related to pain management. "Valuing EBP as critical to the development of pain management guidelines for the unit" is a behavior that shows the nurse recognizes that the best evidence should be used as the foundation for clinical practice. "Describing to the unit council reliable sources for locating clinical guidelines" is demonstration of a behavior in which the nurse shares knowledge on the clinical unit on locating evidence-based clinical guidelines.

8. An older adult is admitted from a skilled nursing home to a medical unit with pneumonia. A review of the medical record reveals that he had a stroke affecting the right hemisphere of the brain 6 months ago and was placed in the skilled nursing home because he was unable to care for himself. Which of these assessment findings does the nurse expect to find? (Select all that apply.) 1. Slow, cautious behavioral style 2. Inattention and neglect, especially to the left side 3. Cloudy or opaque areas in part of the lens or the entire lens 4. Visual spatial alterations such as loss of half of a visual field 5. Loss of sensation and motor function on the right side of the body

8. Answer: 2, 4. A stroke in the right hemisphere produces symptoms on the left side, which may include visual spatial alterations (such as loss of half of a visual field) or inattention and neglect, especially to the left side, and a quick, inquisitive behavioral style. A stroke affecting the right hemisphere of the brain may result in symptoms such as loss of sensation and motor function on the left side of the body. Cloudy or opaque areas in part of the lens or the entire lens indicate cataracts.

8. The nurse reviews the health history of a 48-year-old man and notes that he was started on medications for elevated blood pressure and depression at his last annual physical. He tells the nurse that over the past 6 months he is having difficulty sustaining an erection. The nurse understands that: (Select all that apply.) 1. Nurses are not expected to discuss sexual issues with male patients and the physician should address this. 2. Sexual function can be affected by some medications. 3. Sexually transmitted infections (STIs) can cause complications such as erectile dysfunction and screening should be done. 4. Some men with health issues experience erectile dysfunction. 5. Medications used to treat hypertension and depression seldom interfere with sexual function.

8. Answer: 2, 4. Nurses should complete a holistic assessment on all patients to be able to personalize a plan of care. Nurses who are uncomfortable discussing sexual concerns of patients should seek out training and resources to develop this skill. Many drugs and illnesses can affect sexual function. Antidepressants can alter sexual functioning by blocking neurotransmitters. Antihypertensives can affect sexual function by altering circulation. Erectile dysfunction occurs more frequently in older men but can occur in men as young as 40. STIs may affect sexual functioning but are less likely than medications or illness to be the cause of erectile dysfunction

8. A nurse is assigned to care for an 82-year-old patient who will be transferred from the hospital to a rehabilitation center. The patient and her husband have selected the rehabilitation center closest to their home. The nurse learns that the patient will be discharged in 3 days and decides to make the referral on the day of discharge. The nurse reviews the recommendations for physical therapy and applies the information to fall prevention strategies in the hospital. What discharge planning action by the nurse has not been addressed correctly? 1. Patient and family involvement in referral 2. Timing of referral 3. Incorporation of referral discipline recommendations into plan of care 4. Determination of discharge date

8. Answer: 2. The nurse must make the referral as soon as possible. The other elements of discharge planning, including knowing the discharge date, involving the patient and family in decision-making, and incorporating the referral discipline's recommendations for the patient's care are part of discharge planning

8. Which statement made by a mother being discharged to home with her newborn infant indicates that she understands the discharge teaching related to best sleep practices? 1. "I'll give the baby a bottle to help her fall asleep." 2. "We'll place the baby on her back to sleep." 3. "We put the baby's stuffed animals in the crib to make her feel safe." 4. "I know the baby will not need to be fed until morning."

8. Answer: 2. This is based on the current evidence that shows that parents need to place an infant on his or her back to prevent suffocation. Bottles, stuffed animals, and pillows should not be placed in the bed with an infant.

8. A patient is to receive medications through a small-bore nasogastric feeding. Which nursing actions are appropriate? (Select all that apply.) 1. Verifying tube placement after medications are given 2. Mixing all medications together to give all at once 3. Using an enteral tube syringe to administer medications 4. Flushing tube with 30 to 60 mL of water after the last dose of medication 5. Checking for gastric residual before giving the medications 6. Keeping the head of the bed elevated 30 to 60 minutes after the medications are given

8. Answer: 3, 4, 5, 6. An enteral tube syringe is necessary to avoid dangerous misconnections and accidentally administering the medications through another tube. Flushing the tubing after medication administration clears the tubing of any residual medication and ensures that the tube remains patent. If gastric residuals are high, then the absorption of the enteral tube medication is reduced. Elevating the head of the bed helps to reduce the risk for aspiration. Verification of tube placement is essential before administering anything via a nasogastric tube. Medications are given separately to avoid any drug-to-drug interactions, which could clog the feeding tube.

8. A nursing student is developing a plan of care for a 74-year-old-female patient who has spiritual distress over losing a spouse. As the nurse develops appropriate interventions, which characteristics of older adults should be considered? (Select all that apply.) 1. Older adults do not routinely use complementary medicine to cope with illness. 2. Older adults dislike discussing the afterlife and what might have happened to people who have passed on. 3. Older adults achieve spiritual resilience through frequent expressions of gratitude. 4. Have the patient determine whether her husband left a legacy behind. 5. Offer the patient her choice of rituals or participation in exercise.

8. Answer: 3, 4, 5. Older-adult patients achieve spiritual resiliency in expressing gratitude and finding ways to maintain purpose in life. Leaving legacies maintains a connection between the person left behind and the lost loved one. Older adults frequently use complementary medicine, rituals, and exercise to cope with illness and pain. Belief in the afterlife grows with aging

8. The nurse finds a 68-year-old woman wandering in the hallway and exhibiting confusion. The patients says she is looking for the bathroom. Which interventions are appropriate for this patient? (Select all that apply.) 1. Ask the health care provider to order a restraint. 2. Recommend insertion of a urinary catheter. 3. Provide scheduled toileting rounds every 2 to 3 hours. 4. Institute a routine exercise program for the patient. 5. Keep the bed in high position with side rails down. 6. Keep the pathway from the bed to the bathroom clear.

8. Answer: 3, 4, 6. There are no appropriate conditions for this patient to be restrained. A patient who repeatedly wanders may require the temporary use of restraints to keep him or her safe. However, the use of alternatives to restraints is preferred, and if a restraint is required, use the least restrictive. A urinary catheter is not inserted to avoid having a patient use the bathroom. The patient should have a low bed so that if the patient falls, the risk of injury may be lessened.

8. An older-adult patient is admitted following a hip fracture and surgical repair. Before ambulating the patient postoperatively on the evening of surgery, which of the following would be most important to assess? (Select all that apply.) 1. Patient's usual dietary intake 2. Time and date of the patient's last bowel movement 3. Preadmission activity tolerance 4. Baseline heart rate 5. Patient's home living situation

8. Answer: 3, 4. Although all of these questions should be included on the initial assessment for the patient before first ambulation, establishing a baseline activity tolerance and heart rate will help avoid overexerting the patient and help develop expectations (safe target heart rate) that are safe and reasonable as mobility is increased.

8. Which of the following indicates that additional assistance is needed to transfer a patient from the bed to the stretcher? (Select all that apply.) 1. The patient is 5 feet, 6 inches and weighs 120 lb. 2. The patient speaks and understands English. 3. The patient is returning to unit from recovery room after a procedure requiring conscious sedation. 4. The patient has a history of being able to stand independently. 5. The patient received analgesia for pain 30 minutes ago

8. Answer: 3, 5. Although the patient was discharged from the procedure recovery room, the patient did have conscious sedation and is still having effects from an analgesic for pain. These factors will affect whether the patient can safely participate in the transfer; therefore, additional help would be needed to safely transfer the patient from the bed to the stretcher.

8. The nurse is observing as the student nurse performs a respiratory assessment on a patient. Which action by the student nurse requires the nurse to intervene? 1. The student stands at a midline position behind the patient, observing for position of the spine and scapula. 2. The student palpates the thoracic muscles for masses, pulsations, or abnormal movements. 3. The student places the bell of the stethoscope on the anterior chest wall to auscultate breath sounds. 4. The student places the palm of the hand over the intercostal spaces and asks the patient to say "ninety-nine."

8. Answer: 3. Breath sounds should be auscultated using the diaphragm of the stethoscope. Auscultate in a systematic pattern over the posterior and anterior chest wall

8. A nursing student is giving a presentation to a group of other nursing students about the needs of patients with mental illnesses in the community. Which statement by the student indicates that the nursing professor needs to provide further teaching? 1. "Many patients with mental illness do not have a permanent home." 2. "Unemployment is a common problem experienced by people with a mental illness." 3. "The majority of patients with mental illnesses live in longterm care settings." 4. "Patients with mental illnesses are often at a higher risk for abuse and assault."

8. Answer: 3. Many patients with severe mental illnesses are homeless, unemployed, and at a greater risk for abuse and assault. Patients with mental illnesses no longer routinely live in long-term psychiatric institutions but instead receive resources in the community.

8. A home health nurse notices significant bruising on a 2-year-old patient's head, arms, abdomen, and legs. The patient's mother describes the patient's frequent falls. What is the best nursing action for the home health nurse to take? 1. Document her findings and treat the patient. 2. Instruct the mother on safe handling of a 2-year-old child. 3. Contact a child abuse hotline. 4. Discuss this story with a colleague.

8. Answer: 3. Nurses are mandated reporters of suspected child abuse. These assessment findings possibly indicate child abuse.

8. A 63-year-old woman is a family caregiver for her 88-year-old mother who has dementia. The caregiver asked the home health nurse how to manage her mother when she becomes confused and violent. The best instructional method a nurse can use for this situation is: 1. Demonstration 2. Preparatory instruction 3. Role-playing 4. Group instruction with other family caregivers

8. Answer: 3. Role-playing is effective in teaching a person how to respond to another person's behavior. The technique involves rehearsing a desired behavior. Preparatory instruction is for an individual who is about to undergo a procedure that typically causes anxiety. Demonstration is used when psychomotor skills are being taught. Group instruction would be useful as a way for the caregiver to discuss problems confronted during caregiving but is less effective for responding to a specific behavior.

8. The assistive personnel (AP) informs the nurse that the electronic blood pressure machine on the patient who has recently returned from surgery after removal of her gallbladder is flashing a blood pressure of 65/46 and alarming. Place the care activities in priority order. 1. Press the start button of the electronic blood pressure machine to obtain a new reading. 2. Obtain a manual blood pressure with a stethoscope. 3. Check the patient's pulse distal to the blood pressure cuff. 4. Assess the patient's mental status. 5. Remind the patient not to bend her arm with the blood pressure cuff

8. Answer: 4, 1, 3, 2, 5. First priority is to verify that the patient's blood pressure is providing adequate blood flow to the brain and critical organs. Movement interferes with electronic blood pressure measurement; recycling the machine will obtain a blood pressure while you are assessing the patient. Check the distal pulse to verify circulation to the extremity and then obtain manual blood pressure if needed. Patient education can prevent false values and decrease patient anxiety with alarms

8. Elizabeth, who is having unprotected sex with her boyfriend, comments to her friends, "Did you hear about Kathy? You know, she fools around so much; I heard she was pregnant. That would never happen to me!" This is an example of adolescent: 1. Imaginary audience. 2. False-belief syndrome. 3. Personal fable. 4. Sense of invulnerability

8. Answer: 4. Adolescents can be risk takers and believe that they are immune to the negative consequences of behaviors; they are just beginning to be future oriented in their thought process and see everything as black or white.

8. The nurse is discussing the advantages of using computerized provider order entry (CPOE) with a nursing colleague. Which statement best describes the major advantage of a CPOE system within an electronic health record? 1. CPOE reduces the time necessary for health care providers to write orders. 2. CPOE reduces the time needed for nurses to communicate with health care providers. 3. Nurses do not need to acknowledge orders entered by CPOE in an electronic health record. 4. CPOE improves patient safety by reducing transcription errors.

8. Answer: 4. Although the other answers loosely describe some positive aspects of CPOE, option 4 provides the best description of the major advantage CPOE offers—the reduction of transcription errors, which reduces medical errors and creates a safer patient care environment

8. Which statement made by the parents of a 2-month-old infant requires further education by the nurse? 1. "I'll continue to use formula for the baby until he is at least a year old." 2. "I'll make sure that I purchase iron-fortified formula." 3. "I'll start feeding the baby cereal at 4 months." 4. "I'm going to alternate formula with whole milk, starting next month."

8. Answer: 4. Infants should not have regular cow's milk during the first year of life. It is too concentrated for the infant's kidneys to manage. There is also an increased risk for developing milk-product allergies. .

8. A nurse in the recovery room is monitoring a patient who had a left knee replacement. The patient arrived in recovery 15 minutes ago. The nurse observes the patient to be restless, turning frequently, and groaning; the patient's heart rate is 92 compared with 76 preoperatively. Blood pressure is stable since admission to the recovery room. The nurse reviews the medical orders for analgesic therapy. The nurse notes that the postop dose of an ordered analgesic has not yet been given. What is most likely to cause the nurse to reflect on the patient's situation? 1. The patient is recovering normally. 2. The symptoms reflecting restlessness 3. The patient's blood pressure trend 4. The delay in administration of the analgesic

8. Answer: 4. Reflection is a nurse's conscious effort to think about nursing interventions and outcomes. It usually occurs in the presence of a trigger event, which involves a breakdown or perceived breakdown in practice such as the delay or omission of a treatment.

9. Which of these statements are true regarding disinfection and cleaning? (Select all that apply.) 1. Proper cleaning requires mechanical removal of all soil from an object or area. 2. General environmental cleaning is an example of medical asepsis. 3. When cleaning a wound, wipe around the wound edge first and then clean inward toward the center of the wound. 4. Cleaning in a direction from the least to the most contaminated area helps reduce infections. 5. Disinfecting and sterilizing medical devices and equipment involve the same procedures.

9. Answer: 1, 2, 4 Proper cleaning and disinfection are processes that occur prior to sterilization, with cleaning always done from dirty to clean to decrease the risk of further infection and contamination.

8. A 34-year-old single father who is anxious, tearful, and tired from caring for his three young children tells the nurse that he feels depressed and doesn't see how he can go on much longer. Which statement would be the nurse's best response? 1. "Are you thinking of suicide?" 2. "You've been doing a good job raising your children. You can do it!" 3. "Is there someone who can help you during the evenings and weekends?"

8. Answer: 4. You need to get information about what the gentleman means when he says he can't go on any longer. He might be thinking of turning his children over to a grandparent or seeking other child-care arrangements. Asking about suicide initially might be premature. Asking "Are you thinking of suicide?" prematurely might shut the patient down entirely. If the patient talks about suicide, for safety reasons it is very important to further discuss his suicidal thoughts and refer to the appropriate health care professional. Asking the open-ended question provides an opportunity to understand what the person is thinking and open lines of communication.

8. In preparing to collect a nursing history for a patient admitted for elective surgery, which of the following data are part of the review of present illness in the nursing health history? 1. Current medications 2. Patient expectations of planned surgery 3. Review of patient's family support system 4. History of allergies 5. Patient's explanation for what might be the cause of symptoms that require surgery

8. Answer: 5. The nursing health history has several components. A review of present illness or health concerns includes a question asking the patient what provokes or precipitates symptoms. Gathering a patient's explanation for what might be the cause of symptoms is the appropriate approach. Past health history is the component that includes a medication history and history of allergies. Patient expectations of treatment is another component. A review of family interaction and support is part of the family history component.

8. Place the following steps in the correct order for administration of patient-controlled analgesia: 1. Insert drug cartridge into infusion device and prime tubing. 2. Wipe injection port of maintenance IV line vigorously with antiseptic swab for 15 seconds and allow to dry. 3. Demonstrate to patient how to push medication demand button. 4. Secure connection and anchor PCA tubing with tape. 5. Instruct patient to notify a nurse for possible side effects or changes in the severity or location of pain. 6. Insert needleless adapter into injection port nearest patient. 7. Apply clean gloves. Check infuser and patient-control module for accurate labeling or evidence of leaking. 8. Program computerized PCA pump as ordered to deliver prescribed medication dose and lockout interval. 9. Attach needleless adapter to tubing adapter of patient-controlled module

8. Answer: Correct order of steps is as follows: 3, 5, 7, 1, 9, 2, 6, 4, 8. Instruct patient before final pump preparation.

8. The nurse has just witnessed her patient go into cardiac arrest. What priority interventions should the nurse perform at this time? (Select all that apply.) 1. Perform chest compressions. 2. Ask someone to bring the defibrillator to the room for immediate defibrillation. 3. Apply oxygen via nasal cannula. 4. Place the patient in the high Fowler's position. 5. Educate the family about the need for CPR

8. Answers: 1 and 2. Applying oxygen won't help the patient as he or she is not breathing. The patient needs to be supine for compressions to be effective. The family does need to be educated, but this is not the priority for the nurse at this time. The nurse could delegate this task to a member of the health care team who is not actively engaged in the resuscitation.

8. Match the nursing interventions on the left with the complication to be prevented on the right. An intervention may apply to more than one complication. Nursing Intervention Complication 1. Offering glasses or hearing aid a. Deep vein thrombosis 2. Early ambulation b. Wound infection 3. Strict aseptic technique c. Delirium 4. Deep breathing exercise d. Atelectasis 5. Hydration

8. The correct answers are 1. c; 2. a, c; 3. b; 4. d; 5. a, d

9. The nurse delegates needed hygiene care for an alert elderly patient who had a stroke. Which intervention would be appropriate for the assistive personnel to accomplish during the bath? (Select all that apply) 1. Checking distal pulses 2. Providing range-of-motion (ROM) exercises to extremities 3. Determining type of treatment for stage 1 pressure injury 4. Changing the dressing over an intravenous site 5. Providing special skin care

9. Answer 2, 5. Providing ROM exercises may be delegated to assistive personnel. The nursing assistive personal can also give skin care. The other activities should be performed by the nurse.

9. Which measures does a nurse follow when being asked to perform an unfamiliar procedure? (Select all that apply.) 1. Checks scientific literature or policy and procedure 2. Determines whether additional assistance is needed 3. Collects all necessary equipment 4. Delegates the procedure to a more experienced nurse 5. Considers all possible consequences of the procedure

9. Answer: 1, 2, 3, 5. A more experienced nurse can assist, but unless there is an emergency and there is no time to prepare, you would not delegate. Instead you would learn how to perform the procedure. When performing an unfamiliar procedure, check policy and procedure, determine whether assistance is needed, prepare supplies and equipment, and consider all possible consequences of the procedure.

9. Which of the following statements indicate that the new nursing graduate understands ways to remain involved professionally? (Select all that apply.) 1. "I am thinking about joining the health committee at my church." 2. "I need to read newspapers, watch news broadcasts, and search the Internet for information related to health." 3. "I will join nursing committees at the hospital after I have completed orientation and better understand the issues affecting nursing." 4. "Nurses do not have very much voice in legislation in Washington, DC, because of the nursing shortage." 5. "I will go back to school as soon as I finish orientation."

9. Answer: 1, 2, 3. Nurses need to be actively involved in their communities and be aware of current issues in health care. Staying abreast of current news and public opinion through the media is essential. Nurses need to join nursing committees to be involved in decision making. Nurses have a powerful voice in the legislature

9. A nurse is performing a home care assessment on a patient with a hearing impairment. The patient reports, "I think my hearing aid is broken. I can't hear anything." After determining that the patient's hearing aid works and that the patient is having trouble managing the hearing aid at home, which of the following teaching strategies does the nurse implement? (Select all that apply.) 1. Demonstrate hearing aid battery replacement. 2. Review method to check volume on hearing aid. 3. Demonstrate how to wash the earmold and microphone with hot water. 4. Discuss the importance of having wax buildup in the ear canal removed. 5. Recommend a chemical cleaner to remove difficult buildup.

9. Answer: 1, 2, 4. The earmold should be wiped with warm water and a mild soap. The microphone should not be allowed to get wet. Chemical cleaners should not be used. For difficult buildup the brush that was included with the aid should be used to sweep away debris.

9. During a home visit, a patient states, "I am really upset about my heart failure. I can't go out to eat anymore with my friends, I have no energy, and I don't even want to talk on the phone. All I do is focus on how this disease has changed my life and how much time I have left to live." How should the nurse respond? (Select all that apply.) 1. "Let's talk about going out to lunch. What is making you hesitant about eating with your friends?" 2. "Tell me about what types of activities you were doing before you knew you had heart failure." 3. "Eventually you will get used to all of the changes. You are doing OK." 4. "My mother has heart failure, and she has adjusted to diet, activity, and medication changes." 5. "How has your heart failure affected your energy level?"

9. Answer: 1, 2, 5. The patient's responses to these statements and questions will help the nurse better understand how heart failure is affecting the patient's ability to cope with the diagnosis and will guide topics for patient education. Option 3 provides false reassurance that is not supportive, and option 4 changes the focus of conversation away from the patient's concerns.

9. Which statements properly apply an ethical principle to justify access to health care? (Select all that apply.) 1. Access to health care reflects the commitment of society to principles of beneficence and justice. 2. If low income compromises access to care, respect for autonomy is compromised. 3. Access to health care is a privilege in the United States, not a right. 4. Poor access to affordable health care causes harm that is ethically troubling because nonmaleficence is a basic principle of health care ethics. 5. If a new drug is discovered that cures a disease but at great cost per patient, the principle of justice suggests that the drug should be made available to those who can afford it

9. Answer: 1, 2, and 4. Access to health care services can be justified through the application of the principles of justice, beneficence, respect for autonomy, and nonmaleficence. While option 3 is an opinion that can be justified with ethical analysis, no justification is offered in this statement, so this option is not correct. Option 5 again describes a point of view, but no ethical principles are described that support this view. Option 6 is incorrect because justice refers to fairness in the distribution of resources and basing access to medication only on income may not be fair

9. Which of the following are examples of the conventional reasoning form of cognitive development? (Select all that apply.) 1. A 35-year-old woman is speaking with you about her recent diagnosis of a chronic illness. She is concerned about her treatment options in relation to her ability to continue to care for her family. As she considers the options and alternatives, she incorporates information, her values, and emotions to decide which plan will be the best fit for her. 2. A young father is considering whether or not to return to school for a graduate degree. He considers the impact the time commitment may have on the needs of his wife and infant son. 3. A teenage girl is encouraged by her peers to engage in shoplifting. She decides not to join her peers in this activity because she is afraid of getting caught in the act. 4. A single mother of two children is unhappy with her employer. She has been unable to secure alternate employment but decides to quit her current job. 5. A young man drives over the speed limit regularly because he thinks he is an excellent driver and will not get into a car accident

9. Answer: 1, 2. A person using conventional reasoning sees moral reasoning based on his or her own personal internalization of societal and others' expectations. A person wants to fulfill the expectations of the family, group, or nation and also to develop a loyalty to and actively maintain, support, and justify the order

9. Which of the following are common barriers to effective discharge planning? (Select all that apply.) 1. Ineffective communication among providers 2. Lack of role clarity among health care team members 3. Sufficient number of hospital beds to manage patient volume 4. Patients' long-term disabilities 5. The patient's cultural background

9. Answer: 1, 2. Barriers to effective discharge planning include ineffective communication, lack of role clarity among health care team members, and lack of resources. The presence of long-term disability is not a barrier but a characteristic of some patients who need greater discharge planning. A patient's cultural background is not a barrier unless you do not consider cultural factors in planning for discharge.

9. A new nurse graduate is in orientation on a surgical unit and is being mentored by an experienced nurse. Which action completed by the new nurse graduate requires intervention by the experienced nurse? (Select all that apply.) 1. The new nurse stops documenting about a dressing change to take a patient some water. 2. The new nurse gathered the medications for two different patients at the same time. 3. The new nurse asked an AP to help transfer a patient from the bed to a wheelchair before discharge. 4. The new nurse educates a patient about pain management when administering a pain medication to a patient. 5. The new nurse gathers all equipment necessary to start a new IV site before entering a patient's room.

9. Answer: 1, 2. Organizational skills help a nurse deliver care safely and effectively. In answer choice 1, the nurse interrupts documentation to attend to a basic patient need. In this situation, the nurse could have asked an AP to get the water or completed documentation and then gotten the water for the patient. Limiting interruptions is important in preventing errors. In answer choice 2, the nurse gathers medications for two patients at one time. Nurses should prepare medications for only one patient at a time to prevent medication errors

9. The nurse in a new community-based clinic is requested to complete a community assessment. Order the steps for completing this assessment. 1. Structure or locale 2. Social systems 3. Population

9. Answer: 1, 3, 2. To begin a community assessment, the structure and geographic boundaries of the community are identified. Look at the structures in the community (e.g., schools, churches, types of residences). Next, obtain data about the population and the demographics of the community. Who are the residents of the community, what is the age range, and which types of ethnicity are represented? Last, review the social systems in the community

9. The school nurse is counseling an adolescent male who is returning to school after attempting suicide. He denies substance abuse and has no history of treatment for depression. He says he has no friends or family who understand him. Critical thinking encourages the nurse to consider all possibilities, including which of the following? (Select all that apply.) 1. Adolescents often explore their sexual identity and expose themselves to complications such as sexually transmitted infections (STIs) or unplanned pregnancy. 2. Peer approval and acceptance are not important in this agegroup. 3. Lesbian, gay, bisexual, and transgender (LGBTQ+) youth often experience stress from identification with a sexual minority group. 4. Knowledge about normal changes associated with puberty and sexuality can decrease stress and anxiety. 5. Adolescence is a time of emotional stability and self-acceptance.

9. Answer: 1, 3, 4. Adolescents are establishing their identity and exploring their sexual preference. Those who identify with a sexual minority group often experience stress and isolation from peers. They need clear and accurate information about physiological and emotional changes occurring in their body. Peer influence is high during this time, but support from family and health care professionals is equally important to adolescents.

9. A 51-year-old adult comes to a medical clinic for an annual physical exam. The patient is found to be slightly overweight and reports being inactive, walking only 2 to 3 times a week with his wife after work. He has good muscle strength and coordination of lower extremities. Which of the following recommendations from the Physical Activity Guidelines for Americans should the nurse suggest? Choose all that apply 1. Move more and sit less throughout the day. 2. Participate in at least 90 minutes a week of moderate-intensity aerobic physical activity. 3. Perform muscle-strengthening activities using light weights on 2 or more days a week. 4. Walk at a vigorous pace with wife at least 150 minutes over five days a week

9. Answer: 1, 3, 4. Adults are recommended to move more and sit less throughout the day. Some physical activity is better than none, For substantial health benefits, adults should do at least 150 minutes (2 hours and 30 minutes) to 300 minutes (5 hours) a week of moderate-intensity aerobic physical activity (which includes vigorous walking). Adults should also do muscle-strengthening activities of moderate or greater intensity and that involve all major muscle groups on 2 or more days a week. Balance is also important but more of a focus for older adults

9. Which strategies should a nurse use to facilitate a safe transition of care during a patient's transfer from the hospital to a skilled nursing facility? (Select all that apply.) 1. Collaboration between staff members from sending and receiving departments 2. Requiring that the patient visit the facility before a transfer is arranged 3. Using a standardized transfer policy and transfer tool 4. Arranging all patient transfers during the same time each day 5. Relying on family members to share information with the new facility

9. Answer: 1, 3. Providing a standardized process, policy, and tool can assist in a predictable, safe transfer of important patient information between health care facilities. Communication and collaboration between the sender and receiver of information enable the staff to validate that the information was received and understood. Requiring a patient visit is not always necessary, and relying on family members to share information does not release staff from their responsibilities. Doing patient transfers on the same day and time has no effect on creating a safe patient transfer

9. A nurse cares for a postoperative patient in the PACU. Upon assessment, the nurse finds the surgical dressing is saturated with serosanguineous drainage. Which interventions are a priority? (Select all that apply.) 1. Notify surgeon. 2. Maintain the intravenous fluid infusion. 3. Provide 2 L/min of oxygen via nasal cannula. 4. Monitor the patient's vital signs every 5 to 10 minutes. 5. Reinforce the dressing

9. Answer: 1, 5. The first two priorities are for the nurse to report to the surgeon immediately and to reinforce the dressing as needed. Maintaining intravenous fluids and monitoring vital signs are routine aspects of the patient's plan of care. Providing oxygen requires a prescription; the surgeon has to be notified for a prescription for oxygen

9. A healthy adult patient tells the nurse that he obtained his blood pressure in "one of those quick machines in the mall" and was alarmed that it was 152/72 when his normal value ranges from 114/72 to 118/78. The nurse obtains a blood pressure of 116/76. What would account for the blood pressure of 152/92? (Select all that apply.) 1. Cuff too small 2. Arm positioned above heart level 3. Slow inflation of the cuff by the machine 4. Patient did not remove his long-sleeved shirt 5. Insufficient time between measurements

9. Answer: 1, 5. Using too small of a cuff and not allowing for insufficient time between measurements will result in false-high readings. Arm above heart level and slow inflation result in false low readings.

9. A hospice nurse sits at the bedside of a male patient in the final stages of cancer. He and his parents made the decision that he would move home and they would help him in the final stages of his disease. The family participates in his care, but lately the nurse has increased the amount of time she spends with the family. Whenever she enters the room or approaches the patient to give care, she touches his shoulder and tells him that she is present. This is an example of what type of touch? 1. Caring touch 2. Protective touch 3. Task-oriented touch 4. Interpersonal touch

9. Answer: 1. Caring touch is a form of nonverbal communication. You express this in the way that you hold a patient's hand, give a back massage, gently position a patient, or participate in a conversation. When using a caring touch, you connect with the patient physically and emotionally

9. What is the removal of devitalized tissue from a wound called? 1. Debridement 2. Pressure distribution 3. Negative-pressure wound therapy 4. Sanitization

9. Answer: 1. Debridement is the removal of nonliving tissue, cleaning the wound to move toward healing

9. A nurse is helping a patient perform active assisted range of motion in the right elbow. Which statement describes the correct technique? 1. Support elbow by holding distal part of extremity. 2. Grasp joint with fingers to provide support. 3. Have patient move joint independently. 4. Move the joint past the point of resistance. 5. Perform the exercise a few times only, and gradually build up to more.

9. Answer: 1. During a ROM exercise, supporting the distal part of the extremity is the correct technique. Use a cupped hand and not the fingers to support a joint. Active assisted ROM requires assistance of a care provider. Never move a joint past the point of resistance. Each joint movement needs to be repeated 5 times during a session, with sessions performed 2 to 3 times a day

9. A nurse is conducting a patient-centered interview. Place the statements from the interview in the correct order, beginning with the first statement a nurse would ask. 1. "You say you've lost weight. Tell me how much weight you've lost in the past month." 2. "My name is Terry. I'll be the nurse taking care of you today." 3. "I have no further questions. Is there anything else you wish to ask me?" 4. "Tell me what brought you to the hospital." 5. "So, to summarize, you've lost about 6 pounds in the past month, and your appetite has been poor—correct?"

9. Answer: 2, 4, 1, 5, 3. A patient-centered interview begins with a nurse's self-introduction. It then proceeds to an open ended question that allows a patient to tell his or her story about any health concerns. Listening and acknowledging a patient's concerns then allows you to probe for further information. Summarization lets the patient confirm accuracy of your interpretation of data. Finally, you end an interview by telling the patient you are finished, and letting him or her ask any final questions

9. When teaching a patient about transcutaneous electrical nerve stimulation (TENS), which of the following represent an accurate description of the nonpharmacological therapy? (Select all that apply.) 1. Turn TENS on before patient feels discomfort. 2. TENS works peripherally and centrally on nerve receptors. 3. TENS does not require a health care provider order. 4. Remove any skin preparations before attaching TENS electrodes. 5. Placing electrodes directly over or near the pain site works best.

9. Answer: 2, 4, 5. TENS units act on both the central and peripheral nervous systems. The peripheral effect occurs through activation of the neuroreceptors at or near the source of pain; therefore the electrodes should be placed near the site. A TENS unit requires a health care provider's order that identifies the site for the TENS electrode placement. Remove any hair or skin preparations before attaching the electrodes. Then place the electrodes directly over or near the pain site. Turn the transmitter on to the ordered level when the patient feels pain

9. During a home health visit a nurse talks with a patient and his family caregiver about the patient's medications. The patient has hypertension and renal disease. Which of the following findings place him at risk for an adverse drug event? (Select all that apply.) 1. Taking two medications for hypertension 2. Taking a total of eight different medications during the day 3. Having one physician who reviews all medications 4. Patient's health history of renal disease 5. Involvement of the caregiver in helping with medication administration

9. Answer: 2, 4. The patient is at risk for an adverse drug event (ADE) because of polypharmacy and his history of renal disease, which affects drug excretion. Taking two medications for hypertension is common. Having one physician review all medications and involving a family caregiver are desirable and are safety factors for preventing ADEs.

9. A nurse sees an assistive personnel (AP) perform the following intervention for a patient receiving continuous enteral feedings. Which action would require immediate attention by the nurse? 1. Fastening tube to the gown with new tape 2. Placing client supine while giving a bath 3. Monitoring the client's weight as ordered 4. Ambulating patient with enteral feedings still infusing

9. Answer: 2. A patient receiving continuous enteral feedings should never be placed supine because it increases the risk for pulmonary aspiration. If the nurse needs to lay the patient in the supine position, the feedings should be stopped and restarted when the head of the bed is at 45 degrees.

9. Using Maslow's hierarchy of needs, identify the priority for a patient who is experiencing chest pain and difficulty breathing. 1. Self-actualization 2. Air, water, and nutrition 3. Safety 4. Esteem and self-esteem needs

9. Answer: 2. According to Maslow's theory, basic physiological needs are the patient's first priority, especially when a patient is severely dependent physically. In this example, the patient's need for adequate oxygenation (air) is the priority.

9. Which nursing intervention decreases the risk for catheter-associated urinary tract infection (CAUTI)? 1. Cleansing the urinary meatus 3 to 4 times daily with antiseptic solution 2. Hanging the urinary drainage bag below the level of the bladder 3 Emptying the urinary drainage bag daily 4. Irrigating the urinary catheter with sterile water

9. Answer: 2. Evidence-based interventions shown to decrease the risk for CAUTI include ensuring that there is a free flow of urine from the catheter to the drainage bag.

9. As part of a faith community nursing program in her church, a nurse is developing a health promotion program on breast selfexamination for the women's group. Which statement made by one of the participants is related to the individual's accurate perception of susceptibility to an illness? 1. "I have a door hanging tag in my bathroom to remind me to do my breast self-examination monthly." 2. "Since my mother had breast cancer, I know that I am at increased risk for developing breast cancer." 3. "Since I am only 25 years of age, the risk of breast cancer for me is very low." 4. "I participate every year in our local walk/run to raise money for breast cancer research."

9. Answer: 2. On the basis of health belief model, this statement indicates that the patient is concerned about developing breast cancer and feels that there is a risk or susceptibility based on recognition of a familial link for the disease. Once this link is recognized, the patient may perceive the personal risk.

9. The nurse is performing tracheostomy care on a patient. What finding would indicate that the tracheostomy tube has become dislodged? 1. Clear breath sounds 2. Patient speaking to nurse 3. SpO2 reading of 96% 4. Respiratory rate of 18 breaths/minute

9. Answer: 2. Patient phonation is a sign that the TT is not in its proper place. All the other findings are normal assessment findings. Refer to Skill 41.2.

9. Traditional Chinese medicine (TCM) is used by many patients. Which statement most accurately describes intervention(s) offered by TCM providers? 1. Uses acupuncture as its primary intervention modality 2. Uses many modalities based on the individual's needs 3. Uses primarily herbal remedies and exercise 4. Is the equivalent of medical acupuncture

9. Answer: 2. TCM practitioners use a variety of interventions that are based on individual patient assessment findings and needs. Modalities include herbal therapies, acupuncture, moxibustion, cupping, prescribed exercise such as tai chi or qi gong, and lifestyle changes. Although acupuncture is often confused with TCM, when it is used alone, acupuncture is not a whole system of medicine. Rather the National Institutes of Health/National Center for Complementary and Alternative Medicine considers it to be a mind-body technique that is often referred to as medical acupuncture. Although herbal therapies and exercise are considered to be part of the treatment repertoire of the TCM provider, these modalities are not considered to be primary interventions.

9. A married couple has three children. The youngest child just graduated from college and is moving to a different city to take a job. The other two children left the home several years ago. Both of their parents are older and are beginning to need help to maintain their home. What assessment questions will help the nurse determine the family's functioning? (Select all that apply). 1. Which transitions or changes in your family are you currently experiencing? 2. Are your children having any problems that are affecting your family right now? 3. Describe a recent family conflict and how your family resolved it. 4. What coping strategies do you typically use as a family? 5. Who is involved in helping care for your parents?

9. Answer: 3, 4, 5. This family is recently without children and has older parents to assist. Assessment questions about family functioning address how individuals behave in relation to one another, which is reflected in questions that determine how the family completes routine activities (e.g., making meals, doing laundry), communicates, and solves problems

9. The nurse is evaluating how well a patient newly diagnosed with multiple sclerosis and psychomotor impairment is coping. Which statements indicate that the patient is beginning to cope with the diagnosis? (Select all that apply.) 1. "I'm going to learn to drive a car, so I can be more independent." 2. "My sister says she feels better when she goes shopping, so I'll go shopping." 3. "I'm going to let the occupational therapist assess my home to improve efficiency." 4. "I've always felt better when I go for a long walk. I'll do that when I get home." 5. "I'm going to attend a support group to learn more about multiple sclerosis."

9. Answer: 3, 5. Inviting the occupational therapist into the patient's home and attending support groups are early indicators that the patient is recognizing some of the challenges of the disease and participating in positive realistic activities to cope with the stressors related to changes in physical functioning. The other options relate to independence and other coping strategies but do not address coping with the specific challenges of the disease

9. A patient has undergone surgery for a femoral artery bypass. The surgeon's orders include assessment of dorsalis pedis pulses. The nurse will use which of the following techniques to assess the pulses? (Select all that apply.) 1. Place the fingers behind and below the medial malleolus. 2. Have the patient slightly flex the knee with the foot resting on the bed. 3. Have the patient relax the foot while lying supine. 4. Palpate the groove lateral to the flexor tendon of the wrist. 5. Palpate along the top of the foot in a line with the groove between the extensor tendons of the great and first toes

9. Answer: 3, 5. To palpate the dorsalis pedis pulses (located in the feet), ask the patient to relax the foot, and then palpate along the top of the foot in a line with the groove between the extensor tendons of the great and first toes. Placing fingers behind the medial malleolus is a technique for assessing the posterior tibial pulse. Having a patient slightly flex the knee is a technique for assessing the popliteal artery behind the knee. Palpation of the groove lateral to the flexor tendon of the wrist is the technique to assess the radial artery.

9. When providing postmortem care, which actions are necessary for the nurse to complete? 1. Locating the patient's clothing 2. Calling the funeral home 3. Providing culturally and religiously sensitive care in body preparation 4. Providing postmortem care to protect the family of the deceased from having to view the body

9. Answer: 3. A deceased person's body deserves the same respect and dignity as that of a living person and needs to be prepared in a manner consistent with the patient's cultural and religious beliefs

9. A nurse is taking a sleep history from a patient. Which statement made by the patient needs further follow-up? 1. "I feel refreshed when I wake up in the morning." 2. "I use soft music at night to help me relax." 3. "It takes me about 45 to 60 minutes to fall asleep." 4. "I take the pain medication for my leg pain about 30 minutes before I go to bed.

9. Answer: 3. Good sleep-hygiene practices indicate that individuals should fall asleep within 30 minutes of going to bed. Taking 45 to 60 minutes to fall asleep indicates a potential sleep problem and requires follow-up on sleep-hygiene practices. If an individual does not fall asleep within 30 minutes, encourage him or her to get out of bed and do a quiet activity until he or she feels sleepy.

9. A nurse enters a patient's room and begins a conversation. During this time the nurse evaluates how a patient is tolerating a new diet plan. The nurse decides to also evaluate the patient's expectations of care. Which of the following is appropriate for evaluating a patient's expectations of care? 1. On a scale of 0 to 10 rate your level of nausea. 2. The nurse weighs the patient. 3. The nurse asks, "Did you believe that you received the information you needed to follow your diet?" 4. The nurse states, "Tell me four different foods included in your diet."

9. Answer: 3. It is important to evaluate whether you have met a patient's expectations of care. This includes asking about the patient's perceptions of care. Evaluating patient expectations determines the patient's satisfaction with care and strengthens partnering between you and the patient

9. A 15-year-old patient tells the nurse that she is sexually active. What is the best action by the nurse? 1. Contact her parents to alert them of her need for birth control. 2. Explain that having sex is not appropriate for her age-group. 3. Counsel her on safe sex practices and on minimizing health risks. 4. Ask her to have her partner come to the clinic for STI testing.

9. Answer: 3. Nurses need to be sensitive to the emotional cues from adolescents before initiating health teaching to know when the teen is ready to discuss concerns. In addition, discussions with adolescents need to be private and confidential. Adolescents define health in much the same way as adults and look for opportunities to reach their physical, mental, and emotional potential.

9. A nurse is preparing to teach a patient who has sleep apnea how to use a CPAP machine at night. Which action is most appropriate for the nurse to perform first? 1. Allow patient to manipulate machine and look at parts. 2. Provide a teach-back session. 3. Set mutual goals for the education session. 4. Discuss the purpose of the machine and how it works

9. Answer: 3. Planning should occur before any form of implementation or evaluation. The nurse should build from simple to more complex information. Learning about the purpose of the machine and how it works is basic information needed for the patient to understand and be motivated to use it. Allowing the patient to manipulate the machine will precede instruction on its actual use. Teach-back will inform the nurse as to the patient's level of learning.

9. A nurse assesses a 78-year-old patient who weighs 108.9 kg (240 lb) and is partially immobilized because of a stroke. The nurse turns the patient and finds that the skin over the sacrum is Copyright 2021 © by Elsevier, Inc. All rights reserved. very red and the patient does not feel sensation in the area. The patient has had fecal incontinence on and off for the past 2 days. The nurse identifies the nursing diagnosis of Risk for Impaired Skin Integrity. Which of the following outcomes is appropriate for the patient? 1. Patient will be turned every 2 hours within 24 hours. 2. Patient will have normal formed stool within 48 hours. 3. Patient's ability to turn self in bed improves. 4. Erythema of skin will be mild to none within 48 hours

9. Answer: 4. Reduced erythema is the only outcome that measurably assesses condition of patient's skin and within a set time frame. It is realistic. The ability to turn self is an outcome measuring mobility status. Normal formed stools is an outcome focused on improving bowel incontinence. Turning a patient is an intervention and not an outcome.

9. A 34-year-old female executive has a job with frequent deadlines. She notes that when the deadlines appear, she tends to eat highfat, high-carbohydrate foods. She also explains that she gets frequent headaches and stomach pain during these deadlines. After receiving health education from the nurse, the executive decides to try yoga. In this scenario yoga is used as a(n): 1. Outpatient referral. 2. Counseling technique. 3. Health promotion activity. 4. Stress-management technique

9. Answer: 4. Relaxation techniques, such as imagery, biofeedback, and yoga, help recondition the patient's response to stress. Yoga is an ancient practice of controlling body and mind by which there is a physical and mental harmony. It is frequently used as an effective intervention for stress and stress-related physical symptoms.

9. The patient states, "I have diarrhea and cramping every time I have ice cream. I am sure this is because the food is cold." Based on this assessment data, which health problem does the nurse suspect? 1. A food allergy 2. Irritable bowel syndrome 3. Increased peristalsis 4. Lactose intolerance

9. Answer: 4. These symptoms are consistent with lactose intolerance, and they occur with ingestion of dairy products

9. A nurse is caring for a patient who has poor pain control. The patient has a history of opioid abuse. During the day the patient made frequent requests for a pain medication. In order to make an effective clinical decision about this patient, the nurse needs to ask questions about the data available on the patient to make a thorough and thoughtful decision. The nurse asks herself, "How does my view about the patient's pain tolerance compare with the patient's, and does that pose a problem?" This is an example of: 1. A question about assumptions. 2. A question about evidence. 3. A question about procedure. 4. A question about perspective.

9. Answer: 4. Thoughtful clinical decision making involves asking questions about your perspective, such as how does your own view compare with that of the patient

9. Place the steps of administering an intradermal injection in the correct order. 1. Inject medication slowly. 2. Note the presence of a bleb. 3. Advance needle through epidermis to 3 mm. 4. Using nondominant hand, stretch skin over site with forefinger. 5. Insert needle at a 5- to 15-degree angle into the skin until resistance is felt. 6. Cleanse site with antiseptic swab.

9. Answer: 6, 4, 5, 3, 1, 2. This is the correct sequence of steps to administer an intradermal injection

9. A nursing assessment reveals a patient in the home setting who has reduced mobility following recovery from a stroke. The patient has weakness in the left leg and arm. The patient has a walker, which he has never used before, and his wife tells the nurse that he is unsteady in using the walker. The patient fell while in the hospital. The physical therapist came to the home, but the wife tells the nurse, "We are not sure how to get my husband upstairs. The therapist explained how to use the walker, but we have questions." The nurse developed the following concept map. Place the links between the nursing diagnoses in the correct direction

9. Answer: Arrows should go directly from Impaired Mobility to Risk for Fall. The patient's weakness after a stroke places him at risk for falling in the home. Arrows should go directly from Lack of Knowledge to Risk for Fall. The patient's uncertainty on how to use the walker and how to ascend floors adds risk for falling. Arrows should go directly from Lack of Knowledge to Impaired Mobility because having uncertainty on how to use a walker might further impair the patient's ability to ambulate

Chapter 1 1. You are preparing a presentation for your classmates regarding the clinical care coordination conference for a patient with terminal cancer. As part of the preparation you have your classmates read the Nursing Code of Ethics for Professional Registered Nurses. Your instructor asks the class why this document is important. Which statement best describes this code? 1. Improves self-health care 2. Protects the patient's confidentiality 3. Ensures identical care to all patients 4. Defines the principles of right and wrong to provide patient care

Answer: 4. When giving care, it is essential to provide a specified service according to standards of practice and to follow a code of ethics. The code of ethics is the philosophical ideals of right and wrong that define the principles you will use to provide care to your patients. It serves as a guide for carrying out nursing responsibilities to provide quality nursing care and as a guide to the ethical obligations of the profession.

5. Contemporary nursing requires that the nurse has knowledge and skills for a variety of professional roles and responsibilities. Which of the following are examples of these roles and responsibilities? (Select all that apply.) 1. Caregiver 2. Autonomy 3. Patient advocate 4. Health promotion 5. Genetic counselor

Answer: 1, 2, 3, 4. Each of these roles or skills includes activities for the professional nurse. Each of these is used in direct care or is part of the professionalism that guides nursing practice.

Which of the following are examples of a nurse participating in primary care activities? (Select all that apply.) 1. Providing prenatal teaching on nutrition to a pregnant woman during the first trimester 2. Assessing the nutritional status of older adults who come to the community center for lunch 3. Working with patients in a cardiac rehabilitation program 4. Providing home wound care to a patient 5. Teaching a class to parents at the local grade school about the importance of immunizations

Answer: 1, 2, 5. Primary care activities are focused on health promotion. Health promotion programs contribute to quality health care by helping patients acquire healthier lifestyles. Health promotion activities help keep people healthy through exercise, good nutrition, rest, and adopting positive health attitudes and practices.

7. Health care reform will bring changes in the emphasis of care. Which of these models is expected from health care reform? 1. Moving from an acute illness to a health promotion, illness prevention model 2. Moving from an illness prevention to a health promotion model 3. Moving from hospital-based to community-based care 4. Moving from an acute illness to a disease management model

Answer: 1. Health care reform also affects how health care is delivered. There is greater emphasis on health promotion, disease prevention, and management of illness

5. A nurse is providing restorative care to a patient following an extended hospitalization for an acute illness. Which of the following is an appropriate goal for restorative care? 1. Patient will be able to walk 200 feet without shortness of breath. 2. Wound will heal without signs of infection. 3. Patient will express concerns related to return to home. 4. Patient will identify strategies to improve sleep habits

Answer: 1. Restorative interventions focus on returning patients to their previous level of function or on reaching a new level of function limited by their illness or disability. The goal of restorative care is to help individuals regain maximal functional status and to enhance quality of life through promotion of independence

3. The nurse spends time with a patient and family reviewing a dressing change procedure for the patient's wound. The patient's spouse demonstrates how to change the dressing. The nurse is acting in which professional role? 1. Educator 2. Advocate 3. Caregiver 4. Communicator

Answer: 1. The nurse is demonstrating the role of educator. An educator explains concepts and facts about health, describes the reason for routine care activities, demonstrates procedures such as home-care activities, reinforces learning or patient behavior, and evaluates the patient's progress in learning through return demonstration.

6. Match the advanced practice nurse specialty with the statement about the role. 1. Clinical nurse specialist 2. Nurse anesthetist 3. Nurse practitioner 4. Nurse-midwife a. Provides independent care, including pregnancy and gynecological services b. Expert clinician in a specialized area of practice such as adult diabetes care c. Provides comprehensive care, usually in a primary care setting, directly managing the medical care of patients who are healthy or have chronic conditions d. Provides care and services under the supervision of an anesthesiologist

Answer: 1b, 2d, 3c, 4a. The role statements describe the activities performed and the role of the advanced practice nurse specialty. Nurse midwives care for women who are pregnant or have women's health needs. Clinical nurse specialists typically see hospitalized patients with a specific type of illness or health problem. Nurse practitioners usually practice in a primary care setting and care for patients who are healthy or have minor acute or stable chronic conditions. Certified nurse anesthetists care for patients during the surgical experience and administer anesthesia during surgery.

2. A nurse is caring for a patient with end-stage lung disease. The patient wants to go home on oxygen and be comfortable. The family wants the patient to have a new surgical procedure. The nurse explains the risk and benefits of the surgery to the family and discusses the patient's wishes with them. The nurse is acting as the patient's: 1. Educator. 2. Advocate. 3. Caregiver. 4. Communicator

Answer: 2. An advocate protects the patient's human and legal right to make choices about his or her care. An advocate may also provide additional information to help a patient decide whether to accept a treatment or may find an interpreter to help family members communicate their concerns

9. A critical care nurse is using a new research-based intervention to correctly position her ventilated patients to reduce pneumonia caused by accumulated respiratory secretions. This is an example of which Quality and Safety in the Education of Nurses (QSEN) competency? 1. Patient-centered care 2. Evidence-based practice 3. Teamwork and collaboration 4. Quality improvement

Answer: 2. The use of a research intervention to improve patient care brings the evidence-based practice gained from the research to the bedside

4. Which of the following nursing activities is provided in a secondary health care environment? 1. Conducting blood pressure screenings for older adults at the Senior Center 2. Teaching a patient with chronic obstructive pulmonary disease purse-lipped breathing techniques at an outpatient clinic 3. Changing the postoperative dressing for a patient on a medical-surgical unit 4. Doing endotracheal suctioning for a patient on a ventilator in the medical intensive care unit

Answer: 3. In secondary care the diagnosis and treatment of illnesses are traditionally the most common services. Secondary services are usually provided in an acute care setting. Inpatient medical-surgical units provide secondary care. Critical care units provide tertiary care

4. The examination for registered nurse (RN) licensure is the same in every state in the United States. This examination: 1. Guarantees safe nursing care for all patients. 2. Ensures standard nursing care for all patients. 3. Provides a minimal standard of knowledge for an RN in practice. 4. Guarantees standardized education across all prelicensure programs.

Answer: 3. RN candidates must pass the NCLEX-RN® to attain licensure. Regardless of educational preparation, the examination for RN licensure is exactly the same in every state in the United States

8. A nurse meets with the registered dietitian and physical therapist to develop a plan of care that focuses on improving nutrition and mobility for a patient. This is an example of which Quality and Safety in the Education of Nurses (QSEN) competency? 1. Patient-centered care 2. Safety 3. Teamwork and collaboration 4. Quality improvement

Answer: 3. This is an example of the competency of teamwork and collaboration. This competency focuses on the nurse functioning effectively within nursing and interprofessional teams, fostering open communication, mutual respect, and shared decision-making to achieve quality patient care.

Which of the following statements is true regarding Magnet® status recognition for a hospital? 1. Nursing is run by a Magnet manager who makes decisions for the nursing units. 2. Nurses in Magnet hospitals make all of the decisions on the clinical units. 3. Magnet is a term that is used to describe hospitals that are able to hire the nurses they need. 4. Magnet is a special designation for hospitals that achieve excellence in nursing practice.

Answer: 4. Magnet® status is a process and review that hospitals go through that shows achievement of excellence in nursing practice. The designation is given by the American Nurses Credentialing Center and focuses on demonstration of quality patient care, nursing excellence, and innovations in professional practice.

Chapter 2 1. Which activity performed by a nurse is related to maintaining competency in nursing practice? 1. Asking another nurse about how to change the settings on a medication pump 2. Regularly attending unit staff meetings 3. Participating as a member of the professional nursing council 4. Attending a review course in preparation for a certification examination

Answer: 4. Maintaining ongoing competency is a nurse's responsibility. Earning certification in a specialty area is one mechanism that demonstrates competency. Specialty certification has been shown to be positively related to patient safety.

10. The nurses on an acute care medical floor notice an increase in pressure injury formation in their patients. A nurse consultant decides to compare two types of treatment. The first is the procedure currently used to assess for pressure injury risk. The second uses a new assessment instrument to identify at-risk patients. Given this information, the nurse consultant exemplifies which career? 1. Clinical nurse specialist 2. Nurse administrator 3. Nurse educator 4. Nurse researcher

Answer: 4. The nurse researcher investigates problems to improve nursing care and to further define and expand the scope of nursing practice. He or she often works in an academic setting, hospital, or independent professional or community service agency.

Chapter 25 1. A patient asks a nurse to provide instruction on how to perform a breast self-exam. Which domains are required to learn this skill? (Select all that apply.) 1. Affective domain 2. Sensory domain 3. Cognitive domain 4. Attentional domain 5. Psychomotor domain

Answers and Rationales for Review Questions 1. Answer: 3 and 5. For a patient to perform a breast self-exam it will be necessary to understand the purpose of the exam and why it is performed the way it is. Cognitive learning in this scenario involves a patient acquiring information to further develop his or her understanding and thinking processes so that the patient can make a decision based on a self-exam finding. Psychomotor learning in this case involves actual use of the hands to palpate in symmetrical areas of the breast correctly. Affective domain does not apply unless the nurse decides the patient's values prevent self-exam adherence. There are no attentional or sensory learning domains


Conjuntos de estudio relacionados

EAQ Psych - Foundations and Modes of Care

View Set

Course 9: Lesson 5: Automated Underwriting

View Set

Psychology Chapter 10, 11, 13 Vocabulary Review

View Set

assignments/missed practice test ?

View Set